0% found this document useful (0 votes)
12 views

Ray Optics note

The document provides an overview of optics, focusing on ray optics and wave optics, and discusses the behavior of light at the interface of two media, including reflection, refraction, and absorption. It details the characteristics and definitions related to spherical mirrors, including types, focal length, and the new Cartesian sign convention. Additionally, it explains the mirror formula applicable to both concave and convex mirrors, along with rules for image formation and ray diagrams.

Uploaded by

spotify.75g
Copyright
© © All Rights Reserved
We take content rights seriously. If you suspect this is your content, claim it here.
Available Formats
Download as PDF, TXT or read online on Scribd
0% found this document useful (0 votes)
12 views

Ray Optics note

The document provides an overview of optics, focusing on ray optics and wave optics, and discusses the behavior of light at the interface of two media, including reflection, refraction, and absorption. It details the characteristics and definitions related to spherical mirrors, including types, focal length, and the new Cartesian sign convention. Additionally, it explains the mirror formula applicable to both concave and convex mirrors, along with rules for image formation and ray diagrams.

Uploaded by

spotify.75g
Copyright
© © All Rights Reserved
We take content rights seriously. If you suspect this is your content, claim it here.
Available Formats
Download as PDF, TXT or read online on Scribd
You are on page 1/ 102

□ □□□

□ □□
□u
CHAPTER
u

Ray OpHcs and


□pHcal Instruments

9.1 INTRODUCTION 9.2 BEHAVIOUR OF LIGHT AT THE


INTERFACE OF TWO MEDIA
1. What is optics ? What are its two main branches ?
Optics. It is the branch of physics which deals with the 2. Name the different effects that may occur as light
study of nature, production and propagation of light. The travels from one optical medium to another. State the
laws of reflection of light.
subject of optics can be divided into two main branches:
rays optics and wave optics. Behaviour of light at the interface of two media.
1. Ray or geometrical optics. It concerns itself with When light travelling in one medium falls on the
surface of a second medium, the following three effects
the particle nature of light and is based on
may occur:
(i) the rectilinear propagation of light and
(z) A part of the incident light is turned back into the
(li) the laws of reflection and refraction of light. first medium. This is called reflection of light.
It explains the formation of images in mirrors and
lenses, the aberrations of optical images and the Normal
working and designing of optical instruments.
Incident
2. Wave or physical optics. It concerns itself with ray
the wave nature of light and is based on the pheno Medium 1

mena like

(i) interference
(ii) diffraction and Reflecting
surface
{Hi) polarisation of light.
Medium 2
In the previous chapter, we have learnt that light is
an electromagnetic wave in which the electric and Refracted

magnetic fields vary harmonically in space and time. ray

The visible light consists of waves with wavelengths


ranging from 4000 A to 7500 A Fig. 9.1 Reflection and refraction of light.
(9.1)
PHYSICS-XII

{ii) A part of the incident light is transmitted into Spherical mirrors are of two types :
the second medium along a changed direction. (/) Concave mirror. A spherical mirror in which the
This is called refraction of light. outer bulged surface is silvered polished and the reflection of
{Hi) The remaining third part of light energy is light takes place from the inner hollow surface is called a
absorbed by the second medium. This is called concave mirror.

absorption of light
(ii) Convex mirror. A spherical mirror in which the
Laws of reflection of light. Reflection of light takes inner hollow surface is silvered polished and the reflection of
place according to the following two laws : light takes place from the outer bulged surface is called a
{/) The angle of incidence is equal to the angle of convex mirror.

reflection, i.e., .Zi = Zr.


4. Define pole, centre of curvature, radius of curvature,
(ii) Tlie incident ray, the reflected ray and the principal axis, linear aperture, angular aperture, principal
nonnal at the point of incidence all lie in the focus, focal length, and focal plane of a spherical mirror.

w
same plane. Definitions in Connection with Spherical Mirrors.
Normal In Fig 9.4, let APB be a principal section of a spherical
Incident Reflected

Flo Silvered
Reflecting

ee
s surface
surface
Aperture

Fr
Mirror F
IS Principal axis
Focus
for
Pole Centre of '
ur
curvature ,

Fig. 9.2 The incident ray, reflected ray and the normal to the Radius of
curvature
ks

reflecting surface lie on the same plane.


Yo

The above laws of reflection are valid both in case


oo

of plane and curved reflecting surfaces. Fig. 9.4 Characteristics of a concave mirror,
B

9.3 SPHERICAL MIRRORS mirror, i.e., the section cut by a plane passing through
re

pole and centre of curvature of the mirror.


3. What are spherical mirrors ? What are their two
types ?
1. Pole. It is the middle point P of the spherical mirror.
ou
ad

2. Centre of curvature. It is the centre C of the sphere


Spherical Mirrors. As shown in Fig. 9.3(a), consider of which the mirror forms a part.
a hollow glass sphere being cut by a plane. The section
Y

APB, cut by the plane, forms a part of a sphere and is 3. Radius of curvature. It is the radius (R = AC
known as a spherical surface. If either side of this or BC)of the sphere of zvhich the mirrorforms a part.
nd
Re

spherical surface is silvered, we get a spherical mirror. 4. Principal axis. The line PC passing through the
A spherical mirror is a reflecting surface which afrms pole and the centre of curvature of the mirror is called
Fi

part of a hollow sphere. its principal axis.


Reflecting 5. Linear aperture. It is the diameter AB of the
surfaces circular boundary of the spherical mirror.
6. Angular aperture. It is the angle ACB subtended
by the boundary of the spherical mirror at its centre
- Silvered
surface
of curvature C.
7. Principal focus. A narrow beam of light parallel to
Vi the principal axis either actually converges to or
appears to diverge from a point F on the principal
{b) {c) axis after reflection from the spherical mirror. This
point is called the principal focus of the mirror. A
Fig. 9.3 (a) A hollow sphere cut by a plane concave mirror has a real focus while a convex
(b) concave mirror (c) convex mirror. mirror has a virtual focus, as shown in Fig. 9.5.
RAY OPTICS AND OPTICAL INSTRUMENTS 9.5

Rea! focus
2. All distances are measured from the pole of the
mirror.

3. All distances measured in the direction of


incident light are taken to be positive.
4. All distances measured in the opposite direction
of incident light are taken to be negative.
5. Heights measured upwards and perpendicular
(«)
to the principal axis are taken positive.
6. Heights measured downwards and perpen
Virtual focus
dicular to the principal axis are taken negative.

./
Mirror
Heights
F C
upwards positive

Object Incident light \


on left
-► j-axis

Distances against
incident light y Distances along
Fig. 9.5 Principal focus of (a) a concave mirror Heights negative y incident light
downwards
(b) a convex mirror. positive
negative

8. Focal length, It is the distance (/ = PF) between Fig. 9.7 New Cartesian sign convention.
the focus and the pole of the mirror. According to this sign convention, the focal length and
9. Focal plane. The vertical plane passing through the radius of curvature are negative for a concave mirror
principal focus and perpendicular to the principal and positive for a convex mirror.
axis is called afcal plane. When a parallel beam of
light is incident on a concave mirror at a small 6. Derive a relationship between the focal length and
angle to the principal axis, it is converged to a radius of curvature of a spherical mirror.
point in the focal plane of the mirror, as shown Relation between f and R. Consider a ray AB
in Fig. 9.6. parallel to the principal axis, incident at point B of a
spherical mirror (concave or convex) of small aperture.
After reflection from the mirror, this ray converges to
point F (in case of a concave mirror) or appears to
B

C F P

(«)
Fig. 9.6 Focal plane of concave mirror.

NOTE A line joining any point of the spherical


A
mirror to its centre of curvature, will be normal to the
mirror at that point. (b)

5. State the new cartesian sign convention used for P F C

spherical mirrors.
New Cartesian Sign Convention for Spherical
Mirrors. According to this sign convention : K R

1. All ray diagrams are drawn with the incident Fig. 9.8 Relation between / and R of
light travelling from left to right. (fl) a concave mirror (b) a convex mirror.
9.4 PHYSICS-Xil

diverge from point f (in case of a convex mirror), principal fbais in the case of a convex mirror will [Fig. 9.10(b)],
obeying the laws of reflection. Thus f is the focus of the after reflection, become parallel to the principal axis.
mirror, C is the centre of curvature, CP = the radius of
curvature and BC is a normal to mirror at point B.
According to the law of reflection,
\
Zi = Zr
C
As AB is parallel to PC, F

Za = Zi

In A BFC, Zr=Za

Hence CF = FB
Fig. 9.10 (a) A ray through F becomes parallel to the principal
For a mirror of small aperture, axis after reflection from a concave mirror.

w
FB- FP CF-FP
Hence CP=CF+ FP=FP+ FP=2FP
R
R=2f
^ 2
or or

or Focal length = -i x Radius of curvature


Flo
ee
F C

7. State the rules used for drawing images formed by

Fr
spherical mirrors. Draw ray diagrams showing the
formation of images by concave and convex mirrors for
different object positions on the principal axis.
Fig. 9.10 (b) A ray directed through f becomes parallel
for
to the
ur
Rules for drawing images formed by spherical principal axis after reflection from a convex mirror.
mirrors. The position of the image formed by spherical
mirrors can be found by considering any two of the (Hi) A ray passing through the centre of curvature in the
ks

following rays of light coming from a point on the object. case of concave mirror [Fig. 9.11(fl)], and directed towards
Yo

(/) A ray proceeding parallel to the principal axis will, the centre of curvature in the case of a convex mirror
oo

after reflection, pass through the principalfocus in the case of [Fig. 9,n(b)]/fl//s normally (Zi = Zr = 0°) and is reflected
a
concave mirror [Fig. 9.9(a)], and appear to come from back along the same path.
B

focus in the case of a convex mirror [Fig. 9.9(b)].


re
ou
ad

c F
Y

/
nd
Re

Fig. 9.9 (a) A ray parallel to the principal axis passes through F Fig. 9.11 (a) A ray passing through C is reflected back along
Fi

after reflection from a concave mirror. of same path after reflection from a concave mirror.

F C
P F C

Fig. 9.9 (6) A ray parallel to the principal axis appears to


V
come from F after reflection from a convex mirror.
Fig. 9.11 (b) A ray directed towards C is reflected back along of
same path after reflection from a convex mirror.
(ii) A ray passing through the principal focus in the case (iv) For the ray incident at any angle at the pole, the
of a concave mirror [Fig. 9.10(<i)], and directed towards the
reflected ray follows the laws of reflection.
RAY OPTICS AND OPTICAL INSTRUMENTS

Formation of Images by Concave Mirrors : Formation of image by Convex Mirror :


(«) Object beyond C The image is For any position of the object between co and pole P,
2. Real
the image is
1. Between C and F
1. Behind the mirror 2. Virtual
3. Inverted 4. Smaller than object.
3. Erect 4. Smaller than object.
A

A
B

O / F C

\
Fig. 9.12 (e)
1. At C

3. Inverted 9.4 THE MIRROR FORMULA


8. State the mirror formula. Is the same formula
applicable to both concave and convex mirrors ? Write the
assumptions used in the derivation of mirror formula.
Mirror formula The mirror formula is a mathematical
relationship between object distance u, image distance v and
the focal length f of a spherical mirror. This relation is
1+1=1
u /
(c) Object between f and C. The image is This formula is applicable to all concave and convex
1. Beyond C 2. Real mirrors, whether the image formed is real or virtual.
3. Inverted 4. Larger than object. Assumptions used in the derivation of mirror formula :
{/) All the rays are paraxial.
(//) The aperture of the mirror is small.
{Hi) The object placed on the principal axis is small
in size.

9(rt). Draw a ray diagram to show the image formation


when the concave mirror forms a real, inverted and diminished
image of an object. Obtain the mirror formula and write the
expression for the linear magnification.
Derivation of mirror formula for a concave mirror
(d) Object between F and P. The image is
when it forms a real, inverted and diminished image
1. Behind the mirror 2. Virtual
Consider an object AB placed on the principal axis
3. Erect 4. Larger than object. beyond the centre
of curvature C of a
B
concave mirror of
%
A
small aperture, as
shown in Fig. 9.13.
P
After reflection of
C
F 0 I light from the
mirror, a real inver

/ ted image A'B' is


formed.

Fig. 9.12 (d) Fig. 9.13


PHYSiCS-XII

Using Cartesian sign convention, we find between C and f of a concave mirror; a real, inverted
Object distance. BP = -m and magnified image A'B' is formed beyond C on the
same side as the object.
Image distance. BP = -v

Focal length, FP = -f
Radius of curvature. CP = -R=-2f
Now AA'B'C~AABC

A'B' _ CB' CP-B'P - R + v


...(1)
AB BC~ BP-CP -u+ R

As Z A'PB'= Z APB, therefore, AA'B'PAABP.


A'B' _ B'P

w
-V _v
Consequently, ...(2)
AB ~ BP -u u
AABP- AA'B'P

From equations (1) and (2), we get AB _ BP


...(1)
-R + v
-u+ R
V
u
or ~uR + uv =-uv + vR

Flo A'B'~ B'P


A\so,AMNF- AA'B'P.

ee
or vR + uR= 2 uv MN _ FN
A'B~B'F

Fr
Dividing both sides by uvR, we get
For a mirror of small aperture, N is close to P.
1 + 1-.^ But R=2f l + i=l
u V R FN ^ FP
■ V f
for
u
ur
Also, MN=AB
This proves the mirror formula for a concave
mirror, when it forms a real image. AB ^ FP FP
...(2)
ks

A'B' B'F B'P-FP


Linear magnification. The ratio of the height of the
Yo

image to that of the object is called linear or transverse From (1) and (2), we get
oo

magnification or just magnification and is denoted BP FP


by m.
B

B'P B'P-FP

Height of image _ h2 Using Cartesian sign convention, we find


re

m =

Height of object h1 BP = u, B'P = ■V, FP = -f


ou

In Fig 9.13, AAPB~AA'PB' -/ /


ad

-u u
or

A'B' _ B'P
-V V
v-f
Y

AB~ BP or
uv-uf = vf
Applying the new Cartesian sign convention, we Dividing each term by uvf, we get
nd
Re

get 1 1 1
or 1 ^2 + 1
A'S'=-/i2, AB= +h V B'P = -v, BP = ~u
Fi

f V u f u V

- V
This proves the minor formula for the concave mirror.
h - u
1
Linear magnification. In Fig 9.14, AAPB- AA'PB'
^2 V
A'B' _ B'P
Magnification, m = -A-
h
1
u
AB~ BP
9(b). Draw a ray diagram to show the image formation Using Cartesian sign convention, we find
lohen the concave mirror forms a real, inverted and A'B'=-h2, AB = +h, B'P = -v, BP=-u
magnified image of an object. Obtain the mirror formula and ~v
write the expression for linear inagnification.
-u
Derivation of mirror formula for a concave mirror
when it forms a real, inverted and magnified image As Linear magnification, m = i-
V

shown in the ray diagram, when an object AB is placed h


1
u
RAY OPTICS AND OPTICAL INSTRUMENTS 9.7

9(c). Draw a ray diagram showing how a concave forms Linear magnification. In Fig. 9.15, AMPF - AA'B'F
a virtual, erect and magnified image of an object. Obtain the AB' _ FB' A'B' FP+PB'
or
mirror formula and write the expression for linear AB FP
MP FP
magnification.
Derivation of Mirror formula for a concave
Using Cartesian sign convention, we find
mirror when the image formed is virtual Consider an A'B'=+h^, AB = + h^, FP = -/, PB’=+v
object AB placed on the principal axis of a concave -f+v
mirror (of small aperture) between its pole P and focus F. h
1 -/

tn==^ = - — =--. [Using mirror formula]


A'
or
h
1 / u

M
10. Draw a ray diagram showing image formation of an
object by a convex mirror. Obtain mirror formula and write
C F B P B' the expression for the linear magnification.
Derivation of mirror formula for a convex mirror
u
Consider an object AB placed on the principal axis of a
convex mirror of small aperture, as shown in Fig. 9.16.
V

R
After reflection of light from the mirror, a virtual and
erect image A'B is formed behind the mirror.
Fig. 9.15

As shown in Fig. 9.15, a virtual and erect image A'B'is


formed behind the mirror, after reflection from the A

concave mirror.
A'
Using the Cartesian sign convention, we find that
BP^-u P B' F c
Object distance. B

Image distance. P6'=u


Q
FP = -/ H- u m V v-*i
Focal length, ■M
1+

Radius of curvature, CP~ - R=-2 f H- 2/ ●w

Now AABC ~ AA'B'C, therefore Fig. 9.16


AB _ CB _ CP- BP -2/+H ...(1) Using Cartesian sign convention, we find
~^'’'CB'~ CP+ PB'~-2f+v
Object distance. BP = -u
Also AMPF ~ AA'B'F, therefore,
Image distance. PB’^ + v
MP _ FP _ FP
Focal length, FP = + f
A'B'~ FB'~ FP+PB'
Radius of curvature. PC = + R = + 2/
AB -/ ...(2)
or Now AA' B'C ~ AABC
A'B -f + v
A'B' ^ B'C PC-PB'_ R-v ...(1)
From equations (1) and (2), we get ~AB~~^~ BP+ PC~ -u+ R
-2f+u_ -f
As ZA'PB'=ZBPQ^ZAPB
-2 f +v -f + v
Therefore, AA'B'PA ABP.
or 2f-fu-2fv+uv=2f- fv A'B' _ PB' V
Consequently, ...(2)
or -fv-fu + uv=0 or uv = fv+fu AB ~ BP -u

1=1 + 1 From equations (1) and (2), we get


Dividing both sides by uvf, we get R-v V
f ll V
or -uR + uv =-uv + vR
-u+ R -u

This proves the mirror formula for a concave mirror


when it forms a virtual image. or vR + uR-2 uv
9.8 PHYSiCS-XII

Dividing both sides by iivR, we get 9.5 SPHERICAL ABERRATION


A + l=l But R=2f
1
+
1
-
1
12. WJtat is spherical aberration in spherical mirrors ?
u V R u V
/ How can it be reduced ?
This proves the mirror formula for a convex mirror. Spherical aberration. The inability of a spherical
Linear magnification. In Fig. 9.16, mirror of large aperture to bring all the rays of wide beam of
A'B' PB' light falling on it to focus at a single point is called spherical
AA'B'P-^AABP
AB BP aberration. As shown in Fig. 9.17(a), only the paraxial
rays are focussed at the principal focus F. The marginal
Applymg the new Cartesian sign convenhon, we get rays meet the principal axis at a point closer to tlie pole
A'B'=+h^, AB = +hy PB'=+v, BP = - u than the principal focus. The different rays are
V
reflected on to surface known as the caustic curve. This
results in blurred image of the object.

w
Marginal ray
>■
V
Magnification,
h ii
1

11. Write the expressions for the linear magniifcation


produced by a spherical mirror in terms of u, v and f. Flo Paraxial ray

ee
Linear Magnification in terms of ii and f. The

Fr
mirror formula is
7/
- + i= ^ Caustic
U V
/ Large concave
for
curve
ur
spherical mirror
Multiplying both sides by u, we get Fig. 9.17 (a) Spherical aberration in a spherical mirror.
u ll f~u
ks

or 1 Spherical aberration can be reduced by following


^ / V
/ / methods ;
Yo
oo

m= - 1-^ 1. By using spherical mirrors of small apertures.


u f-u 2. By using stoppers so as to cut off the marginal rays.
B

Linear magnification in terms of v and /. As 3. By using parabolic


re

mirrors. Marginal Ray


1 1 ^ j_
U
As shown in Fig. 9.17(&),
ou
ad

Multiplying both sides by v, we get a parabolic mirror focusses Point focus


Y

V all the rays in a wide Paraxial ray


or _2
u
/ u
/ / parallel beam to a single
point on the principal axis *■
nd
Re

m= -
V ^ f-V and thus spherical aberra >■ Concave
tion is reduced. parabolic mirror
M
/
Fi

Fig. 9.17 (fe) No spherical aberration


For Your Knowledge 9.6 USES OF in a parabolic mirror.
> The same mirror formula is valid for both concave CURVED MIRRORS
and convex mirrors whether the image formed is real
or virtual. 13. Give some uses of spherical and parabolic mirrors.
Uses of concave mirrors :
> If Im 1 >1 the image is
> If I m I< I, the image i.s diminished. 1. A concave mirror is used as shaving or make-up
> If I i = 1, the image is of the same size as the object. mirror because it forms a magnified and erect
image of the face when it is held closer to the face.
> If m is positive (or v is positive), the image is virtual and
erect . 2. Doctors use concave mirrors as head mirror. The
mirror is strapped to the doctor's forehead and
> If m is negative (or v is negative), the image is real and
light from a lamp after reflection from the mirror is
inverted.
focussed into the throat or ear of the patient.
RAY OPTICS AND OPTICAL INSTRUMENTS

3. A small concave mirror with a small hole at its Examples based on


centre is used in the doctor's ophthalmoscope. The Formation of Images by
doctor looks through the hole from behind the Spherical Mirrors
mirror while a beam of light from a lamp
reflected from it is directed into the pupil of Formulae Used

patient's eye which makes the retina visible. 1. For any spherical mirror, f - Rl 2
4. Concave mirrors are used as reflectors in head 1
2. Mirror formula, - + — =
1 1 2

lights of cars, railway engines, torch lights, etc. u V f R


The source is placed at the focus of a concave ^ f f-v
mirror. The light rays after reflection travel over 3. Magnification, m=—
ii f -ii f
a large distance as a parallel intense beam.
4. Magnification m is -ve for real images and +ve for
Uses of convex mirrors : virtual images.
A convex mirror is used as a rear view mirror in 5. / and R are -ve for a concave mirror and +ve for a
automobiles. The reason is that it always forms a small convex mirror.

and erect image and it has a larger field of view than 6. For a real object u is -ve, v is -ve for real image
that of a plane mirror of the same size. and +ve for virtual image.
Plane mirror
7. Do not give any sign to unknown quantity. The
sign will automatically appear in the final result.
Units Used

The quantities /, u, v, /jj and are all in m or cm


while magnification m has no units.

Example 1. An object is placed (i) 10 cm, (ii) 5 cm in front


Small Field of view
of a concave mirror of radius of curvature 15 an Find the
(a)
position, nature and magnification of the image in each case.
Convex mirror [NCERTJ

Solution. As R is negative for a concave mirror, so


, K 15 = - 7.5 cm
2

(/■) Here object distance, u = -10cm.


T By mirror formula,
Large Field of view
(b) 1 _ J__l. 1_
v~ f u~-7.5 -10
Fig. 9.18 Field of view of
-2.5 _ 1
(a) a plane minor (ft) a convex minor.
~ 7.5 X 10 ~ 30
Uses of parabolic mirrors : or u = - 30 cm

1. A concave parabolic mirror can focus a wide As V is -ve, a real image is formed 30 cm from the
parallel beam to a single point. This property is mirror on the same side as the object.
used by dish antennas to collect and bring to V -30

focus microwave signals from satellites. Magnification, m = - u -10


= -3

2. When a source of light is placed at the focus of a


Tlie image is magnified, real and inverted,
paraboloidal mirror, the reflected beam is
accurately parallel and is thrown over a very (if) Here object distance, « = - 5 cm
large distance. Due to this property, para By mirror formula.
boloidal mirrors are used as reflectors in search
lights, car head lights, etc.
1 1_1__J 1 _-5 + 7.5_ 1
V 7 u"-7.5 -5~ 7.5x5 15
3. They are used in astronomical telescopes of large
i? = +15 cm
aperture for overcoming spherical aberration. or
9.10 PHYSICS-Xll

As V is +ve, a virtual image is formed 15 cm behind


the mirror. V /« 40 10 40 8
Magnification, m = - — ^=3 or V = + 8 cm
u -5
As V is +ve, the image is virtual and erect and is
The image is magnified, virtual and erect. formed at 8 cm behind the mirror.

Example 2. If you sit in a parked car, you glance in the rear 2


h V +8
Magnification, m = — = +0.8
view mirror R=2 in and notice a jogger approaching. If the 1 u -10
jogger is running at a speed of 5 ms“^, how fast is the image
of the jogger moving when the jogger is (a) 39 m (b) 29 m Size of image. h2 = 0.8 X = 0.8 X 5 = 4 cm
(c) 19 m (d) 9 m aivay ? (NCERT)
As the needle is moved farther away from the
Solution. As the rear view mirror is convex, so mirror, the image shifts towards the focus and its size

w
R=+2m, /=R/2=+lm goes on decreasing. When the needle is far off, it
From mirror formula, appears almost as a point image at the focus.
1=1-1 V =
/w Example 4. A square wire of side 3.0 an is placed 25 cm

When,
t?
M = - 39 m.
/ u u-f

Flo away from a concave mirror of focal length 10 an What is


the area enclosed by the image of the wire ? (The centre of the

ee
wire is on the axis of the mirror, with its tzuo sides normal to
lx (-39) = —
39 m the axis). INCERT]

Fr
-39-1 40
Solution. Here, u =-25 cm. / =-10 cm
As the jogger moves at a constant speed of 5 ms"^ As
1 1 1
the position of the jogger after 1 s,
for
/
ur
U V

H = -39 + 5 = -34m
1=1 ^ ^ 1
Position of the image after 1 s,
v~ f u -10 25
ks

i;' =
lx (-34) 34 -5+2 3
Yo

=— m
-34-1 35
oo

50 50

Difference in the position of the image in 1 s is 50


B

or t? = cm

V -v' ■= ^_34 1365-1360 3


re

40 35 1400 V 50 2
Now m~-
5 1 u 3x25 3
m
ou

1400 280
ad

_ Side of image of v^dre (hf)


1 -1
.●. Average speed of the image = Side of square wire (/ij)
Y

ms .
280

For «=-29m, -19 m and -9 m, the speeds of .●. Side of image of wire, fi2
2 2
nd
Re

image will be = — X h, = — x3=-2cm


1 -1 1 -1 .1
3 ^ 3
-1
ms , — ms and — ms
respectively.
Area enclosed by the image of wire =(2)^ =4 cm^.
Fi

150 60 10

The speed becomes very high as the jogger 5. A concave mirror of focal length 10 an is
approaches the car. The change in speed can be expe- plt^ced at a distance of 35 cm from a wall. How far from the
rienced by anybody while travelling in a bus or a car. should an object be placed to get its image on the wall ?
Example 3. A 5 cm long needle is placed 10 cm from a Wall
A
convex mirror of focal length 40 cm. Find the position,
nature and size of the image of the needle. What happens to A'
the size of image when the needle is moved farther away from B,
P
the mirror ? [CBSE SP 11]
F
Solution. Here h.^ = +5 cm,
u = -10 cm, B'

/ =+40 cm
Fig. 9.19
RAY OPTICS AND OPTICAL INSTRUMENTS 9J1

Solution. Here, / = -10 cm, i; = - 35 cm Solution. The image I of the object O formed by
From mirror formula,
plane mirror should be at 24 cm behind the mirror or
12 cm behind the convex mirror. For no parallax
1 ^_1 1 1 1 between tlie images formed by the two mirrors, the
/ 10 35 14
u
image formed by the convex mirror should also lie at /.
or i/ = -14 cm Therefore, for convex mirror
[/ = OP = -36 cm ; i? = P/ = + 12 cm
Distance of the object from wall
=35-14 = 21 cm.
1-1+1= Jl 1_-i1i1=1-
Example 6. An object is placed at a distayice of 40 an on / u V 36 12 " 36 18
the principal axis of a concave mirror of radius of curvature or
/ = 18 cm
30 an By how much does the image move if the object is Radius of curvature of convex mirror = 36 cm.
shifted towards the mirror through 15 cm ?
Solution. In first case : Example 8. A concave mirror produces a two times
enlarged virtual image of an object placed 15 cm away from
w = - 40 cm, R = - 30 cm or / = -15 cm the mirror,
From mirror formula, {a) Find the focal length of the mirror,
1-1_1=-1_ 1- (b) By how much distance should the object be displaced
v
/ « " 15 40 and in what direction, in order to get two times
enlarged real image of the object ? [CBSE 20C]
1

24
or V =-2A cm Solution, (a) For virtual image : m = +2, u = -15 cm

In second case : The object is shifted towards the m =


/ 2 =
/
mirror by 15 cm, so f-u / + 15
w’=-(40-15) = -25 cm / = 30 cm.
From mirror formula.
(b) For real image : m = ●2, / =-30 cm
1
, / -30
V' / u'~ 15 25 “ 75 m = => -2 =
-30-h'
f-u'
or v' -- 37.5 cm
=> 60+2iP=-30
Distance through which the image shifts w'=-45cm
= ij' —V =~ 37.5 + 24 = -13.5 cm
Displacement of the object =»'-» = -45 -(-15)
i.e., the image shifts 13.5 cm farther from the mirror.
= -30 cm i.e., 30 cm away from the mirror.
Example 7. An object is placed at a distance of 36 anfrom
a convex mirror. A plane mirror is placed in between so that Example 9. An object is placed in front of a concave mirror
the two virtual images so formed coincide. If the plane mirror offocal length of 12 cm. There are two possible positions _ of
is at a distance o/24 cm from the object, find the radius of the object for which the image formed ,s three tunes the size
curvature of the convex mirror. of the object,
(a) Draw the ray diagram for the each case, and
(b) Find the distance between the two positions of the
object. (CBSE OD 20, 20C)

Solution. Magnification,
m =
/ or
-12
= ±3
f-u -12-»
1 c

.-. £<=-16 cm or -8 cm

{a) For real image : m = -3. u = -16 cm

1^12 cm-4^12 cm-4 For virtual image : m = + 3, 11=-8 cm

Fig. 9.20 The two ray-diagrams are shown on next page.


PHYSICS-XIl

A'

I
.4

B' c
i
B'
^12 an
16 an
◄-
A'

(0 Fonnadon of real image (li) Formation of virtual image

Fig. 9.21

w
(i>) Distance between the two positions But h^=3h\, /i2=^2' / = “20cm, Uj =-50cm
=-8-(-16) = 8 cm. 1 _ -20-tf2 or u -30 cm.
Example 10. A concave mirror forms a real image of an 3 -20 + 50
2 ”

Flo
object kept at a distance 9 cm from it. If the object is taken Example 12. >1 thin rod of length f/3 is placed along the
away from the mirror by 6 cm, the image size reduces to |f/i optic axis of a concave mirror of focal length f such that its

ee
of its previous size. Find the focal length of the mirror. image which is real and elongated, just touches the rod.
(CBSE OD 20] will be the magnification ?

Fr
[HT 91]
f Solution. The image of the rod placed along the
Solution. Magnification,
f-u optical axis will touch the rod only when one end of
the rod AC is at the centre of curvature of the concave
for
ur
f mirror( PC =2/, AC = //3).Then the image of the end
In^rsf case, u = - 9 cm m =

/+9 C of the rod will be formed at the same point C.


ks

In second case, u = -15 cm :. m' = —-—


Yo

/ + 15 K//3H
oo

A' Rod
, 1 C /i
P
But w =-m
B

4 Image ^ 2/
re

/ + 15 4 / +9 Fig. 9.22
ou
ad

or
3/(/ + 7) = 0 For the end A of the rod, we have

= PA=FC-AC=2 ^f~L^lL
Y

f ~-7 cm. r/^0] u


3 3
Example 11. Two objects Pand Q when placed at different From mirror formula, —
1 1 1 1 3 2
nd
Re

positions in front of a concave mirror offocal length 20 cm, ^ / u / 5/ 5/


form real images of equal size. Size of object P is three times Thus, the image of A is formed at A' at a distance
Fi

size of object Q.Ifthe distance of Pis 50 cm from the mirror, 5//2 from the pole P{PA' =5 f H).
find the distance ofQfrom the mirror. [CBSE OD 20] Length of the image
Solution. For object P, = A'C = PA'-PC =
5/ /
-^-2/ = -^.
2 ■' 2
^2 f
m = ~ - —-—
Magnification = CA! ^f/2 = 1.5.
h, f-u^ CA f/3
For object Q, robiems For Practice
/ ^
m=-4- f■'
h:1 1. An object is kept 20 cm in front of a concave mirror
/-“2 of radius of curvature 60 cm. Find the nature and
position of the image formed. ICBSE OD 20]
m
_/-»2
m' (Ans. Virtual, erect image at 60 cm
ftj h:^ f-u^ behind the mirror)
RAY OPTICS AND OPTICAL INSTRUMENTS 9.13

2. An object is placed at a distance of 15 cm from a 7. Here « = -60cm.

convex mirror and image is formed at a distance of In first case,


5 cm from tlie mirror. Calculate the radius of V
m = —
curvature of the mirror. (Ans. 15.0 cm) u

3. A candle flame 3 cm high is placed at a distance of 1 V


or = + 30 cm
3 m from a wall. How far from the wall must a con 2 -60
cave mirror be placed so that it may form 9 cm high
Now
1 1 1_ Jl J_ = J.
image of the flame on the same wall ? Also find the 60 30 60
/ u V
focal length of the mirror. (Ans. 4.5 m, - 1.125 m)
4. A dentist concave mirror has a radius of curvature
or / = + 60 cm
of 30 cm. How far must it be placed from a small hi second case,
cavity in order to give a virtual image magnified in =
1 V
or V--
u

five times ? (Ans. 12 cm) 3 u 3

5. Calculate the distance of an object of height h from a A 1 ^ 1 1 3 1


or u - -120 cm.
As - + -
concave mirror of radius of curvature 20 cm, so as H V / u u 60
to obtain a real image of magnification 2. Find the
location of image also. [CBSE 08.16] 8. Refer to Fig. 9.23.
(Ans. u = -15cm, u = -30cm)

\
Screen
6. A concave mirror forms a real image four times as
tall as the object placed 10 cm in front of mirror. -1.5 m ,T

P
Find the position of the image and the radius of 1 O
curvature of the mirror.

(Ans. 40 cm in front of the mirror, R = - 16 cm) /


7. When an object is placed at a distance of 60 cm from
a convex spherical mirror, the magnification Fig. 9.23
produced is 1/2. Where should the object be placed
Let«=OP = -xm and /P =-(x + 1.5) m
to get a magnification of 1/3 ? (Ans. - 120cm) >y
4cm
8. An object of 1 cm^ face area is placed at a distance of Areal magnification = Icm^ = 4
1.5 m from a screen. How far from the object should
a concave mirror be placed so that it forms 4 cm^ .●. Linear magnification =- -Ji = -2
image of object on the screen ? Also, calculate the
(Negative sign for real image)
focal length of the mirror. (Ans. - 1.5 m, - 1 m) V
As m = -
HINTS ii

-(a-+1.5)
3. Refer to Fig. 9.19. Let BP= at. -2=- or a = 1.5 m
- X
Then A' P = 3+ x metre
- 1.5 m, =- 3m
So »= -a m and y = - (a + 3) m
A
As m- — = —V -9 cm a + 3 Now l-l+l- ^ .●. / = -1 m.
u 3 cm a / n V 1.5 3

or a = 1.5 m
9.7 REFRACTION OF LIGHT
H = -1.5m and y =-4.5 m
14. What is meant by refraction of light ?
1 1 1
j L = _®
4.5" I-9 or / = Refraction of light. When light travels in the same
-1.125 m
/ u V 1.5
homogeneous medium, it travels along a straight path.
5. For concave mirror, R = -20 cm => /=-10cm However, when it passes obliquely from one
V transparent medium to another, the direction of its
For real image. m = - = -2 => v = 2ii
u path changes at the interface of the two media. This is
called refraction of light.
A —1 = -1 + -1
As or J__J_ 1__^
f V u -10 ~ 2u u~ 2u The phenomenon of the change in the path of light os
it passes obliquely from one transparent medium to
M =
|x(-10) = -15 cm and i; = 2m = -30 cm.
another is called refraction of light.
9.14 PHYSICS-XII

The path along which the light travels in the first The ratio V2 is called refractive index of second
medium is called incident ray and that in the second
medium with respect to first medium. The second law
medium is called refracted ray. The angles which the
was first deduced by a Dutch scientist Willibord Snell in
incident ray and the refracted ray make with the
normal at the surface of separation are called angle of 1621, so it is also known as Snell's law of refraction.
incidence (/) and angle of refraction (r) respectively. 9.9 REFRACTIVE INDEX
Incident Normal
16. Define refractive index of a medium in terms of
(i) speed and (ii) wavelength, of light. What is relative
refractive index ?
Refractive index in terms of speed of light. The
refractive index of a medium may be defined in terms
of the speed of light as follows :

w
The refractive index of a medium for a light of given
Refracted
wavelength may be defined as the ratio of the speed of
ray
Normal light in vacuum to its speed in that medium.
Incident
ray

Glass
Normal

Flo Refractive index - Speed of light in vacuum


Speed of light in medium

ee
Air
c
or

Fr
= -

V
Air
Glass
Refractive index of a medium with respect to
Refracted vacuum is also called absolute refractive index.
for
ur
ray Refractive index in terms of wavelength. Since the
ib) (c) frequency (v) remains unchanged when light passes
from one medium to another, therefore,
Fig. 9.24 Refraction of light (a) from rarer to denser
ks

medium
(b) from denser to rarer medium „ = £ = i-V„X'’ ^
Yo

vac

V .Xmed XV Xmed
oo

(c) no refraction for normal incidence.


It is observed that The refractive index of a medium may be defined as
B

the ratio of wavelength of light in vacuum to its


1. When a ray of light passes from an optically rarer
re

wavelength in that medium.


medium to a denser medium, it bends towards
the normal (Z r < Zi\ as shown in Fig. 9.24(a). Relative refractive index. The relative refractive
ou

2. When a ray of light passes from an optically index of medium 2 with respect to medium 1 is defined
ad

denser to a rarer medium, it bends away from ratio of speed of light (u^) in medium 1 to the
Y

the normal (Zr>Zi), as shown in Fig. 9.24(b). of light (^2) in medium 2 and is denoted by ^2-
V
3. A ray of light travelling along the normal passes Thus
1
P2 = -"
1
nd

undeflected, as shown in Fig. 9.24(c). Here V


Re

2
Zi = Zr = 0°. As refractive index is the ratio of two similar
Fi

9.8 LAWS OF REFRACTION OF LIGHT physical quantities, so it has no units and dimensions.
17. State the factors on which the refractive index of a
15. State the laws of refraction of light. medium depends.
Laws of refraction of light. The phenomenon of Factors on which the refractive index of a medium
refraction of light obeys the following two laws : depends. These are as follows :
First lauK The incident ray, the refracted ray and the 1. Nature of the medium.
normal to the interface at the point of incidence all lie in the
2. Wavelength of the light used.
same plane.
3. Temperature.
Second law. The ratio of the sine of the angle of inci
4. Nature of the surrounding medium.
dence and the sine of the angle of refraction is constant for a
given pair of media. It may be noted that refractive index is a charac
teristic of the pair of the media and also depends on the
Mathematically, sin r
= Vo/^ a constant, wavelength of light, but is independent of the angle of
incidence.
RAY OPTICS AND OPTICAL INSTRUMENTS 9.15

For Your Knowledge 9.11 PRINCIPLE OF REVERSIBILITY OF LIGHT

> optical density is a quantity quite different from mass 20. State the principle of reversibility of light. Use this
density. Optical density is the ratio of the speed of principle to show that the refractive index of medium 2
light in two media while mass density is the mass per with respect to medium 1 is reciprocal of the refractive
unit volume. Interestingly, an optically denser medium index of medium 1 with respect to medium 2.
may have mass density less flran an optically rarer Principle of reversibility of light. This principle states
medium. For example, the mass derrsity of turpentine that if the final path of a ray of light after it has suffered
is less than that of water but turpentine is optically several reflections and refractions is reversed, it retraces its
more denser than water. path exactly.
N

9.10 CAUSE OF REFRACTION


A-
18. Describe the cause of refraction of light.
Cause of refraction of light. Light travels with B Rarer (1)
X
different speeds in different media. The bending of light Denser (2)
or refraction occurs due to the change in the speed of light as
it passes from one medium to another. Larger the change
in the speed of light as it passes from one medium to C
another, the more is the bending due to refraction. The Plane
Snell's law of refraction may be written as N' mirror

1 sin I
-11
F2 = 3
sin r
^2 Fig. 9.25 Principle of reversibility of light.
From the above equation, we can note the As shown in Fig. 9.25, consider a ray of light AB
following results : incident on a plane surface XY, separating rarer
(i) If Uj >Vy then V2 >^ i > sin r or i>r medium 1 (air) from denser medium 2 (water). It is
i.e., the refracted ray bends towards the normal, refracted along BC.
The medium 2 is said to be optically denser Let angle of incidence,
than medium 1. Hence a ray of light bends towards Z ABN = i
the normal as it refracts from a rarer medium into a
denser medium.
and angle of refraction,
ZCBN'=r
(ii) If <i?2, then V2 sin i <sin r or i < r From SnelTs law of refraction
i.e., the refracted ray bends away from the
sin i
normal. The medium 2 is said to optically rarer 1
1^2 ...(1)
than medium 1. Hence a ray of light bends away sin r

from the normal as it refracts from a denser medium Suppose a plane mirror is placed perpendicular to
into a rarer medium. the path of ray BC. This reverses the beam along its
19. Give the physical significance of refractive index. own path. Therefore, for the reversed ray, we have
Physical significance of refractive index. The Angle of incidence. ZCBN'=r
refractive index of a medium gives the following two Angle of refraction, Z ABN = i
informations :
Again, from Snell's law
(i) The value of refractive index gives information sin r 2
●●●(2)
about the direction of bending of refracted ray. sm i

It tells whether the ray will bend towards or


Multiplying equations (1) and (2), we get
away from the normal. sin i sin r 1 2
{ii) The refractive index of a medium is related to sm r
— = P2^ 1^1
sm z
the speed of light. It is the ratio of the speed of 1
light in vacuum to that in the given medium. or 1= V2^ Vi or
1
F2 =
For example, refractive index of glass is 3/2.
This indicates that the ratio of the speed of light
Thus the refractive index of medium 2 with respect to
in glass to that in vacuum is 2 : 3 or the speed of
medium 1 is reciprocal of the refractive index of medium 1
light in glass is two-third of its speed in vacuum.
with respect to medium 2.
9.16 PHYSICS-Xll

9.12 REFRACTION THROUGH A RECTANGULAR 22. A ray of light is incident at angle i on a rectan
GLASS SLAB AND LATERAL SHIFT gular slab of thickness t and refractive index )i. Obtain an
21. Discuss the refraction through a glass lab and expression for the lateral displacement of the emergent
ray. Can lateral displacement exceed t ?
show that emergent ray is parallel to the incident ray but
laterally displaced. Expression for lateral displacement. Fig. 9.27 shows
Refraction through a rectangular glass slab. the path of the ray undergoing refraction through the
slab PQRS. Let t be the thickness of the slab and x, the
Consider a rectangular glass slab PQRS, as shown in
Fig. 9.26. A ray AB is incident on the face PQ at an angle lateral displacement of the emergent ray. Then from
right A BEC, we have
of incidence i.^. On entering the glass slab, it bends
towards normal and travels along BC at angle of an
X = BC sin (/- r)
refraction r^. The refracted ray BC is incident on face SR
at an angle of incidence Tlie emergent ray CD bends

w
away from the normal at an angle of refraction r2-
A. 1 Q-r
i.I <

iB Air
P
,N

l^\ '‘n
I
s
Glass
Q

Flo
ee
I I's R
J2‘ N

Fr
\
s

s ic \
\
R
Air N

1 Fig. 9.27 Calculation of lateral displacement.


for
ur
I
I
S^^Lateral From right A BFC, we have
D displacement BF BF t
= cos r or BC ~
BC
ks

cos r cos r
Fig. 9.26
Yo

t
Using Snell's law for refraction at face PQ, X =
sin(/ - r) ...(1)
oo

cos r
sin i
1 _ a
...(1) t
B

P
g
sm .r
1
cos r
[sin i cos r - cos i sin r]
re

For refraction at face SR,


. cos i sin r
sin 22 1 = t sin 1 -
Pa ...(2)
ou

cos r
ad

a
sm K
2
g
From Snell's law,
Y

Multiplying (1) and (2), we get sin I sin i


P=- or sm r =
sin 1 sm i sm r
P
I
X ^=1
nd
Re

sm .r
1 sin ^ sin^ i
and cos r =
-^1-sin^ r T-
As PQ ]I SR, therefore, i2 = 'i ^ hence
Fi

sm i sin n1 cos i. sm I
lx = 1 Hence x = t sm t
sm n
sin ^ .^l/2
1 sin^ i
P 1-
or
sm ij = sin ^ or

Thus the emergent ray CD is parallel to the incident X = / sin I 1-


cos 1

ray AB, but it has been laterally displaced with respect


or
...(2)
(|i^ - sin^ if^'^
to the incident ray. This shift in the path of light on
emerging from a refracting medium with parallel faces is Clearly, x tends to a maximum value when i 90°,
so that sin / -> 1 and cos i -> 0. Thus
called lateral displacement.
X = t sin90° = f
Hence lateral shift is the perpendicular distance between max

the incident and emergent rays, when light is incident i.e., the displacement of the emergent ray cannot exceed the
obliquely on a refracting slab with parallel faces. thickness of the glass slab.
RAY OPTICS AND OPTICAL INSTRUMENTS 9,11

From equation (2), it may be noted that the lateral Thus, by knowing the refractive indices of any two
shift produced by a glass slab increases with media like glass and water with respect to air, the
refractive index of glass with respect to water or vice
(j) the increase in the thickness of the glass slab, versa can be calculated.
(ii) the increase in the value of the angle of
incidence, and 9.14 PRACTICAL APPLICATIONS OF
{Hi) the increase in the value of the refractive index REFRACTION
of the slab.
24. Why is the apparent depth of an object placed in a
9.13 REFRACTION THROUGH A denser medium less than the real depth ? For viewing
near the normal direction, show that the apparent depth is
COMBINATION OF MEDIA
real depth divided by the refractive index of the medium.
23. For a ray of light undergoing refraction through a What is normal shift ? Write an expression for it.
combination of three media, show that Real and apparent depths. It is on account of
V2X V3X Vi=l- refraction of light that the apparent depth of an object
placed in denser medium is less than the real depth.
Refraction through a combination of media.
Figure 9.29 shows a point object O placed at the
Fig. 9.28 shows the refraction of a ray of light from air bottom of a beaker filled with water. The rays OA and
(1) to water (2), glass (3) and finally to air. As all
OB starting from O are refracted along AD and BC,
boundaries are parallel planes, emergent ray is parallel
to the incident ray. Thus the angle of emergence is
respectively. These rays appear to diverge from point /.
equal to the angle of incidence. Eye
D C
A.
I
B Air(l)
r
-I-
1 r Water {2)'-. A B
I
t' t
Apparent r
C
Glass (3) Cu depth I

± _ _
●a
<0
i
Dt.
I I OS I

O
Fig. 9.28 Refraction through a combination of media.
For the ray going from medium 1 to medium 2, Fig. 9.29 Real and apparent depths.
1 sin I
1^2 = - So / is the virtual image of O. Clearly, the apparent
sm q
For the ray going from medium 2 to medium 3,
depth A1 is smaller than the real depth AO. That is why
sm
a water tank appears shallower or an object placed at
the bottom appears to be raised.
sm r^ From Snell's law, we have
For the ray going from medium 3 to medium 1,
Cl) sin I sin ZAOB AB/BO _ BI
■> sm r, =

sm r sin ZAIB AB/BI BO


sm I

As the size of the pupil is small, the ray BC will


Multiplying the above three equations, we get
enter the eye only if B is close to A. Then
V2X Vi =1 B/ - AI and BO ~ AO
1
Moreover, Va = 1 ^ AO
1^2^ ^1^1
1
V (I) CO

V3 Real depth
or
^3 = V
or Refractive index =
V2 ^^3 Apparent depth
w Real depth
or or Apparent depth = Refractive index
P- w
9.18 PHYSICS-XII

As the refractive index of any medium (other than 26. The sun near the horizon appears flattened at
vacuum) is greater than unity, so the apparent depth is sunset and sunrise. Why ?
less than the real depth. . . n ● r .
^ Apparent flattening of the sun at sunrise and
Normal shift. The height through which an object sunset. The sun near the horizon appears flattened.
appears to be raised in a denser medium is called normalThis is due to atmospheric refraction. The density and
shift. Clearly the refractive index of the atmosphere decrease with
Normal shift = Real depth - Apparent depth altitude, so the rays from the top and bottom portions
or d=AO-AI=AO-
AO of the sun on the horizon are refracted by different
M degrees. This causes the apparent flattening of the sun.
1 But the rays from the sides of the sun on a horizontal
= AO 1--
11
plane are generally refracted by the same amount, so
the sun still appears circular along its sides.
V

w
1
or d=t 1-- Examples based on
(j) Refraction of Light
Clearly, the normal shift in the position of an object (li) Lateral shift and
when seen through a denser medium depends on two
factors :

Flo
(ill) Real and Apparent Depths
Formulae Used

ee
1. Tlie real depth of the object or the thickness (f)
of the refracting medium. 1. Refractive index = Speed of light in vacuum

Fr
2. The refractive index of the denser medium. The Speed of light in medium
higher the value of p, greater is the apparent
c
or
p =-
shift 'd'. V
for
ur
25. Explain how does the refraction of light affect the 2 _ Wavelength in vacuum \
length of the day ? Wavelength in medium X'
ks

Apparent shift in the position of the sun at sunrise 3. Snell's law, V2 *


Yo

and sunset. Due to the atmospheric refraction, the sun


” sm r
^2 1^1
oo

is visible before actual sunrise and after actual sunset. or


p.j sin z = pj sin r
4. ^2=2^
B

\ I
Apparent ^
position -- - S
re

1
of sun Atmosphere M3
5. ’pj X ^3 X ^pj = 1 or ^3 =
Horizon
V2
ou
ad

6. Lateral shift of a ray through a rectangular slab,


t
Y

Actual
X =
sin (i - r)
COS r

position
of sun cos i
nd

= f sin I 1 -
Re

(p^^-sin^o'/^
Fig. 9.30 Refraction effect at sunset and sunrise.
Real depth f
Fi

7. p =
With altitude, the density and hence refractive Apparent depth Apparent depth
index of air-layers decreases. The light rays starting t

from the sun S travel from rarer to denser layers. They Apparent depth = —
bend more and more towards the normal.
1
However, an observer sees an object in the direction 8. Apparent shift =t 1
of the rays reaching his eyes. So to an observer standing
on the earth, the sun which is actually in a position S 9. Total apparent shift for compound media
N

below the horizon, appears in the position S', above the = t1


1-— +^2 +
horizon. The apparent shift in the direction of the sun
Ml Mzj
is by about 0.5°. Thus the sun appears to rise early by Units Used
about 2 minutes and for the same reason, it appears to
set late by about 2 minutes. This increases the length of All distances are in metre, angles in degrees and
the day by about 4 minutes. refractive index p has no units.
RAY OPTICS AND OPTICAL INSTRUMENTS 9.19

Example 13. A ray of light of frequency 5 x Hz is (?) When the ray travels from medium 1 to medium 2,
measured
passed through a liquid. The wavelength (flight sin I _ ]i 2
inside the liquid is found to be 450 x 10" m Calculate the sm r

refractive index of the liquid. [Himachal 98C]


Solution. Here v = 5 x 10^“* Hz, X = 450 x 10 ^m, or
sin30° 4/3 _8
c =3 X 10® ms -1 sinr "3/2 ~9
9
Refractive index of the liquid, or sin r = - X sin 30° = - X — = 0.5625
8 8 8 2
c _ c 3x10
= - = 1.33.
V 'vX"5xl0^‘‘x 450x10"^ r=34°14'.
3
Example 14. Refractive index of glass is - and refractive (ii) When the ray travels from medium 2 to medium 1,
sin I _ li j
index of water is If the speed of light in glass is sm r
1^2
2.00x 10®ms"\/inrf the speed of light in water. sin30° _3/2 _9
3 or
Solution. Given " p *„ =— 2
...(0 sm r 4/3"8

17 c 4 or sin r = - X sin 30° = - x — = 0.4445


and 11 w
...(?/) 9 9 2
V 3
CO
r=26°24'.
On dividing (?) by (/?), we get
Example M. A rectangular glass slab rests in the bottom
V
0) 3,,3 ^9 of a trough of water. A ray of light incident on water surface
V 2" 4 8
at an angle of50°passes through water into glass. Calculate
v..
9
= -X 17 -x2.00xl0®ms"’ the angle of refraction in glass. Given that p for water is 413
0)
8 8 and that for glass is 3/2.
4 3
= 2.25x 10®ms"V 17 (7
Solution. Here 11 11
CO
"3' 2
Example 15. Light of wavelength 4500 A in vacuum enters (7

a
glass block of refractive index 1.5. What is the frequency CO _3/2 _9
11
p.. ~4/3 8
a

and wavelength of light in the glass block ? [ISCE 05] CO

Solution. In vacuum, X = 4500 A = 45 x 10 ® m. hJ ■A

c =3X 10® ms ^ 50^


-I
c 3x10 ms 1
v = — = _!_x10^^Hz
X 45xl0"°m 15 r
r
When light goes from vacuum to the glass block, its C
frequency remains unchanged,
v'=v= —xlO^^Hz = 6.67x V‘
15 a
Glass slab
Wavelength of light in the glass block,
Fig. 9.31
p 1.5
Angle of incidence on water surface, i = 50°
Example 16. A ray of light passes through a plane sin 50° _ 4
boundary separating two media whose refractive indices are sin r 3
p^ =3/2i?nrfp2 =4/3. (i) If the ray travels from medium 1
to medium 2 at an angle of incidence of 30°, what is the angle sm r = — sin 50°
4
of refraction ? (ii) If the ray travels from medium 2 to
medium 1 at the same angle of incidence, what is the angle of -X 0.766 =0.5745
refraction ? 4
3 4
?=30°
Solution. Here 1^ i “ 2 ' .●. Angle of refraction, r = 35.06°
9.20 PHYSICS-XIl

For refraction at water-glass interface, we have Solution. Refractive index.


sin 35.06° 9
Real deptli 1.3 = Real depth
sin y 8
Apparent depth 7.7
8
or sin y = -X 0.5745 =0.5107 . ●● r'=30.7°. Hence, Real depth = 1.3 x 7.7 cm = 10.01 cm.
9
Example 20. The velocity of light in glass is 2 x 10®?ns“^
Example 1 o. A parallel sided glass slab of thickness 6 cm is
and that in air is 3xl0®ms“ . By how much would an ink
made of a material of refractive index J3. When light is dot appear to be raised, when covered by a glass plate 6.0 cm

incident on one of the parallel faces at an angle of 60°, it thick ?


emerges from the other parallel face. Find the lateral shift of Solution. Here v = 2 x 10^ ms -I , c=3x 10 ms -1
the emergent beam.
Solution. Figure 9.32 shows the path ABCD of the Refractive index of glass,
ray refracted by the glass slab. CE is the lateral shift of

w
_ c__3xl0
the emergent ray relative to the incident ray. ^ V 2x10
= 1.5

Real depth =6.0 cm

Flo Apparent depth =


Real depth _ 6.0
1.5
= 4.0 cm

ee
Distance tlirough which the ink dot appears to be

Fr
raised

- 6.0 - 4.0 = 2.0 cm.

Example 21. A microscope is focussed on a coin lying at


for
ur
the bottoin of a beaker. The microscope is now raised up by
1 cm. To what depth should water be poured into the beaker
so that coin is again in focus. Refractive index of water is 4/3.
ks

Solution. Suppose water is poured into the beaker


Yo

upto height h to bring the coin in focus again. Then,


oo

In right ABFC,
f 1
t Normal shift =h 1—
B

cosr =
cosr
re

In right ACEB f 3 h
1 cm =h 1 — h=4cm.
4) 4
ou

sin(i-r) =
ad

BC t/cosr
Example 22. The bottom of a container is a 4.0 cm thick
Y

Lateral shift produced. glass (p =1.5) slab. The container contains two immiscible
t liquids A and B of depths 6.0 cm and 8.0 cm respectively.
X =
sm(i-r) What is the apparent position of a scratch on the outer
nd
Re

cosr

smi sin60°
surface of the bottom of the glass slab when viewed through
By Snell's law, p= — => V3 = the container ? Refractive indices of A and B are 1.4 and 1.3
Fi

smr smr

1
respectively.
r=30°
smr = —
2 Solution. The total apparent shift in the position of
the image due to all the three media is given by
Lateral shift.
1 ^ 1 1 ( 1
6 cm
-sin(60°-30°)
d=t, 1 —^
1— +f3 1-2
I ^i)
X =
cos30 ^2 ^3
1
= 6 tan 30°= 6 X = 2V3 cm = 3.564 cm. Given = 4.0 cm, f2=6.0cm, f3=8.0cm
V3
^3=1.5, ^3=1.3
Example 19. The apparent depth of an object at the bottom 1 ^ f 1 \ f 1 ^
.-. d = 4.0 1 - — +6.0 1—- +8.0 1-
of tank filled with a liquid of refractive index 1.3 is 7.7 cm. 1.5 1.4 1.3
What is the actual depth of the liquid in the tank ?
[CBSE D 91] = 1.33 + 1.71 + 1.85 =4.89 cm.
RAY OPTICS AND OPTICAL INSTRUMENTS

Example 23. A transparent cube of 15 cm edge contains a Height of water level from the bottom
Volume 928 n
small air bubble. Its apparent depth when viewed through = 25.78 cm
one face is 6 cm and when vieioed through the opposite face is nr^ TtX (6)^
4 cm. Find the refractive index of the material of the cube. Depth of water above the glass piece
Solution. When viewed from face 1, = 25.78-8.0=17.78 cm
rd, X
li = —^ = ...(0 Total apparent shift of the bottom
ad,] 6
= t,1 1- - 1-
11
When viewed from face 2,
Pi
rd.^ _ 15-x ...(/O
|J =
1 1 ^
adj 4
= 17.78 1- + 8.0 1-
1.33 1.50

= 4.44 + 2.66 = 7.1 cm

Thus, the image is seen at 7.1 cm above the bottom,

5^ roblems For Practice


1. A film of oil of refractive index 1.20, lies on water of
/1 I
6 cm 2 4 cm refractive index 1.33'. A light ray is incident at 30° in
O ad^ Face 2
the oil on the oil-water boundary. Calculate the
Face 1
angle of refraction in water. (Ans. 27°)
2. A printed page is kept pressed by a glass cube
rd 1 = 1.5) of edge 6.0 cm. By what amount will the
printed letters appear to be shifted when viewed
h 1^(15-X) (Ans. 2.0 cm)
from the top ?
Fig. 9.33 3. A travelling microscope is focussed on a mark made
on a paper. When a slab of 1.47 cm thickness is placed
From (/) and (if), we get
on the mark, the microscope has to be raised
.r 15-X
X = 9 cm through 0.49 cm to focus the mark again. Calculate
6 4 the refractive index of glass. (Ans. 1.5)

From (i), u = - = 1.5. 4. The velocity of light in a transparent medium is


6 1.8xl0®ms”^ while that in vacuum is 3xl0®ms"^
Find by how much the bottom of the vessel con
Example 24. A cylindrical taining the liquid appears to be raised if the depth
vessel of diameter 12 cm of the liquid is 0.25 m. (Ans. 0.1 m)
contains 800 n at? of water. 5. Calculate the index of refraction of a liquid from the
A cylindrical glass piece of following into glass : (a) Reading for the bottom of
diameter 8.0 cm and height an empty beaker : 11.324 cm (b) Reading for the
8.0 cm is placed in the vessel. 8cm bottom of the beaker, when partially filled with the
If the bottom of the vessel liquid : 11.802 cm (c) Reading for the upper level of
under the glass piece is seen by the liquid in the beaker : 12.895 cm (Ans. 1.437)
the paraxial rays (Fig. 9.34),
8 cm ● 6. While determining the refractive index of a liquid
locate its image. The index of — 12 cm experimentally, the microscope was focussed at the
refraction of glass is 1.50 and bottom of a beaker, when its reading was 3.965 cm.
that of water is 1.33. Fig. 9.34
On pouring liquid upto a height 2.537 cm inside the
Solution. Volume of water =800 n cm^ beaker, the reading of the refocussed microscope
was 3.348 cm. Find the refractive index of the
Volume of cylindrical glass piece liquid. (Ans. 1.321)
8 \2 7. A vessel contains water upto a height of 20 cm and
= TZ X 8 =128 71 cm^
2 above it an oil upto another 20 cm. The refractive
indices of water and oil are 1.33 and 1.30
Total volume of water and glass piece
respectively. Find the apparent depth of vessel
= 800 7i + 128 7T=928 7rcm^ when viewed from above. (Ans. 30.4 cm)
9.22 PHYSICS-Xll

HINTS
9.15 TOTAL INTERNAL REFLECTION
1. Refractive index of water relative to oil is
a 27. Explain the phenomenon of total internal
M w _ 1-33
●●● V u> it
1.20
= 1.11 reflection. Under what conditions does it take place ?
0
Derive the relation connecting the refractive index and
From Snell's law. critical angle for a given pair of media.
sm I sin 30° Total internal reflection. If light passes from an
1.11 =
w
sin r sm r
optically denser medium to a rarer medium, then at
1
the interface, the light is partly reflected back into the
or sm r =
2x1.11
= 0.45 denser medium and partly refracted to the rarer
medium. This reflection is called internal reflection.
or r^lT Under certain conditions, the whole of the incident
2. Normal shift. light can be made to be reflected back into the denser

w
i ll f 1 medium. This gives rise to an interesting phenomenon
d=l 1-- =6.0 1- — = 2.0 cm. called total internal reflection.
1.5

3. As fr = t
/'
1
l'^
●. 0.49=1.47 1-
1

Flo Rarer
medium
(1)
Air-waler
interface

ee
Denser
or or H =1.5.

Fr
3 3 medium
Totally
4. Here y= 1.8 X 10®ms"5 c = 3xl0®ms“^ refracted ray
8
_ 3 X10- 5
Refractive index, ^ = -
for
ur
v~ 1.8x10^ ”3 I
C

Real depth = 0.25 m Total internal reflection.


Real depth _ 0.25
ks

Apparent depth = = 0.15 m


5/3 As shown in Fig. 9.35, when a ray of light (ray 1)
Yo
oo

Distance through which bottom appears to be raised travels at a small angle of incidence from a denser
= 0.25 - 0.15 = 0.1 m. medium to a rarer medium, say from water to air, the
B

5. Real depth
refracted ray bends away from the normal so that the
angle of refraction is greater than the angle of
re

= Reading from the upper level of the liquid incidence. As the angle of incidence increases, the
- Reading from the bottom of
corresponding angle of refraction also increases. Then
ou

the empty beaker


ad

= 12.895-11.324= 1.571cm
for a certain angle of incidence (ray 2), the angle of
refraction becomes 90°, i.e., the refracted ray goes along
Y

Apparent depth the surface of separation.


= Reading from the upper level of the liquid
nd

The angle of incidence in the denser medium for


Re

- Reading from the bottom of the beaker


when partially filled with liquid which the angle of refraction in the rarer medium is
90° is called critical angle of the denser medium and is
Fi

= 12.895-11.802= 1.093cm
denoted bp i^.
1.571
P = = 1.437.
1.093 If the angle of incidence is increased beyond i^
6. Real depth = 2.537 cm (ray 3), no light is refracted into the rarer medium
Apparent shift in the position of bottom of the
(since the angle of refraction cannot be greater than
beaker = 3.965 - 3.348 = 0.617 cm
90°), but whole of it is reflected back into the denser
medium in accordance with the laws of reflection. This
Apparent depth = 2.537- 0.617= 1.920 cm
phenomenon is known as total internal reflection.
Real depth 2.537
P = = 1.321 The phenomenon In which a rap of light travelling at
Apparent depth 1.920
an angle of incidence greater than the critical angle
1 1 from denser to a rarer medium is totallp reflected back
7. Total apparent shift = tj 1- —
+ t^ 1- —
into the denser medium is called total internal
1^1 1^2
reflection.
RAY OPTICS AND OPTICAL INSTRUMENTS 9,23

Necessary conditions for total internal reflection : which a traveller sees a shimmering pond of water some
1. Light must travel from an optically denser to an distance ahead of him and in which the surrounding objects
optically rarer medium. like trees, etc, appear inverted.
2. The angle of incidence in the denser medium must be Eye
greater than the critical angle for the two media.
Z Cold air
Relation between critical angle and refractive (denser)
index. From Snell's law, A C

Hot air
sin i 2 1 T
ti] = 2. (rarer)
sm r
V2
When i = i^, r=90° Therefore,
sm 1 1 1
or
^^2=-
sin 90° sm
s.‘
./

If the rarer medium is air, then Hj =1 and =1^


Fig. 9.36 Formation of mirage.
(say) and we get
1
h =- On a hot summer day, the surface of the earth
sm
becomes very hot. The layers of air near the earth are
Thus the refractive index of any medium is equal to more heated than the higher ones. Hence the density
and refractive index of air layers increase as we move
the reciprocal of the sine of its critical angle.
high up. As the rays of light from a distant object like a
Table 9.1 Critical angles of some transparent media tree travel towards the earth through layers of
decreasing refractive index, they bend more and more
Substance
Refractive
Critical angle
away from the normal. A stage is reached when the
index angle of incidence becomes greater than the critical
Water 1.33 48.75° angle, the rays are totally reflected. These rays then
Crown glass 1.52 4114° move up through layers of increasing refractive index,
37.31°
and therefore undergo refraction in a direction opposite
Dense flint glass 1.65
to that in the first case. These rays reach the observer's
Diamond 2.42 24.41°
eyes and he sees an inverted image of the object, as if
formed in a pond of water.
9.16 APPLICATIONS OF TOTAL INTERNAL
3, Totally reflecting prisms. Refer to Section 9.17.
REFLECTION
4. Optical fibres. Refer to Section 9.18.
28. Discuss the important applications of the
phenomenon of total internal reflection. 9.17 TOTALLY REFLECTING PRISMS
Applications of total internal reflection. The 29. What is a totally reflecting prism ? How can it be
phenomenon of total internal reflection can be used to used to (i) deviate a ray through 90° (ii) deviate a ray
explain some effects observed in daily life and also it through 180° and (Hi) invert an image without the
finds use in some optical devices as explained below: deviation of the rays.
1. Sparkling of diamond. The brilliancy of Totally reflecting prism. A right-angled isosceles
diamonds is due to total internal reflection. As the prism, i.e., a 45°-90°-45° prism is called a totally reflecting
refractive index of diamond is very large, its critical prism. Whenever a ray falls normally ori any face of
angle is very small, about 24.4°. The faces of diamond such a prism, it is incident on the inside face at 45°, that
are so cut that the light entering the crystal suffers total is at an angle greater than the critical angle of glass
internal reflections repeatedly, and hence gets (about 42°); hence this ray is always totally internally
collected inside but it comes out through only a few reflected,
faces. Hence the diamond sparkles when seen in the These prisms may be used in three ways :
direction of emerging light. (i) To deviate a ray through 90°. As shown in
2. Mirage. It is an optical illusion observed in deserts or p^g 9.37(A), as the light is incident normally on one of
over hot extended surfaces like a coal-tarred road, due to faegg containing right angle, it enters the prism
9.24 PHYSICS-XII

without deviation. It is incident on the hypotenuse face 2. The reflecting properties of prisms are perma
at an angle of 45®, greater than the critical angle. The nent, while these are affected by tarnishing in
light is totally internally reflected. Having been case of plane mirrors.
deviated through 90®, the light passes through third 3. No multiple images are formed in prisms, while
face without any further deviation. Such prisms are a plane mirror forms a number of faint images
used in periscopes.
in addition to a prominent image.
R
Q
9.18 OPTICAL FIBRES
A

31. What are optical fibres ? On which principle do


[
A

45°V they work ? How does light propagate through an optical


B
B
B' fibre ? What is a light pipe ?
7 Optical fibres. These days we find in the market

w
Q
A'
some decorative lamps provided with fine plastic
A' B' R fibres. At their one ends, the fibres are fixed over an

(0)
electric lamp while their free ends form a fountain like

Fig. 9.37 (a) To deviate a ray through 90®


Flo structure. When the lamp is switched on, the light
travels from the bottom of each fibre and appears at the

ee
(b) To invert an image with deviation of rays through 180®. tip of free end as a bright dot of light. The plastic fibres
in these lamps are optical fibres. The working of optical

Fr
(it) To invert an image with deviation of rays fibres is based on the phenomenon of total internal reflection.
through 180° As shown in Fig. 9.37(h), the light is
incident normally on the hypotenuse face, it first An optical fibre is a hair-thin long strand of quality glass
or quartz surrounded by a glass coating of slightly lower
for
suffers total internal reflection from one shorter face
ur
and then from the other shorter face. The final beam reflective index. It is used as a guided medium for trans
mitting an optical signal from one place to another.
emerges through the hypotenuse face, parallel to the
Buffer coating
ks

incident beam. The deviation is 180°. Such a prism is Refractive


called a porroprism.
Yo

index, n.
oo

(Hi) To invert an image without deviation of rays.


(Erecting prism). As shown in Fig. 9.38, the light enters
B

at one shorter face at an angle. After refraction, it is


re

totally reflected from the hypotenuse face and then Refractive

refracted out of the otlier shorter face to become parallel to index, p,


the incident beam. The rays do not suffer any deviation, Buffer coating
ou
ad

only their order is reversed. The incident ray, which is on Fig. 9.39 Optical fibre structure.
the top, emerges from the bottom of the prism. Sudi
Y

prisms are called erecting prisms and are used in Construction. An optical fibre consists of three main
binoculars and in projection lanterns. parts :
nd
Re

(0 Core. The central cylindrical core is made of


high quality glass/silica/plastic of refractive
Fi

index p ^ and has a diameter about 10 to 100 pm.


(ii) Cladding. The core is surrounded by a glass /
plastic jacket of refractive index ^2 Iri a
Fig. 9.38 To invert an image without deviation of rays. typical optical fibre, the refractive indices of core
and cladding may be 1.52 and 1.48 respectively.
30. State the advantages of totally reflecting prisms (Hi) Buffer coaling. For providing safety and
over plane mirrors. strength, the core cladding of optical fibres is
Advantages of totally reflecting prisms over plane enclosed in a plastic jacket.
mirrors. The totally reflecting prisms have many Propagation of light through an optical fibre. As
advantages over plane mirrors as reflectors : shown in Fig. 9.40(fl), when light is incident on one end
1. In prisms, the light is totally reflected, while there of the fibre at a small angle, it goes inside and suffers
is always some loss of intensity in case of plane repeated total internal reflections because the angle of
mirrors.
incidence is greater than the critical angle of the fibre
RAY OPTICS AND OPTICAL INSTRUMENTS 9.25

material with respect to its outer coating. As there is no various parts of stomach into the central portion of the
loss of intensity in total internal reflection, the light pipe to produce a final image with excellent
outcoming beam is of as much intensity as the incident details. The technique is called endoscopy,
beam. Even if the fibre is bent, light easily travels 2. They are used in transmitting and receiving electrical
through along the fibre. signals in telecommunication. The electrical signals are
first converted to light by suitable transducers. Each
- Cladding fibre can transmit about 2000 telephone conversations
low (i = 1.48 without much loss of intensity.
3. They are used for transmitting optical signals
and two dimensional pictures.
4. In the form of photometric sensors, they are used
. Glass core
for measuring the blood flow in the heart.
high ji = 1.52
5. In the form of refractometers, they are used to
measure refractive indices of liquids.
Examples based on
Total Internal Reflection
Formulae Used

1. Critical angle, = Angle of incidence in denser


Fig. 9.40 (a) Propagation of light through an optical fibre. medium for wWch angle of refraction is 90° in
rarer medium.

2. Refractive index of denser medium, p = r


sm
Bundle of

glass fibres 3. Total internal reflection occurs when i > i^.


Units Used

Angle is in degrees and p has no units.

Example 25. Find the value of critical angle for a material


of refractive index [CBSE F 941

Solution. Here p =J3


sm = i=A_ = :i^ = 0.5773
p S 3
Fig. 9.40 (fa) A light pipe. .'. Critical angle, f - 35.3°.
Example 26. Calculate the speed of light in a medium,
A bundle of optical fibres is called a light pipe. A single
fibre cannot be used to see the complete image of an whose critical angle is 30°. [CBSE OD 10]

object. But, if the image is broken into a large number Solution. Here i^ =30°
of fine dots and each portion of the image is seen 1
As
through a separate fibre, the complete image can be P- = -
v sm
seen. A light pipe can be used to transmit such an
image accurately. zj = c sin =3 x 10® x sin 30°
32. Mention some important applications of optical = 3 X 10® X 0.5
fibres. = 1.5 X 10® ms"\
Applications of optical fibres. Some of the
important applications are as follows : Example 27. Determine the direction in which a fish under
water sees the setting sun. Refractive index of water is 1.33.
1. As a light pipe, optical fibres are used in medical and
optical examination. A light pipe is inserted into the Solution. Figure 9.41 shows the setting sun in the
stomach through the mouth. Light transmitted through direction of water surface. A ray of light starting from
the outer layers of the light pipe is scattered by the the sun enters the eye of the fish.
PHYSICS-XII

Apparent
position of Solution. The path of the refracted ray is shown in
sun as seen Fig. 9.43.
by fish ' Normal
I I
40°!

Air

Glass
15“

I
I
I

Fig. 9.43 I

Fig. 9.41
Clearly, r + 15° = 45® or r = 30°
The apparent position of the sun makes an angle Refractive index of glass,

w
with the vertical.
P-=-
sin i sin 45° _ 1 - = >/2
From Snell's law sin r sin30°
sin 90° 1
1=-L
sm
— = 1.33 or sin =
1.33
= 0.7518

Flo sin =
p -Ji

ee
i^ = sm'\0.7518) = 48.7° Critical angle, = 45°.

Example 20. A ray of light passes from glass {[l ^ =312) to

Fr
Angle between the apparent position of the sun
and the horizontal =90 - 48.7 = 41.3°. water ^ _ 4 ^3^ What is the critical angle of incidence ?
Solution. From Snell's law of refraction,
Example 28. In the following ray diagram, calculate the
for
sin f _ p
ur
speed of light in the liquid of unknown refractive index.
cu

sin r p
[CBSE OD 17C]
sin i^ _ 1.33
ks

Air sin 90° 1.5


or
sin i^ = 0.8867
Yo
oo

*— 30 cm —●
i^ = sin“\0.8867) = 62°28'.
Example 31. Figure 9.44(a) shows a ray of light falling
B

'r
E
o normally on the face AB of an equilateral glass prism having
re

refractive index ^ placed in water of refractive index |. Will


o

‘c
Fig. 9.42 Liquid
this ray suffer total internal reflection on
ou
ad

striking theface AC ? Justify your answer.


Solution. The critical angle i^ is shown in Fig. 9.42. [CBSE 18]
Y

Clearly, Solution. When the ray strikes on


3 3
face AC, angle of incidence, r=60°.
nd

sin =
Re

5 Fig. 9.44 (a)


. .
sini=sm60 -— . .no V3 1.732 = 0.866
Fi

1 5 2 2
^i = -
smi^ 3 Refractive index of glass w.r.t. surrounding water is
Also,
Speed of light in air w
_^_3/2 9
P,=
Speed of light in the liquid ^ P
l^w 4/3 8
8
5 3x10 For total internal reflection,
3 V the required critical angle of
incidence is given by
or V = -x3xl0® =1.8xl0®ms"\
5
sin ic = -=-=0.89
Fig. 9.44 (b)
p 9
Example 29. Calculate the critical angle for gl ass-air

surface, if a ray of light which is incident in air on tlie Clearly, i < i^ [●.● sin/<sini^]
glass surface is deviated through 15°, when angle of Hence, the ray would not suffer total internal
incidence is 45°.
[CBSE OD 03] reflection on striking the face AC.
RAY OPTICS AND OPTICAL INSTRUMENTS

Example 32. A ray of light incident on the horizontal


surface of a glass slab at 70° just grazes the adjacent vertical
surface after refraction. Calculate the critical angle and
refractive index of the glass. r

Solution. As shown in Fig. 9.45, the refracted ray


will graze the vertical surface BC only when the ray QR ^ = 1.25

is incident at critical angle i^.


Clearly,

Fig. 9.46

Solution. The critical angle i^ is given by



sm
■ =
^ 1 ^4
1.25 5

As V ^ therefore
sin r = sin (90° - i^) = cos i^ = i-fi T =-
5J 5
From Snell's law,
sin I
= 1.25
sm r

3
Using Snell's law for refraction at face AB, we get or sin i = 1.25 x sin r =1.25 x — =0.75
5
sin 70°
or i = 48.6°
sm r

sin 70° If the angle of incidence at vertical surface is greater


or sm r = than then / will be less than 48.6°. Hence the
P
maximum value of i, for which total internal reflection
sin 70° occurs at the vertical surface, is 48.6°.
or
sin (90° - g =
P
Example 34. A point source of light S is placed at the
or cos =
sin 70° bottom of a vessel containing a liquid of refractive index 5/3.
P A person is viewing the source from above the surface. There
For refraction at face BC, we have
is an opaque disc of radius 1 cm floating on the surface. The
centre O of the disc lies vertically above the source S. The
sm =
1
liquid from the vessel is gradually drained out through a tap.
P What is the maximum height of the liquid for which the
source cannot be seen at all ? [irri
. _= sin _ 1/n
tan
cos sin70°/|.i
Solution. As shown in Fig. 9.47, suppose the height
OS = h is such that Z OSA = i^. Then any other ray from
1 1
= 1.0642
S will be totally internally reflected because then the
sin 70° 0.9397 angle of incidence would be greater than i^.
OA r

or
i; = 46°47' In AOSA sin i^ = AS
1 1
Hence P=.-
sm sin 46°47' A B

1
= 1.372.
\h
0.7288 .1, k

Example 33. For a situation shown in Fig. 9.46, find the


maximum angle ifor which the light suffers total internal I,

reflection at the vertical surface. [AIIMS 20151 Fig. 9.47 S


9.28 PHYSICS-XII

1
Also sm I =-
4. A ray PQ incident normally on the refracting face
P
BA is refracted in the prism BACmade of material of
1 r 1 refractive index 1.5 (Fig. 9.49).
or
r + h^ Complete the path of ray
through the prism. From
+ h^ or which face will the ray
emerge ? Justify your answer.
h -1 =1 -1 [CBSE OD 161
3
5. Find the maximum angle of
4
=— =1.33 cm. refraction when a ray of light
3
is refracted from glass
Example 35. The refractive index of water is 4/3. Obtain (p = 1.5) to air. (Ans. 90°) Fig. 9.49

w
the value of the semivertical angle of the cone within which 6. A luminous object S is located at the bottom of a big
the entire outside view xvould be confined for a fish under pool of liquid of refractive index p and depth h. The
water. Draw an appropriate ray diagram. ICBSE SP 03]
object S emits rays upwards in all directions, so that
Solution. Clearly, the fish can see the outside view
of the cone with semivertical angle,
0 .
Flo a circle of light is formed at the surface of the liquid
by the rays whicli are refracted into the air. What
happens to the rays beyond the circle ? Determine

ee
the radius and the area of the circle.

Fr
(Ans. Rays beyond the circle are totally reflected
^ -

into the liquid, Radius = , ^ _ , Area = —)


6 i
for
ur
t
{
c 7. A liquid of refractive index 1.5 is poured into a

cylindrical jar of radius 20 cm upto a height of


20 cm. A small bulb is lighted at the centre of the
ks

Fish

bottom of the jar. Find the area of the liquid surface


Yo
oo

through which the light of the bulb passes into air.


Fig. 9.48
[ISCE 98]
B

1 4 1
But P=- or (Ans. 1004.8 cm ^)
3 sin Ip
re

sin Ip 8. A point source of monochromatic light'S' is kept at


or smz. = -=0.75
the centre of the bottom of a cylinder of radius
^ 4 15.0 cm. The cylinder contains water (refractive
ou
ad

index 4/3) to a height of 7.0 cm. Draw the ray diagram


0/2 = ip=sin-l (0.75) = 48.6° and calculate the area of water surface through
Y

■> roblems For Practice


which the light emerges in air. [CBSE D 15C]
(Ans. 197.82 cm^)
nd
Re

1. Determine the value of the angle of incidence for a HINTS


ray of light, travelling from a medium of refractive
Fi

index p^ = -s/^ into the medium of refractive index 1. From Snell's law, p^sinz =p2Sinr
Pj = 1 so that it just grazes along the surface of Here P2=l/ r = 90° (for just grazing)
separation. [CBSE F17] .'. V2sin i = 1 xsin 90°
(Ans. 45°) 1
or sin z = z=45°.
2. The velocity of light in a liquid is 1.5xl0®ms"^ and V2 ^
in air, it is 3 x 10®ms~^. If a ray of light passes
from
this liquid into air, calculate the value of critical 2. Here r; = 1.5 xl0®ms“^
angle. [Punjab 991 c=3x1()3 ms -1
(Ans. 30°) Refractive index of the liquid,
3. An optical fibre (p = 1.72) is surrounded by a glass c 3x10
8
1 1
coating (p = 1.50). Find the critical angle for total p =-
V 1.5 X 10
8 = 2 or sin Zp = —
P 2
internal reflection at the fibre-glass interface.
(Ans. 60.7°) .'. Critical angle, = 30°.
RAY OPTICS AND OPTICAL INSTRUMENTS 9.29

3. sin =
1.50
= 0.8721
8. Refer to Fig. 9.51. The light rays emerge through a
1.72 circle of radius r.

= 60.7®. Radius, r = htani^


1 . . 1 3
4. sm L = — = - = 0.6667 sm«, = — = -
1.5 3 ^ H 4
\2
t; = 41.8® O
A
-'c = f-UJ =4
. 3 4 3
^ 4 4? ■>17
P'
Area of the patch through which light emerges
Q. Normal = nr^ = Tih^ tan^
30°/

= n x(7f X-7 = 637icm^ = 197.82 cm^.


60°
B C
9.19 SPHERICAL LENSES
Fig. 9.50 33. What is a leus ? Mention the different types of
For face AC, i = 30° i.e., i < i^. spherical lenses.
The ray will not suffer total internal reflection but it Spherical lenses. Most of us are familiar with
will emerge from face AC bending a^vay from the lenses. As magnifying glasses, lenses have been in use
normal. for centuries. Lenses used in spectacles help us to read
5. When the ray of light is incident at the critical angle with comfort. Various optical instruments like camera,
the angle of refraction is maximum and is equal projector, microscope, telescopes, etc., cannot function
to 90°. without lenses.

6. Refer to Fig. 9.51. The light rays emerge tlirough a A lens is a piece of a refracting medium bounded by
circle of radius r. two surfaces, at least one of which is a curved surface.

A o B The commonly used lenses are the spherical lenses.


These lenses have either both surfaces spherical or one
h spherical and the other a plane one. Lenses can be
‘c t
C (
divided into two categories :
'r!;
(/) Coni’ex or converging lenses, and
(ii) Concave or diverging lenses.
S
(/) Convex or converging lens. It is thicker at the
Fig. 9.51 centre than at the edges. It converges a parallel beam of
sin light on refraction through it. It has a real focus.
Radius, r = h tan = h.
cos Types of convex lenses :
1 (fl) Double convex or biconvex lens. In tliis lens, both
surfaces are convex,
h

1 (fj) Planoconvex lens. In this lens, one side is convex


1-
and the other is plane,
(c) Concavoconvex. In this lens, one side is convex
and the other is concave.
Area of patch = Kt^

7. Area of the liquid surface through which light


A
passes into air

>2
3.14 x(20) 3.14x400
cm 2=1004.8 cm2. Biconvex Planoconvex Concavoconvex
■ (1.5)2-1 1.25
Fig. 9.52 Different types of convex lenses.
9.30 PHYSICS-XIt

(ii) Concave or diverging lens. It is thinner at the (iv) Optical centre. If a ray of light is incident on a lens
centre than at the edges. It diverges a parallel beam of such that after refraction through the lens the emergent ray
light on refraction through it. It has a virtual focus. is parallel to the incident ray, then the point at which the
Types of concave lenses : refracted ray intersects the principal axis is called the optical
(fl) Double concave or biconcave lens. In this lens, both ceiitre of the lens. In Fig. 9.55{i7), O is the optical centre of
sides are concave. the lens. It divides the thickness of the lens in the ratio
of the radii of curvature of its two surfaces. Thus :
(b) Planoconcave lens. In this lens, one side is plane
and the other is concave,
(c) Convexoconcave lens. In this lens, one side is 0P2 P2C2
convex and the other is concave. If the radii of curvature of the two surfaces are

V7 equal, then the optical centre coincides with the


geometric centre of the lens.

w
For the ray passing through the optical centre, the
incident and emergent rays are parallel. However, the
emergent ray suffers some lateral displacement
lA
Biconcave Planoconcave Convexoconcave
Florelative to the incident ray. This lateral displacement
decreases with the decrease in thickness of the lens.

ee
Hence a ray passing through the optical centre of a thin
Fig. 9.53 Different types of concave lenses. lens does not suffer any lateral deviation, as shown in

Fr
Figs. 9.55(b) and (c).
9.20 DEFINITIONS IN CONNECTION WITH
SPHERICAL LENSES
for
ur
A
34. Define the various terms in connection with
-I
I - R

Pi/^O Pi
spherical lenses. C2
ks

Definitions in connection with spherical lenses :


/
Yo

(0 Centre of curvature (C). The centre of curvature of \< 'b


oo

the surface of a lens is the centre of the sphere of which it


forms a part. Because a lens has tzvo surfaces, so it has tzvo
B

(a)
centres of curvature,
re

(ii) Radius of curvature (K). The radius of curvature of


the surface of a lens is the radius of the sphere of which the
ou
ad

surface forms a part.


O'
(Hi) Principal axis (€^€2). It is the line passing
Y

through the two centres of curvature of the lens.


(c)
nd

Optical ' i Radius of


Re

Centre of
centre\ A curvature
curvature Fig. 9.55 Optical centre,
Fi

\ \
(y) Principal foci and focal length :
C2 0

^2 Principal First principal focus. It is a fixed point on the principal


axis
axis such that rays starting from this point (in convex lens)

Radius of
Centre of Optical curvature
centre ’
curvature
«2, o

F' lO
T
f
C, o
C2
R1
Principal *■
axis

—/--H /’

Fig. 9.54 Characteristics of convex and concave lenses. Fig. 9.56 First principal focus and first focal length.
RAY OPTICS AND OPTICAL INSTRUMENTS 9.31

or appearing to go towards this point (in concave lens), 3. The distances measured in the direction opposite
after refraction through the lens, become parallel to the to the direction of the incident light are taken
principal axis. It is represented by Fj or F'. The plane negative.
passing through this point and perpendicular to the 4. Heights measured upwards and perpendicular
principal axis is called the first focal plane. The distance to the principal axis are taken positive.
between first principal focus and the optical centre is 5. Heights measured downwards and perpen
called the first focal length. It is denoted by or /'. dicular to the principal axis are taken negative.
Second principal focus. It is a fixed point on the
principal axis such that the light rays incident parallel to the Height
Incident light
A
principal axis, after refraction through the lens, cither
converge to this point (in convex lens) or appear to diverge
O
from this point (in concave lens). The plane passing Negative distance ^
Positive distance

through this point and perpendicular to principal axis against incident light
along incident light
is called the second focal plane. The distance between - ve Height
the second principal focus and the optical centre is
called the second focal length. Itis denoted by /2or /. Fig. 9.58 New Cartesian sign convention for a spherical lens.

Consequences of the sign convention :


1. The focal length of a converging lens is positive and
that of a diverging lens is negative.
O F F )
>
2. Object distance is always negative.
~o
3. The distance of real image is positive and that of
h virtual image is negative.
f f 4. The object height h^ is always positive. Height
^2 of virtual erect image is positive and that of
real inverted image is negative.
Fig. 9.57 Second principal focus and first focal length.
5. The linear magnification m = h2/h.^ is positive
Generally, the focal length of a lens refers to its for a virtual image and negative for a real
second foc^ length. It is obvious from the above image.
figures that the foci of a convex lens are real and those Before deriving formulae for spherical lenses, we
of a concave lens are virtual. Thus the focal length of a £■ consider refraction by a single spherical surface.
convex lens is taken positive and the focal length of a concave
lens is taken negative. 9.22 REFRACTION AT A CONVEX
If the medium on both sides of a lens is same, then SPHERICAL SURFACE
the numerical values of the first and second focal
36. By stating the sign-convention and assumptions
lengths are equal. Thus used, derive the relation between the distance of object,
/=/’● distance of image and radius of curvature of a convex
(vi) Aperture. It is the diameter of the circular spherical surface, when refraction takes place
boundary of the lens. (i) from optically rarer to optically denser medium and
the image formed is real,
9.21 NEW CARTESIAN SIGN CONVENTION
(ii) from optically rarer to optically denser medium and
FOR SPHERICAL LENSES
the image formed is virtual,
35. State the new Cartesian sign convention for (Hi) from optically denser to optically rarer medium and
spherical lenses. What are the important consequences of the image formed is real, and
this sign convention ? (iv) from optical denser to optically rarer medium and the
New Cartesian sign convention for spherical lenses. image formed is virtual.
1. All distances are measured from the optical New Cartesian sign convention for refraction at a
centre of the lens. spherical surface.
2. The distances measured in the same direction as 1. All distances are measured from the pole of the
the incident light are taken positive. spherical surface.
9.32 PHYSICS-XII

2. The distances measured in the direction of NM _


a - tan a =
[v P is close to M]
incident light are positive. OM OP

3. The distances measured in the opposite direction P - tan P =


NM , NM
of incident light are negative. MI PI

Assumptions used in the study of refraction at a and Y - tan Y =


NM^ NM
spherical surface : MC PC

1. The object taken is a point object placed on the From Snell's law of refraction.
principal axis.
sin I = pj sm r
2. The aperture of the spherical refracting surface
is small. As / and r are small, so

3. The incident and refracted rays make small sin i - i and sin r - r
angles with the principal axis so that the sines or

w
tangents of these angles may be taken equal to Pii = P2r
the angles themselves. or
P^[a+Y] = P2[Y-P]
Refraction at a convex spherical surface :
(0 The object lies in rarer medium and the image
formed is real. In Fig. 9.59, APB is a convex refracting Floor ^
OP
+ NM
PC
= P2
NM
PC
NM
PI

ee
1 1
surface which separates a rarer medium of refractive or + J 1_
= ^2

Fr
OP PC
index p j from a denser medium of refractive index ^2- PC PI

Let P be tlie pole, C be the centre of curvature and


P2_P2~Pl
R = PC be the radius of curvature of this surface. or +
OP PI PC
for
ur
Suppose a point object O is placed on the principal
axis in the rarer medium. Starting from the point object Using new Cartesian sign convention, we find
O, a ray ON is incident at an angle i. After refraction, it Object distance. OP = -w
ks

bends towards the normal CN at an angle of refraction


Image distance, PI = + v
Yo

r. Another ray OP is incident normally on the convex


oo

Radius of curvature, PC = + R
surface and passes undeviated. The two refracted rays
meet at point I. So I is the real image of point object O.
B

A -U V R
re

Rarer-p, r
Denser - P2 or
^2 1-^1 _^2 “1-4
r V u R
ou
ad

a Note
0
P M C I
If first medium is air, then p^=/ and
Pg = p, we have
Y

R
p l_p-l
R
nd

V u
Re

B
u V
(ii) The object lies in the rarer medium and the
Fi

Fig. 9.59 Refraction from rarer to denser medium, image formed is virtual. When the object O in the
when the image is real.
A

Draw NM perpendicular to the principal axis. Let Rarer - p. I Denser - P2


a, p and y be the angles, as shown in Fig. 9.55. r

In ANOC, i is an exterior angle, therefore, a

I O P M C
i = a -I- Y

Similarly, from A NIC, we have R


V
Y =r+p
B
or
r = Y-P
Suppose all the rays are paraxial. Then the angles Fig. 9.60 Refraction from rarer to denser medium,
i, r, a, p and y will be small.
when the image is virtual.
RAY OPTICS AND OPTICAL INSTRUMENTS

rarer medium lies close to the pole P of the convex


refracting surface, the two refracted rays appear to
diverge from a point I on the principal axis, as shown
in Fig. 9.60. So / is the virtual image of the point
object O.
From ANOC, 1 = a + y

From ANCI, r=p+Y


B
Suppose all the rays are paraxial. Then the angles
i, r, a, P and y will be small Fig. 9.61 Refraction from denser to rarer medium
when the image is real.
.'. a - tan a =
NM _ NM [●.* M is close to P]
OM ~ OP Suppose all the rays are paraxial. Then the angles
NM _NM i, r, a, P and y will be small.
P-tanP =
IM ~ IP a - tan a =
NM _ NP [●.● M is close to P]
OM ~ OP
NM _NM
and y - tern y = NM _NM
MC PC
p - tan P =
MI ~ PI
From Snell's law of refraction.
NM_NM
and y - tan y =
Pj sm I = 1^2 sin r CM ~ CP
As i and r are small, so From Snell's law of refraction, for refraction from
sin i - i and sin r - r denser to rarer medium, we have

pp = P2^ |.i2 sm I = pj sin r


or
Pj(a+y) = P2(P+y) As i and r are small angles, so
[NM NM NM NM
sini-i and sinr-r
or
= ^2 IP PC
OP PC
P2» = l-li^
1 1 1 1

IP^ PC P2(Y-a) = Pi(P + y)


or +
or
OP PC

'NM NM 'NM NM
Pi P2_P2-Pi or
or 1^2 PI CP
OP IP PC CP OP

Using new Cartesian sign convention, we find that J 1_ 1 1


=P1 Pl'^ CP
or
1^2
Object distance. OP = -u CP OP

Image distance, IP = -v
Pi Pz ^
or
Radius of curvature. PC = + R PI OP CP

Hi 1^2 _F2~Hi Using the new Cartesian sign convention, we have


- u - V R
Object distance. OP = -«
H2 Hi H2-H1 Pl = + v
or Image distance,
V u R
Radius of curvature. CP = -R

(Hi) The object lies in the denser medium and the P2 _ Hi H2


image formed is real. Fig. 9.61 shows a convex V - u -R
refracting surface which is convex towards the rarer
medium. The point object O lies in the denser medium. Hi H2_Hi~H2
The two refracted rays meet at point I. So I is the real V u R

image of the point object O.


(iz;) The object lies in the denser medium and the
From ANOC, y = i + a or j =y - a image formed is virtual. If the point object O placed on
From AN/C, r=P+ y the principal axis lies close to the pole of the refracting
9.34 PHYSICS-XII

surface, then the two refracted rays appear to come 9 23 REFRACTION AT A CONCAVE
from the point / as shown in Fig. 9.62. So / is the ' SPHERICAL SURFACE
Virtual image of the point object O.
From ANOC, i + y = a or 1 =a -7 37. By stating the sign conventions and assumptions
From ANIC,
used, derive the relation between object distance, image
r-i-y=p or r = ^-y
distance and radius of curvature of a concave spherical
surface when the refraction takes place (i) from optically
A
r rarer to optically denser mediwn and (ii) from optically
Denser ● pj
N
Rarer-p,
denser to optically rarer medium.
I
For new Cartesian sign convention and the assump
CL
tion used, refer to the answer of the previous question.
C o / M P Refraction at a concave spherical surface,

2 (i) The object lies in the rarer medium. In Fig. 9.63,

w
u

R APB is a concave refracting surface separating two


B media of refractive indices p, and P2-
r

Fig. 9.62 Refraction from denser to rarer medium


when the image is virtual.
Flo Rarer -pj
A

Denser - p2

ee
I
Suppose all the rays are paraxial. Then the angles
I, r, a, P and y will be small. ct--VP .-n>

Fr
o I c M P
. a ~ tan a = NM ^NM ~R
OM “ OP [●.● M is close to P] V
for
ur
P - tan P =
NM _ NM
M ~ IP B

y - tan y = NM ^ NM Fig. 9.63 Refraction at a concave surface when


ks

CM ~ CP the object lies in the rarer medium.


Yo

From Snell's law of refraction, for refraction from Let


oo

denser to rarer medium, we have P = Pole of the concave surface APB


B

P2 sin i = Pj sin r C = Centre of curvature of the concave surface


re

As i and rare small angles, so O = Point object placed on the principal axis
sini-i and sinr-r
/ = Virtual image of point object O
ou
ad

P2^ = p^r In ANOC, y is an exterior angle, therefore


or
P2(a-y) = Mi(3-y) y =a+i or i = y ~a
Y

or
'NM NM' 'NM NM' Similarly, from A NIC, we have
P2 = Pi
.OP CP. y =P+ r
nd

. IP CP or r = y-p
Re

or J 1^' ■j ■]_' Suppose all the rays are paraxial. Then the angles i,
P2
Fi

OP CP
= Pi r, a, p and y will be small.
JP CP.
NM NM
or _Pl I ^^2 _ P1-P2 . a - tan a =
OM OP
[●.● M is close to P]
IP OP CP
NM„ NM
Using the new Cartesian sign convention, we have P - tan P -
IM IP
Object distance. OP = -u
NM_ NM
y- tan y =
Image distance, IP = -v CM CP
Radius of curvature, CP = -R
From Snell's law of refraction,
^ P2 _ P1-P2 Pj sin I = P2 sin r
-R
-v -u
As i and i are small angles, so
or
Pi P2_Pi~P2 sin i - i and sin r - r
V ii R
pp' = P2r
RAY OPTICS AND OPTICAL INSTRUMENTS

or
M, [Y-a] = ^2[Y-Pl or
^2[a + Y] = Mi[P + Yl
' NM NM NM , tm NM NM
NM NM
or
or
= ^2 ^^2 PC IP PC
CP OP . CP IP OP
1 1 1 1
1 1
J
IP ^ PC
+
or
Pi
or
P2 = P1
CP OP CP IP. OP PC

Pi Pi P1-P2
Pi , P2 ^P2"Pl
IP ^ OP
or or
OP IP CP PC

Using new Cartesian sign convention, we find Using new Cartesian sign convention, we find
OP ^-u
Object distance, OP = -u Object distance,
lP = -v
Image distance, IP = -v Image distance,
Radius of curvature, CP = ~ R Radius of curvature. PC = + R

-Pi +
P2 _ P2 Pi "Pt , P2 _Pl~P2
- u - V - R -V - u R

P2 Pi _P2~Pl Pi f^2 _ Pi P2
or
or
V u R V u R

{ii) The object lies in the denser medium. As For Your Knowledge
shown in Fig. 9.64, when the point object O is placed in > For both convex and concave spherical surfaces, the
the denser medium, the refracted rays appear to refraction formulae are same, only proper signs of ii, v
diverge from a point I in the denser medium. So I is the and R are to be used.
virtual image of the point object O. > For refraction yrom rarer to denser tnedium, the refraction
From A NOC> i=a +y formula is
P2 Pi P2-P1 ...(1)
From A NIC, r =p + Y
V u R
A
> For refraction from denser to rarer medium, we inter
Denser - N Rarer-jij
I r change |ij and and obtain the refraction formula.
Pi P2 P1-P2 ...(2)
V u R
a

O 1 P M C > If the rarer medium is air (|ij = 1) and the denser


V
medium has refractive index f (f.e., 1^2 = m)/ then for
refraction from rarer to denser medium, from (1) we
u R
get the relation :
B P _1-P“^ ...(3)
V u R
Fig. 9.64 Refraction at a concave surface when the For refraction from denser to rarer medium, we put
object lies in the denser mediuin.
/H2 = 1/P P* (2) relation :
Suppose all the rays are paraxial. Then the angles i, 1/p 1 (1/P)-1 -(4)
r, a, p and y will be small. V u R

a - tan a = NM ^ ^ [●.* M is close to P] > For an object placed in air, the refraction formula (3) is
OM " OP applicable, i.e., — — “
V u R

P - tan p = NM ^ hm
IM ~ IP As R is positive for a convex surface, v will be negative
if the value of u is less than R / (p - 1). In that case, the
Y - tan Y = NM ^ ^ image will be formed in air and will be virtual.
MC ~ PC
As R is negative for a concave surface, the value of v
From Snell's law of refraction, for refraction from will also be negative for all negative values of u. Thus
denser to rarer medium, we have image will always be formed in air and will be virtual.
ji 2 sin / = p j sin r > The factor ——^
R
is called poiver factor of the
As i and r are small angles, so
spherical refracting surface. It gives a measure of the
sin/-i and sinr-r degree to which the refracting surface can converge or
diverge the rays of light passing through it.
P2' = Pi^
PHYSICS-Xll

Examples based on Therefore,


Refraction through Spherical Surfaces h = -12 cm. R = + 3cm, |.i2=1.5
Formulae Used

1. For refraction from rarer to denser medium, 1.5

V u R

2. For refraction from denser to rarer medium,


lf_L _ _ Ml ~\^2
V u R

3. Power of a surface. Fig. 9.65


P-M2-M1 -1 (For air) M2

w
R R As
V u R
PiR
4. First principal focal length. /i=- 1.5 1 1.5-1 0.5
M2-M1
V 12 3 3

5. Second principal focal length, /j =


M2-Mi
Flo
or 1.5
V
0.5
3
1 _2-l
12
1
12 “12

ee
fi _ M2 or u = 1.5 X 12 = 18 cm
A Ml

Fr
As V is positive, so a real image is formed at 18 cm
Units Used from the end P of the dumbbell.

Distances u, v,f and R are in metre, power P is in Example 38. The diameter of a glass sphere is 10 cm. A
for
ur
dioptre (D), refractive indices ^1,^2 M have beam of light strikes the sphere, which converges at point
no units.
20 cm behind the pole of the spherical surface. Find the
position of the image ifp =1.5. [CBSE OD 19]
Example 36. Light from a point source in air falls
ks

on a
convex spherical glass surface {p. =1.5, radius of curvature Solution. In the absence of glass sphere, the light
Yo

rays wdll converge at point O. So O acts as virtual object


oo

= 20 cm). The distance of light source from the glass surface


is 100 cm. At what position is the image formed ? for the image / for refraction at the first surface.
B

[NCERT ; CBSE OD 16]


Solution. Here|ij =1, «=-100cm
re

o
I
R = + 20 cm,
ou
ad

[R is +ve for a convex refracting surface] 15 cm


Y

As =
v u R
Fig. 9.66
nd

1.5 1 _ 1.5-1 _ 1 u = PO = + 20 cm,


Re

+
V lOO” 20
Mi=l
Fi

or A-J__ 1 _ 3 R = + — = + 5 cm
2v ~ 40 100 ~ 200 2

v= + 100 cm As the light passes from rarer to denser medium, so


M2 Ml _M2~Mi
Thus the image is formed at a distance of 100 cm
V u R
from the glass surface, in the direction of incident light.
1.5 1 1.5-1 0.5 1
Example 37. A glass dumbbell of length 30 cm and
or
V 20 5 ~ 5 ~10
refractive index 1.5 has ends of 3 cm radius of curvature.
Find the position of the image formed due to refraction at one
or
u ~io 20
end only, when the object is situated in air at a distance of 20
V = + —X 1.5 = + 10 cm
12 cm from the end of the dumbbell along the axis.
or
3
Solution. Refraction occurs from air to glass at Thus the image is formed at the other end (/) of the
convex spherical surface APB. diameter.
RAY OPTICS AND OPTICAL INSTRUMENTS 931

Example 39. What curvature must be given to the As light passes from denser to rarer medium, so
bounding surface of p -1.5 for virtual image of an object in Ml P2 _ -^2
the medium of\i=lat W cm to be formed at a distance of V u R

40 cm. Also calculate power of the surface and two principal or 1 ^ 1.5 _ 1-1.5
focal lengths of the surface. v 20 -10
Solution. Here H = -10cm, t; = -40cm, =X
or
1 1 3 ^2-3 _ 1
|i2=1.5 V 20 40" 40 40
As the object is placed in the rarer medium, so or u = - 40 cm

Negative sign shows that the image is virtual. It is


R V It formed on the same side of the refracting surface as the
object at a distance of 40 cm from the pole P.
1.5-1_ 1.5
or
R ”-40 10 40 16 Example 41. A small air bubble in a glass sphere of radius
2 cm appears to be 1 cm from the surface when looked at,
or i? = 16 X 0.5 = 8 cm
along a diameter. If the refractive index of glass is 1.5, find
As R is positive, the refracting surface is convex. the true position of the air bubble.
Power of surface. Solution. Here incident ray OA is in glass and
1.5-1 0.5 refracted ray AB is in air. / is the final image of the air
= 6.25 D bubble at O.
R 8 cm 0.08 m

First principal focal length,


1x8
= -16 cm
/i=- 0.5
M2-M1 = 1

Second principal focal length,


>
p^R 1.5 X 8
= 24 cm
fi 0.5
M2-M1

Example 40. A point 'O' marked on the surface of a glass


sphere of diameter 20 cm is viewed through glass from the 2cm —►
position directly opposite to the point O. If the refractive
index of the glass is 1.5, find the position of the image Fig. 9.68
formed. Also, draw the ray diagram for the formation of the R = -2 cm
image. [CBSE OD 19] Hereii,=l, t?=-lcm.
Ml M2 _Mi -M2
Solution. Figure 9.67 shows a glass sphere of As
v u R
radius 10 cm. The mark O on its surface acts as object.
The incident ray OA is in glass and refracted ray AB is
1 1.5 1-1.5 1
or
1.5 1
.1 = 1
-1 -2 4 u 4 4
in air. / is the image of O. Thus (/

1.5 X 4
=1, p2=1-5' H = OP = -20cm or u = -
5
= -1.2 cm

R = -10 cm [Minus sign taken for. refraction


at concave surface] Thus the air bubble O lies at 1.2 cm from the
refracting surface within the sphere.
Example 42. An empty spherical flask of diameter 15 cm is
placed in water of refractive index A parallel beam of light
1
strikes the flask. Where does it get focussed, when observed
I from within the flask ?
Solution. Figure 9.69 shows a spherical flask
placed inside water. The centre of the flask is the centre
of curvature of the spherical refracting surface. A
parallel beam of light falling on the flask diverges and
Fig. 9.67
appears to come from point 1.
PHYSICS-XII

1 1.5 _ 1-1.5
From 00
i>2 15 -15
— = J-
A-p
^2 10 " 30
/
A = J_ Jl-A
^2 10 30 "30
1^2 “ 7-5 cm
Distance of the final image from the centre
Fig. 9.69 = (15 + 7.5)cm = 22.5 cm.
4 15
Here |a j = 1, H2-3- « = - CO, K = + — cm
roblems For Practice
2
As tlie light travels from denser to rarer medium,

w
so 1. A spherical convex surface of radius of curvature
or
1 4/3 1-4/3 20 cm, made of glass (p = 1.5) is placed in air. Find
V u R V - 00 15/2 the position of the image formed, if a point object is
-45 placed at 30 cm in front of the convex surface on the
or V =
2
= -22.5 cm.

Example 43. A parallel beam of light is allowed to fall Flo principal axis. [CBSE SP 18]
(Ans. Virtual image at 180 cm in air)

ee
on a

transparent spherical globe of diameter 30 cm and refractive


f
Fr
index 1.5. Find the distance from the centre of the globe at
which the beam of light can converge. tJEE Main June 22] O P C
Solution. If the glass medium were continuous, the 30 cm 20 cm —H
for
ur
parallel rays would have met the principal axis at point
So /’ is the real image of the object at infinity.
30
ks

p=1.5, i/ = -oo, R= + = +15 cm


2 ●Fig. 9.71
Yo

Let PV = V 1
oo

2. A convex refracting surface of radius of curvature


' First surface
20 cm separates two media of refractive indices 4/3
B

and 1.60. An object is placed in the first medium


Second surface

A
= 4 / 3) at a distance of 200 cm from the refracting
re

Final image surface. Calculate the position of the image formed.


P
From 00
^ r_.-. (Ans. At 240 cm in denser medium)
ou

'C
ad

/
3. A concave spherical surface of refractive index 3/2 is
immersed in water of refractive index 4/3. If a point
Y

object lies in water at a distance of 10 cm from the pole


cm 15 cm-*!*- 15 cmH of the refracting surface, calculate the position of the
nd

image. Given that radius of curvature of the spherical


Re

Fig. 9.70
surface is 18 cm. (Ans. Virtual image at 10.52 cm
For refraction from rarer to denser medium,
Fi

from pole and inside the water)


4 l_p-l
4. A mark placed on the surface of glass sphere is
V u R
viewed through glass from a position directly oppo
_ 1.5 1 1.5-1 site. If the diameter of the sphere is 10 cm and
V
1
00 15 refractive index of glass is 1.5, find the position of
Uj = 45 cm the image. (Ans. 20 cm towards mark from
the surface opposite to mark)
The rays before meeting at the point /'suffer another
5. A glass sphere of radius 15 cm has a small bubble
refraction at the second surface. The real image /'acts
as virtual object and / is tlie final real image. 6 cm from its centre. The bubble is viewed along a
diameter of the sphere from the side on which it
M = PT= PV- PP'=45 -30 =15 cm, R =-15 cm
lies. How far from the surface will it appear to be
For refraction from denser to rarer medium, if the refractive index of glass is 1.5 ?
1 P_l-P (Ans. Virtual image is seen at 7.5 cm
R
V u
from the spherical surface)
RAY OPTICS AND OPTICAL INSTRUMENTS 9.39

6. An object is placed 50 cm from the surface of a glass 3. Here u-- 10cm, pj = 4/3, ^2 = 3/2, R = - 18cm
sphere of radius 10 cm along the diameter. Where ...Mz-Mi . 3/2 4/3^3/2-4/3
will the final image be formed after refraction at As^- -18
V u R V -10
both the surfaces ? p of glass = 1.5.
or V = -10.52 cm, in water.
(Ans. At 20 cm from the centre of the sphere)
7. A spherical surface of radius 30 cm separates two 4. Here» = -10cm, R = - 5cm, Pj = 1, P2 = 1-5, u = ?
As refraction occurs from denser to rarer medium
transparent media A and B with refractive indices
at concave surface, so
1.33 and 1,48 respectively. The medium A is on the
convex side of the surface. Where should a point or
1
- +
1.5 1-1.5_ 1
object be placed in medium A so that the paraxial V u R V 10” -5 10
rays become parallel after refraction at the surface ?
On solving, v~-20 cm.
(Ans. At 266 cm from the pole)
5. Refer to Fig. 9.73. Here u= PO = 9cm, Pi = 1,
8. Figure 9.72 shows a small air bubble inside a glass
P2 = 1.5, v = l
sphere (p = 1.5) of radius 10 cm. The bubble is Use formula for refraction from denser to rarer
4.0 cm below the surface and is viewed normally
medium.
from the outside. Find the apparent depth of the
bubble. (Ans. 3 cm below the surface)
-N

h-15 cm —'

Fig. 9.73
Fig. 9.72
6. Refer to Fig. 9.74. For refraction at first face AP^.
» = -50cm, R = +10cm, pj = 1, P2 = 1-5, u'=?
HINTS
N
1. Herepj=l, p2=1.5, » = -30cm, R = +20cm .N
As light travels from rarer to denser medium, so A'
B.''
M2 Ml _ ^2-1^1
V u R
I1
0 Pz I
or 1.5 ^ 1 ^ 1
V 30 20 40 h-M
u v'
or
1.5 1 1 _ 3-4 1

V 40 30” 120 120 Fig. 9.74


y=-1.5x120 = -180 cm
As refraction occurs from rarer to denser medium,
The negative sign shows that a virtual image is
formed in air. M2 Ml M2-M1 1.5 1 1.5-1 0.5
so or
V u R v' 50 10 10
2. Here p^ = 4 / 3, P2 = 1-60, u = - 200 cm, R = + 20 cm
On solving, v' = 50 cm.
M2 Ml
As
V u R For refraction at second surface BP^. The real image
1.60 4/3 1.60-4/3 /j acts as virtual object,
V -200 20 w = /j ?2 = v' - 2R = 50 - 20 = 30 cm,
or v = + 240 cm, in denser medium. v = ?, R = -10cm.
PHYSICS-XII

As refraction occurs from denser to rarer medium Lens maker's formula for a double convex lens. As
at concave surface, so shown in Fig. 9.75, consider a thin double convex lens
1 1.5 1-1.5
or of refractive index ^2 placed in a medium of refractive
V u R V 30 ~ - 10 index p,. Here Pj <^2- Let B and D be the poles,
On solving, t; = 10 cm and Cj be the centres of curvature, and and 1?2
be the radii of curvature of the two lens surfaces ABC
Distance of final image from the centre of the
sphere = 10 + 10 = 20 cm. and ADC, respectively.
7. Here R = + 30cm, pj=1.33, Normal
at Ad
Normal
atN
Pj - 1.48, v=<30, u = ? A

Pi Pi
V u R
1.48 1.33 1.48-1.33 B D

w
or *■
O C2 I
00 u 30 C, I1

1.33x30
or u = ~ = - 266 cm. R, R1
0.15 c

8. Herep, = ],p2 = 1.5^ »= OP= - 4cm, R = -10cm


Fi F2_Fi-P2 . 1 1,5 _ 1-1.5 Flo Fig. 9.75 Refraction through a double convex 1ms.
Vi

ee
As
V u R V -4" -10
Suppose a point object O is placed on the principal

Fr
40
or
K = -—=●* 3 cm, inside glass sphere. axis in the rarer medium of refractive index p^. The ray
1 o OM is incident on the first surface ABC. It is refracted
along MN, bending towards the normal at this surface.
for
ur
9.24 LENS MAKER'S FORMULA If the second surface ADC were absent, the ray MN
38. Derive the lens maker's formula for a double convex would have met the principal axis at So we can treat
lens. State the assumptions made and the convention of Ij as the real image formed by first surface ABC in the
ks

signs used. medium of refractive index P2.


Yo

For refraction at surface ABC, we can write the


oo

Lens maker's formula. This formula relates the focal


length of a lens to the refractive index of the lens material relation between the object distance u, image distance
B

and the radii of curvature of its two surfaces. This formula and radius of curvature R^ as
P2 Pt
is so called because it is used by manufacturers to
re

...(1)
R
design lenses of required focal length from a glass of ^1 u
1

given refractive index.


ou

But actually the ray MN suffers another refraction


ad

New Cartesian sign convention for spherical lenses: at surface ADC, bending away from the normal at
Y

(i) All distances are measured from tlie optical point N. The emergent ray meets the principal axis at
centre of the lens.
point I which is the final image of O formed by the
(ii) The distances measured in the direction of lens. For refraction at second surface,acts as a virtual
nd
Re

incident light are positive. object placed in the medium of refractive index P2 and
(m) The distances measured in the opposite ^ image formed in the medium of refractive
Fi

direction of incident light are negative. index pj. Therefore, the relation between the object
Assumptions made in the derivation of lens distance Vy image distance v and radius of curvature
maker's formula : R2 can be written as
(/) The lens used is thin so that the distances Pi P2 _ Pi P2
...(2)
V V
measured from its surfaces may be taken equal 1

to those measured from its optical centre.


Adding equations (1) and (2), we get
(ii) The object is a point object placed on the
principal axis. J ^
(Hi) The aperture of the lens is small. v
^ = (P2-Pi)
u
R, R2
(iv) All the rays are paraxial, i.e., they make very 1
small angles with the normals to the lens faces or 1- ^^2~Pl 1 1
...(3)
R1
and with the principal axis. V u
RAY OPTICS AND OPTICAL INSTRUMENTS 9.41

If the object is placed at infinity = CO), the image object distance image distance v and radius of
will be formed at the focus, i.e., v = f. Therefore, curvature Kj can be written as
1 _ 1 1
...(4) ...(2)
/ ^1 R
1 V V
1

This is lens maker's formula. Adding equations (1) and (2), we get
When the lens is placed in air, Pj =1, and P2 ^^1 1 1
The lens maker's formula takes the form :
V u R
1 1 1

/ R1
or
1
i_ 1 1_
From equations (3) and (4), we have
V 11
JL^i
1 1 1 If an object is placed at infinity, then the image is
formed at the focus i.e., v = f, so
V
u f
This is the thin lens formula which gives relationship 1_ 1 1_
between u, v and /of a lens. f ^4 JL^i ^
39. Derive the lens maker's formula for a double This is lens maker's fortnula.
concave lens.
When the lens is placed in air, p j = 1 and p 2 = P- The
Lens maker's formula for a double concave lens. As maker's formula takes the form :
shown in Fig. 9.76, consider a thin double concave lens
4 = (F-1) J 1_
1
of refractive index P2 placed in a medium of refractive
Pj. Herepj <P2- Let Band £be the poles, and Kj and / Rj R2
R2 be the radii of curvature of the two lens surfaces
ABC and DEF, respectively.
Examples based on
A D Lens Maher's Formula
M2
M 'Hhl Formulae Used
Ml Ml

1. For the lens of material of refractive index P2


O I1
-►
I B E
placed in a medium of refractive index p^,
1 1
1 _ M2-M1
V,
/ 1^1 R, R2
u

C F
2. When the lens is placed in air,
Fig. 9.76 Refraction through a double concave lens. Pj = 1 and P2 = P-
1 iJ 1 1
Suppose a point object Ois placed on the principal
axis in the rarer medium of refractive index pj. First the
" / ^ Rj. ■
spherical surface ABC forms its virtual image /. As 3. f and R are positive for convex surfaces and
refraction occurs from rarer to denser medium, so we negative for concave surfaces.
can write the relation between object distance u, image Units Used
distance v,1 and radius of curvature R,1 as
Focal length / and radii of curvature Rj and Rj are
P2_Pi _P2-Pi ...(1) in metre, refractive indices Pj / P2 ^nd p have no
^1 U units.

But the lens material is not continuous. The ray MN


suffers another refraction at N and emerges along IN. Example 44. The radius of curvature of each face of
biconcave lens, made of glass of refractive index 1.5 is 30 cm.
So I is the final virtual image of the point object O. The
Calculate the focal length of the lens in air.
image / acts as an object for refraction at surface DEF
from denser to rarer medium. So the relation between Solution. Here p =1.5, Rj =-30 cm, R2 = + 30 cm
PHYSICS-XIt

Using lens maker's formula, Example 48. Find the radius of curvature of the convex
surface of a plano-convex lens, whose focal length is 0.3 ni
^=(4-1)
1 1

/ R. and the refractive index of the material of the lens is 1.5.


[ISCE 98 ; CBSE D 10]
1
= (1.5-1) L =-0.5x—= 1
Solution. Here p = 1.5, / = + 0.3 m, Rj = oo, = -R
-30 30 30 30
Using lens maker's formula.
/ = -30 cm .
Example 45. The radii of curvature of the faces of a double J 1_
convex lens are 10 cm and 15 cm. If focal length is 12 cm, f Rl \
zohat is the refractive index of glass ? (NCERT] 1 1
or “ = (1-5-1) — H
Solution. Here f = +12 cm, R, = + 10 cm. + 0.3 00 R

w
R2 = -15 cm, fi = ? 1 1
or = 0.5x— or R = 0.15 m.
1 1 1 0.3 R
As ^=(^1-1)
/ R
Example 49. The focal length of a convex lens made of
1
— = (|i-l) — + — =(u-l)x
1

11 / 5
Flo glass of refractive index (1.5) is 20 cm. What will be its new
focal length when placed in a medium of refractive index

ee
12 ' llO I5J 30
1.25 ? Is focal length positive or negative ? What does it

Fr
or ]x-l = — =Q.5 p = 1.5. signify ? [CBSE SP 22]
12
Solution.
Example 46. A biconvex lens has a focal length 213 times For the glass lens in air,
a
)i.g = 1.5
for
ur
the radius of curvature of either surface. Calculate the
refractive index of lens material. (CBSE D 10] 1 1 1 1 1
2 = (1.5-1)
R
Solution. Here f = — R, Rj = R, R2=-R 1 ^2 R
1 ^2
ks

3
Yo

1 1 1 For the glass lens in medium, '’p = 1.25


oo

ni
As ^ = (h-l)
/ R.1 R, \ _
a
2> 1
B

= ('"^ 1) -1
(1 n
,|=(.-i) f
a

Rj Rn
R^ R
m III
re

or p =1 + 0.75 = 1.75 1^-1 ' 1 1_ 0.25 J 1_


J Rj R^
ou
ad

1.25
1.25 R^ R2
Example 47. The radii of curvature of a double convex lens
Y

0.5x1.25
of glass (p =1.5) are in the ratio 1:2. This lens renders the
III —
= 2.5

rays parallel comingfrom an illuminated filament at a distance fa 0.25


nd

of 6 cm. Calculate the radii of curvature of its surfaces.


Re

fy = 2.5 X 20 cm = +50 cm.


Solution. Here/= +6 cm, p=1.5, R^ =+R,
Fi

New focal length is positive. It signifies that the


R2=-2R given lens is still converging in the given medium.
As -=ii^ -1) — Example 50. The radii of curvature of a double convex lens
/ R
1
are 10 cm and 20 cm respectively. Calculate its focal length
1 when it is immersed in a liquid of refractive index 1.65. State
- = (1.5-1) -+ the nature of the lens in the liquid. The refractive index of
6 I R 2R
1 glass is 1.5.
or = 0.5 X —
6 2R Solution. Here Rj =+10 cm, R2=-20cm,
0.5 X 3 X 6 a n
R =
or
2
- 4.5 cm
P;=1.65, “P^=1.5
a

Rj = + R =+ 4.5 cm I
- ^-5 ^10
and
R2 = - 2 R = - 9.0 cm.
a

h/ 1.65 " 11
RAY OPTICS AND OPTICAL INSTRUMENTS

1 I Dividing (i) by (ii), we get


Now 4 = (V 8 u -1
f R
1
100
or 3|i -4=^1 -1
25 3^1 -1

.11 JLlO 20. 3


4

or ~ = 1.5.
_J_ 3 3
11 ^ 20 ~ 220
Example 53. Aji equiconvex lens of focal
A
or f = -73.33 cm length 15 cm is cut into two equal halves as
The negative value of / indicates that the lens shown in Fig. 9.77. What is the focal length
becomes diverging when immersed in the given liquid, of each half 7
V
(CBSE SP 981

Example 51. If the refractive index from air to glass is 3/2 Solution. For the equiconvex lens, let
and that from air to water is 4/3, find the ratio of focal Rj = + R, R^ = - R Fig. 9.77
lengths of a glass lens in water and in air. Then from lens maker's formula,
a 3 « 4
Solution. Here w
"3' l^ll_2(p-l)
R R R
u>
1^
Vg 3/2 _9 For each half lens, Rj = R, R2 = — 00.
8 a
4/3 8
h-1
Let f^ and be the focal lengths of glass lens in — =(u -1) ■■■{ii)
water and air respectively. Then /' [r -00 R

f
1
= (>g-l)
1 1
...(0 Dividing (i) by (ii), we get ^
/,
Ti>
R
1
or /'=2/=2xl5 = 30 cm.
1 1
...(«/)
fa
8 R
1
roblems For Practice

Dividing (ii) and (i), we get 1. The focal length of an equiconcave lens is ^ times of
a
i-1 radius of curvature of its surfaces. Find the refractive
/,TO — _ 2 = 4:1. index of the material of the lens. Under what con-
i-1
w

fa h.-l
8 8 dition will this lens behave as a converging lens ?
ICBSE OD 20J (Ans. g = 5/3)
Example 52. A double convex lens has a focal length of 25
2. Write the Lens maker's formula and use it to obtain
cm in air. When it is dipped into a liquid of refractive index
4/3, its focal length is increased to 100 cm. Find the the range of values of p (the refractive index of the
refractive index of the lens material. material of the lens) for which the focal length of an
equiconvex lens, kept in air, would have a greater
Solution. For the lens in air : f„ =25 cm. magnitude than that of the radius of curvature of its
[CBSE D 16C]
Let |i = refractive index of lens material two surfaces.
relative to air 3. A biconvex lens has a focal length half the radius of
1 1 1 ’ curvature of either surface. What is the refractive
Then — =(p -1) index of lens material ? (Ans. [i = 2)
R
1
4. Figure 9.78 shows a thin lens with centres of
1 1
or
1
...(0 curvature Q and C2. Find its focal length. Take
25 R |i =1.5. (Ans. + 40 cm)

/f
1

4
For the lens in liquid : f = 100 cm.
1 1 1 C,
i^-1
fi 1^1 R
1

or
1
-1 _1 1_ ...(ii)
10 cm

100 U/3 - 20 cm —

Fig. 9.78
.44 PHYSICS-XII

5. A converging lens has a focal length of 20 cm in air. 13. A converging lens of refractive index 1.5 and of
It is made of a material of refractive index 1.6. If it is focal length 15 cm in air has the same radii of
immersed in a liquid of refractive index 1.3, curvature for both sides. If it is immersed in a liquid
what will be its new focal length ? of refractive index 1.7, calculate the focal length of
[CBSE OD 11; F 17] the lens in the liquid. ICBSE OD 08]
(Ans. 52 cm) (Ans. - 63.75 cm)
6. A plano-convex lens (p = 1.5) has a curved surface HINTS
of radius 15 cm. What is its focal length ?
(Ans. 30 cm) 1, Here/ = ~-R,R =-K, It = + R
4
7.
A plano-convex lens p =1.5 has focal length of 1 ^
18 cm in air. Calculate the radius of curvature of the As

spherical surface. [CBSE F 94]

w
(Ans. 9.0 cm)
3R Iv K R)
The focal length of a concavo-convex lens of radii of
5
curvature 5 cm and 10 cm is 20 cm. What will be its or or
^ 3
focal length in water ? Given “ p u>
= 4/3.

(Ans. - 80cm)
Flo This lens will behave as a converging lens when

ee
9.
A convex lens of focal length / and refractive index immersed in a liquid with P >
1.5 is immersed in a liquid of refractive index (i) 1.6

Fr
2. According to Lens maker's formula.
(ii) 1.3 and (Hi) 1.5. What changes happen to the
focal length of the lens in the three cases ? 1
for
[Ans. (0 - 8 / (ii) + 3.25 / (Hi) co] /
ur
10. The focal length of a plano-convex lens is 20 cm in For an equiconvex lens : = R, R2 = -R
air. Refractive index of glass is 1.5. Calculate (/) the 1 R
●●● 4 =(1^-1) 4R
ks

radius of curvature of lens surface and (ii) its or / =


/ 2(p-l)
Yo

focal length when immersed in liquid of refractive


oo

index 1.6, [Ans. (i) - 10 cm (ii) - 160 cm] For / to greater than R,
11.
The focal length of a glass convex lens in air is 2(p -1) < 1 or 2p < 3 or p < 1.5
B

15 cm. Calculate its focal length, when it is totally Hence, the required range for the values of p is
re

immersed in water. Given Py, =4/3 and 1.0 < p < 1.5.
‘’p S =1.5. [Punjab 01]
3. Here f = R/Z Rj=R, I?2=-R
ou

(Ans. 60 cm)
ad

12. The radius of curvature of each surface of a convex 1 2 f 1


or p = Z
Y

lens is 20 cm and the refractive index of the material


of the lens is 3/2. (/) Calculate its focal length, (ii) If 4. Here both the radii of curvature are positive.
this is cut along the plane AB, what will be the focal
nd
Re

length of the each of the two halves so formed ? R,=+10cm, 1^2=+20cm


4=(1.5-1) —- —
1
(Hi) What happens if the lens is cut along CD ?
Fi

[Ans. (0 20 cm (/«) 40 cm (m)/remains /f 10 20 40

same but intensity of the image decreases] or


/ = + 40 cm.

A
5. Using lens maker's formula when the lens is placed
in air.

1 P
-1 1 1_
C
fa
■D

1 1.6
or -1 1
20
1 JL^
B

or J__ J. 1 1

Fig. 9.79 R, R2 20x0.6 12


RAY OPTICS AND OPTICAL INSTRUMENTS 9.45

When the lens is placed in the liquid, 1


1
ri.5 -il-
O'O
1 1 1 ' /, R, U-3 ) f
-1
// JL^ or
/,= + 3.25/.
1.6 0.3
1 ri.5 2 ^ 00.
1.3 12 1,3x12
Ji U-5
1.3x12
or
fl- 0.3
= 52 cm. 10. (i) For the lens in air:

/^=20cm, |a=1.5, Rj = co, R^=l


6. Here p = 1.5, Rj=co, R2~~l5cm
1
1 1
- + —
CO 15 fa

= 0.5x—= —
15 30
/ = 30 cm.
or
20
1 = (1.5-1)[-- CO
1

7. Here p= 1.5, / = +18cm, R, = oo, 1^2="^ ^2 = “


Using lens maker's formula. (ii) For the lens in liquid:
1 1 P2 = 1-5, P = 1-6, Rj =co, i?2 =-10cm
/ 1 ^^2 -1 1 1

1 1 1 1 fi
= (1.5-1) -+- or - = 0.5 X
18 CO R 18 R ri.5
N ●
1 1 1
-1
or R = 0.5 X 18 = 9.0 cm. U.6 y . oo 10 160

8. Here/j, =-20cm, /^=-5cm, R2 = -10cm /j = -160 cm.


1 1 1
As T-=(V.-i) _1 l_ 11.
fa
8
R,“R2 fa
1

-1)^
1 1 or = (1.5-1) —- —
= (V 15
-20 -5 -10

a J__ jL = i-
^, = 3/Z
or
R, R2 15
When the lens is placed in water.
a
1
Now — = -1 1 l_ ( —1-5 ,11 X —
2
1 1 1
-1 /.
w ai R, R, U/3 15
a
fa u
R, R, 1 2 1

8 15 60
or 4 = 60 cm.
= -1 _ 1 1

4/3 5"^ 10 80
13. Here =15cm, p^ = 1.5, p, = l-7, /,=?
or
4 = - 80 cm. For the lens place in air :
1 / n 1 ^
9.
1
T=(V _ 1) [ _L —L f. [rj Rj
/ R,J
1 1 1
i = (1.5-1) r"^ rJ r
= (1.5-1) J__ J_
or
15
R,~ R^
For the lens immersed in liquid :
or 2 ^__2 1
-1 \L-J-
R, R,"/ /; U‘; R, R2
1 1 1.5 .'l 2 = 'll V 1 1 "1 ^ 1
(o4= -1
1.6
1
j
X —
/ . 1.7
-1
R R J 1.7 R
ft tV; JL^ ^ 0.2 2
— X— or fi = - 63.75 cm.
or
/, = -»/● 1.7 15
9A6 PHYSICS-Xli

9.25 RULES FOR DRAWING lAAAGES Formation of images by spherical lenses :


FORMED BY SPHERICAL LENSES (fl) Object beyond 2F. The image is
40. State the rules used for drawing images jbrmed by (O between F and 2F (li) real
spherical lenses.
(Hi) inverted (iv) smaller
Rules for drawing images formed by spherical
lenses. The position of the image formed by any
A
r
A
spherical lens can be found by considering any two of F I 2f

the following rays of light coming from a point on the O 2F F Image


object. B

(a)
(i) A ray from the object paraUel to the principal
axis after refraction passes through the second princi- (&) Object at 2f. The image is
pal focus ^2 [hi a convex lens, as shown in Fig. 9.80(fl)]

w
(i) at2f {ii) real
or appears to diverge [in a concave lens, as shown in (iff) inverted (iv) same size
Fig. 9.80(b)] from the first principal focus fj.

Flo I
2F

Image

ee
V 6

Fr
(h)
V ZJA (c) Object between IF and F. The image is
(a) (h)
(i) beyond IF (ii) real
for
ur
Fig. 9.80 Path of ray incident parallel to the principal axis of (ill) inverted (iz;) larger
(o) convex lens (b) concave lens.

(if) A ray of light passing through the first principal


ks

focus [in a convex lens, as shown in Fig. 9.81(fl)j or


Yo

appearing to meet at it [in a concave lens, as shown in


oo

Fig. 9.81(b)] emerges parallel to the principal axis after


B

refraction.
(c)
(rf) Object between F and O. The image is
re

(i) behind object (ii) virtual


(Hi) erect (it;) larger
ou
ad

B
^2 ^2
Y

V
(a) (b)
lA I

!
nd

i
Re

Fig. 9.81 Path of a ray passing through focus of /


(a) convex lens (b) concave lens.
Fi

(Hi) A ray of light, passing through the optical (cO


centre of the lens, emerges without any deviation after (e) Object in any position. The image is
refraction, as shown in Figs. 9.82(a) and (b). (i) in front of object (ii) virtual
(Hi) erect (iv) smaller

A
2
B,"
F
O F I

Fig. 9.82 Path of a ray passing through the optical centre lA (e)
(a) convex lens (b) concave lens.
Fig. 9.83 Formation of images by spherical lenses.
RAY OPTICS AND OPTICAL INSTRUMENTS

But MO = AB,
9.26 THIN LENS FORMULA
A B' _ FB'
41. State tJie lens formula. Is the same formula ...(2)
AB ~ OF
applicable to both convex and concave lenses ?
Thin lens formula. Thin lens formula is a mathe From (1) and (2), we get
matical relation between the object distance u, image OB' _ FB' OB' - OF
distance v and focal length / of a spherical lens. This lo~~OF OF
relation is :
Using new Cartesian sign convention, we get
i_i = i
V u f Object distance, BO = - u
Image distance. OB'=+v
In words, we can say that
1 1 1 Focal length, OF = + f
Image distance Object distance Focal length V v-f
This formula is valid for both convex and concave - u f
lenses for both real and virtual images. or vf = ~ uv + uf or uv = uf- vf
42. By stating the sign-convention and assumptions
used, derive the relation between object distance u, image Dividing both sides by uvf, we get
1 1 1
distance v and focal length ffor a thin convex lens, when
it forms real image of an object offinite size. / ^ u

New cartesian sign convention for spherical lenses. This proves the lens formula for a convex lens
Refer answer to Q. 35 on page 9.31. when it forms a real image.
Assumptions used in the derivation of lens formula : 43. Derive thin lens formula for a convex lens when it
(?) The lens used is thin. forms a virtual image.
(if) The aperture of the lens is small, Derivation of thin lens formula for a convex lens
(iff) The incident and refracted rays make small when it forms a virtual image. As shown in Fig. 9.85,
angles with the principal axis. when an object AB is placed between the optical centre
(iv) The object is a small object placed on the O and the focus F of a convex lens, the image A B for
principal axis. med by the convex lens is virtual, erect and magnified.
Derivation of thin lens formula for a convex lens A'
it
when it forms a real image. As shown in Fig. 9.84,
consider an object AB placed perpendicular to the
principal axis of a thin convex lens between its F' and 'vA'A
C. A real, inverted and magnified image A' B’ is I

formed beyond C on the other side of the lens. B' C F' B


O
I F c

K-H

Fig. 9.85 Virtual image formed by a convex lens.

Triangles A' B' O and ABO are similar.


A' B' _B’0 ...(1)
~AB~ BO
Fig. 9.84 Real image formed by a convex lens.
Also, triangles A' B' F and MOF are similar.
A A' B' O and A ABO are similar, A' B' B'F
A' B' _ OB' ...(1) MO ' OF
~aF~ BO
But MO = AB, therefore
Also A A’ B' F and A MOF are similar,
A' B' _ B'F ...(2)
A' B' _ FB'
AB " OF
MO ” OF
PHYSICS-XII

From (1) and (2), we get -V _-f + v


B'O _B'F B'O+OF
- u

BO ~ OF OF or
vf = uf - uv or uv = uf - vf
Using new Cartesian sign convention,
Dividing both sides by uvf, we get
BO = -w, B'0=^-v, OF=+ f 1 1 1
- V
_-v+ f f V u

- u
f
This proves the thin lens formula for a concave lens.
or
-vf = uv ~ uf
or
uv = uf - vf 9.27 LINEAR AAAGNIFICATION
Dividing both sides by uvf, we get 45. Define linear magnification produced by a lens.
Derive expressions for the magnification produced by

w
1 1 1
convex and concave lenses.
/ t. u

Linear magnification. The linear magnification produced


This proves the thin lens formula for a convex lens by a lens is defined as the ratio of the size of the image formed
when it forms a virtual image.
44. Derive the thin lens formula for a concave lens.
Flo by the lens to the size of the object. It is denoted by m. Thus
Size of image /?2

ee
m =
Derivation of thin lens formula for a concave lens. Size of object h1
As shown in Fig. 9.86, suppose O be the optical centre

Fr
and F be the principal focus of concave lens of focal Convex lens. Earlier Fig. 9.84 shows a ray diagram
length f. AB is an object placed perpendicular to its formation of image A S of a finite object AB by
a convex lens.
principal axis. A virtual, erect and diminished image
for
ur
A' B' is formed due to refraction through the lens. Now AAOB-AA'OB'

A' B' _ OB'


AB ~~OB
ks
Yo

Applying the new cartesian sign convention, we get


oo

A'B'=~h2 {Downward image height)


B

AB = + h 1
{Upward object height)
re

OB = - H
{Image distance on left)
OB'^ + v
{Image distance on right)
ou
ad

Fig. 9.86 Virtual image formed by a concave lens. -h^ _ + v or i- V

+ /i - u h u
Y

1 1
As AA'B'O-AABO
h V

A' B' _ B'O :. Magnification, m = ^~


nd

...(1) K u
Re

~AB~~^
Also, AA'BF'-A MOF Concave lens. Fig. 9.86 shows the formation of a
Fi

virtual image A' B' of a finite object AB by a concave lens.


A'B _ FB' Now A AOB~ A A'OB'
MO~Jd
A B' OB'
But MO = AB, therefore
AB OB
AB' _ FB'
AB "FO ...(2) Applying the new Cartesian sign convention, we get
AB'=+h AB = +h 1
2'
From (1) and (2), we get
OB' =-v, OB = -u
B'0_FB' FO-B'O +

BO ~ FO ~ OT + h
1
- u

Using new Cartesian sign convention, we get h V


.’. Magnification, m = —-
BO=-u. B’0 = -v, FO = -f u
RAY OPTICS AND OPTICAL INSTRUMENTS

Linear magnification in terms of u and f. The thin Example 54. A lens 80 ●


lens formula is forms a real image of an
1_1 1 object. The distance of the 60
V u / object to the lens is ucm X.
and the distance of the S, 40
Multiplying both sides by u, we get
image from the lens is v ^
-=! + - _/4-«
u U
E-l-E or = cm. The given graph shows 20
V / V / / the variation of v with u.
m= —
V f (i) What is the nature of 20 40 60 80
u f+u the lens ? (ii) Using this u (cm) -*
graph, find the focal length
Linear magnification in terms of v and /. The thin of this lens. [CBSE F 041 Fig. 9.87
lens formula is
1 1 1 Solution. (0 As the lens forms a real image, it must
be a convex lens.
V u /
(ii) From the graph, when w=20cm, we have
Multiplying both sides by v, we get V =20 cm.
V V V V f-v
1-- m = —= 1- — For the convex lens forming a real image, u is
« / u / /
negative and v and / are positive.
Hence
V_
m = —
f _f-v li = -20 cm, u = + 20 cm
U
/+M / Using thin lens formula,
1 1 1 1
— or /=+10cm.
20 -20 10
For Your Knowledge f 0 u

> The same itiin lens formula is valid for both convex and Example 55. A needle placed 45 cm from a lens forms an
convace lenses and for both real and virtual images. image on a screen placed 90 cm on the other side of the lens.
> When I m I > 1, the image is magnified. Identify the type of the lens and determine its focal length. What
>■ When I r» I < ^ the image is diminished. is the size of image if the size of the needle is 5.0 cm ? [NCERTl
Solution. Here « =-45 cm. u = + 90 cm
> When I m 1 = I the image is of the same size as the object.
> When m is positive (or v is negative), the image is virtual Using thin lens formula,
and erect.
J^^l 11 11+2 / = + 30 cm
> When m is negative (or v is positive), the image is real and / ^ u 90 45 90
inverted.
Positive value of / indicates that the lens is
Examples based on converging. Magnification,
Thin Lens Formula and Linear Magnification m = —
h
2 _
V
or
h^ _ 90
5 ~ - 45
M
[v /i^ =5 cm]
Formulae Used
Size of image, = - 10 cm
1. Focal length of any lens is given by the thin lens
formula, 1 l_l Negative sign indicates that the image is real and
inverted.
/ V u

K ^ / -/~ V Example 56. A beam of light converges at a point P.Now a


2. Magnification, m = -f- convex lens is placed in the path of the convergent beam at
u /+« /
15 cm from P. At what point does a beam converge if the
3. In Cartesian sign convention, uis taken negative convex lens has a focal length 10 cm ? [CBSE OD 19]
4. In case of convex lens, v is positive for real image
and negative for virtual image and / is positive.
5. In case of concave lens u, v and / are all negative. P
6. Magnification m is positive for virtual image and
negative for real image.
Units Used
15 cm
Distances u, v, and / are in cm or m.
Fig. 9.88 (a)
9JQ PHYSICS-XII

Solution. The incident light rays appear to Here CO = 20 cm, AO = 20 + 5 = 25 cm and


converge at point R So P acts as a virtual object for the BO = 20 - 5 = 15 cm.
lens and F is its real image as shown in Fig. 9.88(b). For the image of end A of the needle.
1 = /40 = -25cm, / = + 10cm
u

Using thin lens formula.


1 = 1 J_- ^ 1 3
V
1 / ~ + 10 -25 50
or =50/3=16.67 cm
cm
For the image of end B of the needle.
Fig. 9.88 (b) »2 = BO = -15 cm, /=+10cm

w
w = +15cm, / = +10cm 1 _ 1 ^1 _ 1 — =— or 1^2 = 30 cm
From thin lens formula, ^2 / “2 -15 30

1 = 1 1=J- 1,-1 Hence the length of the image of needle AB


V f^ u 10'^15~6
or

So the image is formed at 6 cm from the lens on the


i'= + 6cm

Flo = v^-v^=30 -16.67 = 13.33 cm.


^9. The radius of curvature of an equiconvex lens

ee
same side as the virtual object R *^1 its focal length. An object of height

Fr
Pv«rv,«i« i=7 A r.. , 5.0 an is placed at a distance of 60 cm from the optical centre
Example 57. A converge,it beam of light passes through a of the lens. Find the position a,id the height of the image
diverging lens of focal length 0.2 m and comes to focus at formed. * [iscE
distance 0.3 m behind the lens. Find the position of the point
for
ur
at which the beam would converge in the absence of the lens. Solution. Here p = 1.5, Rj = + 30 cm, = -30 cm.
[CBSE SP 98] Using Lens maker's formula.
Solution. As the focal length of a diverging lens is
ks

1
negative and the distance measured in the direction of
Yo

incident ray is positive. / R R


oo

/ = -0.2m, i; = + 0.3m ( 1 1 1 2 1
+ _ =0.5 x—=-!-
B

= (1.5-1)
As 1 2_1 L30 30/ 30 30
re

V
1"/ or
/ = + 30 cm
1=1-1=J_ Now = - 60 cm, / = + 30 cm
ou

u V f 0.3 0.2 ~ 6
ad

From thin lens formula,


Y

or u= + 0.12 m

So in the absence of the lens the beam would


1 = 1 + 1=J__1. = ± or t? = 60 cm
V f « 30 60 60
converge at a point 0.12 m from the position of the lens.
nd
Re

Magnification,
Example 58. A needle 10 cm long is placed along the axis h
2 „
V
h^ +60
Fi

of a convex lens of focal length 10 cm such that the middle m = -^


h
or
u 5 cm -60
point of the needle is at a distance of 20 cm from the lens. 1

Find the length of the image of the needle. or


/I2 = ~ 5 cm
Solution. Figure 9.89 shows a needle ABof length Hence a real and inverted image of height 5 cm
10 cm, placed on the axis of a convex lens.
is formed at a distance of 20 cm on the other side of
the lens.

10 cm A Example 60. A double convex lens has 10 cm and 15 cm as


its two radii of curvatures. The image of an object placed
A C B F O
30 cm from the lens, is formed at 20 cm from the lens on the
20 cm
other side. Find the refractive index of the material of the
lens. What will be the focal length ? What will be the focal
Fig. 9.89
length of the lens, if it is immersed in water of refractive
index 1.33 cm ? [ISCE 95]
RAY OPTICS AND OPTICAL INSTRUMENTS 9.51

Solution. Here h = - 30 cm, v = 20 cm, Rj = +10 cm, Solution. Here /=20cm, m = +4 for a virtual
K2 = -15 cm image.

Using thin lens formula, To calculate u, we have

1 1 1 1 1 5 1 m =
/ or 4 =
20
or H = -15 cm .
20 -30 60 12 u+f h + 20
/ V u

To calculate v, we have
or f = 12 cm
20-1’
Using lens maker's formula. m = or 4 = or V =-60 cm.
/ 20
1 1 1
4 = (Vo-l)
/ R
1 Example 63. A luminous object and a screen are placed on
1 1 1 an optical bench and a converging lens is placed between
or ^ = (V
12 8
-1) -L + -L =(V g -l)x-
LlO I5J
them to throw a sharp image of the object on the screen, the
linear magnification of the image is found to be 2.5. The lens
or
a
= 1 + —= 1.5 is now moved 30 cm nearer the screen and a sharp image is
12 again formed. Calculate the focal length of the lens.
When the lens is immersed in water. Solution. In Fig. 9.90, let O and / be the positions of
a object and screen, respectively. Let and be the two
1 1 1
a
-1 conjugate positions of the lens, then
f
If l-i w
R
1
OLj = l^I=x (say)
1.5
\ /
0.17 1 because the u and v values are just interchanged.
-1 i- JL
1.33 y V 10 15 1.33 6 L I ^2
1.33x6
A A
or
/.= 0.17
= 46.94 cm.

Example 61. An illuminated object and a screen are placed O I

90 cm apart. What is the focal length and nature of the lens


required to produce a clear image on the screen, twice the size
[CBSE OD 10]
V
of the object ? y = 30 cm X

Solution. As the image is real, the lens must be a


convex lens and it should be placed between the object Fig. 9.90
and the screen.

Let distance between object and convex lens =x. For the lens in position L,,
then H = OLj =-x, V = 1^1 =30 -r X
u = ~x, v=90~x
But magnification,
Now m = ~ =-2 [Minus sign as image is real] m=- = -2.5
u u

or
90-X
= -2
(minus sign taken as image is real)
~x 30 + ;c
or = -2.5, x=20cm
90 - X
or 90~x=2x or X = —= 30
3 M = - 20 cm and u = 30 + 20 = 50 cm
» = -30cm, u= + 60cm 1 1 1
As
Now
1 1 1 1 1 _ 3 ^ 1 V u
/
/ V u 60 -30 60 20
J_ -1 = 1
or / = 20 cm. 50 20 ~ /
Example 62. The image obtained with a convex lens is 50x20
or /=
erect and its length is four times the length of the object. If 50 + 20
the focal length of the lens is 20 cm, calculate the object and
= 14.3 cm.
image distances. [CBSE OD 10]
9.52 PHYSICS-XII

L M
Example 64. In Fig. 9.91(a), a convex lens L is placed at a
A
distance of 36 cm from a screen. If a point-source P is placed
at 56 cm from the screen, then a circular spot of light of \
diameter equal to the diameter of the lens is formed. Show the O' O
image formation by a ray diagram. Calculate upto what
V /
15 cm 50 cm
distance the source be displaced so that its clear image can be
formed on the screen.
Fig. 9.92

A
36 cm
Solution. For refraction through the convex lens,
/ = + 10 cm, y = +15 cm
S
p
u~ V / " 15 10 ~ 30
V
Screen

w
56 cm
or H = - 30 cm i.e., LO - 30 cm
The image formed first by reflection from the
Fig. 9.91 (a) mirror and then by refraction through the lens will be

Solution. As is clear from Fig. 9.91(b), the circular


Flo located at O' only if the image formed by reflection
from the mirror is formed at O i.e., if distance OM= R.

ee
spot AB of light will be equal to the diameter of the lens Hence LO + OM ~ 30 cm + R = 50 cm
if the image I is formed exactly in the middle of the lens

Fr
or i? = 20 cm.
and the screen.

i/ = -20cm, u = + 18cm Example 66. Calculate the distance d, so that a real image
of an object at 0,15 cm in front of a convex lens of focal
for
ur
36 cm length 10 cm be formed at the same point O. The radius of
curvature of the mirror is 20 cm. Will the image be inverted
or erect ? [CBSE SP 08]
ks

L M

A \
Yo
oo

1
B

18 cm 0

/
re

V
15 cm d
Fig. 9.91 (b)
Fig. 9.93
ou

Using thin lens formula,


ad

Solution. The final image will be formed at the


i=l ^ 1 ^ 19
Y

same point O if the concave mirror reverses the path of


/ ^ u 18 20 ^ 180 light incident on it. For this the image formed by the
To obtain a clear image on the screen, the distance lens must be located at the centre of curvature of
nd

u
Re

of the source from the lens has to be changed. In that mirror M. Then the light will fall normally on M and
case, p = + 36 cm will retrace its path after reflection.
Fi

1 _ 1 1 _ 1 19 _ 14 For refraction through the convex lens,


u V / ~ 36 180 “ 180 M = -15 cm, / = +10 cm
or u = -12.86 cm — = — + — =— H — or I? = 30 cm
V f u 10 -15 30
i.e., the source P should be at a distance of 12.86 cm from For concave mirror, R = 20 cm
the lens, it must be displaced by 20 -12.86 =7.14 cm Hence d = v + R =30 + 20 = 50 cm
towards the lens.
The final image formed at O will be an inverted
Example 65. In Fig. 9.92, the direct image formed by the image.
lens (f= 10 cm) of an object placed at O and that formed Example 67. A convex lens of focal length 20 cm and a

after reflection from the spherical mirror are formed at the concave lens offocal length 15 cm are kept 30 cm apart with
same point O'. What is the radius of curvature of the their principal axes coincident. When an object is placed
mirror ? [CBSE SP 08] 30 cm in front of the convex lens, calculate the position of the
RAY OPTICS AND OPTICAL INSTRUMENTS 9.53

final image formed by the combination. Would this result Example 69. A convex lens offocal length 10 cm is placed
changeif the object were placed 30 cm in front of the concave coaxially 5 cm away from a concave lens of focal length
lens ? Give reason. [CBSE OD 19) 10 an. If an object is placed 30 cm in front of the convex lens,
Solution. For the convex lens ; / = 20 cm, u = -30 cm M ‘he position of the final image formed by the combined
(CBSE OD 09]
system.
From thin lens formula,
Solution. For the convex lens :
i-1 1 = _L_J_ = A or y=+60cm = - 30 cm
/ = + 10 cm.
y ' / w ‘ 20 30 “ 60
From the lens formula,
The image /' formed by the convex lens serves an 1 1 1 1 1 1
object for the concave lens. or I? =+ 15 cm
v
/ u 10 30 15
This image is at 10 cm from the concave lens which
is placed at 5 cm from the convex lens. It will act as a
virtual object.
For concave lens :

« = + 10 cm, / = - 10 cm
1 1 1 1 1 .
- = —+—= + — = 0 or v-os
v / « 10 10

Hence the final image is formed at infinity. The


.●. For the concave lens : image formation is shown below.
● 30 cm 5 cm
u = +(60-30)=+30cm, / = -15cm
1 1
1 ^J_^l ^ 1
or V = ~ 30 cm
v
/ u 15 30 30

No, the result will not change from the principle of


reversibility of light rays.
Example 68. From the ray diagram shown below, calculate
the focal length of the concave lens. Fig. 9.96
/-20cm
7 7>.roblems For Practice
1. A convex lens is used to throw on a screen 10 m
o I
from the lens, a magnified image of an object. If the
magnification is to be 19, find the focal length of the
lens. [CBSE OD 10] (Ans. 0.5 m)
60 cm
10 cm 2. A small object is placed at a distance of 3.0 cm from
Fig. 9.95
a magnifier of focal length 4.0 cm. Find (i) the
position of the image formed, and {ii) the linear
Solution. For the convex lens : [CBSE OD 22]
magnification produced.
/ = + 20 cm, « = - 60 cm, v = 7 [Ans. (0 V = -12cm ; {ii) m = + 4]
From thin lens formula, 3. At what distance should an object be placed from a
1
1=1 + 1= ^ ^ 2
or u = + 30 cm convex lens of focal length 15 cm to obtain an image
V
/ 20 60 ^60 ^ 30
u
three times the size of the object ? [Punjab 041

The image 7' formed by the convex lens serves as an (Ans. 20 cm for real image, 10 cm for virtual image)
object for the concave lens. But the rays converging on 4. A convex lens is placed on an optical bench and is
the concave lens become parallel after refraction moved till it gives a real image of an object at a
through it and form image at infinity. minimum distance of 80 cm from the latter. Find the
.●. For the concave lens : focal length of the lens. If the object is placed at a
distance of 15 cm from the lens, find the position of
H = +(30-10) = +20 cm, u = co, / = ?
the image. (Ans. / = 20cm, virtual and erect
1 1 1
or / = - 20 cm. image at 60 cm from the lens on
/ ^ « 00 20 20 the same side as the object)
9.54 PHYSICS-XII

5. A convex lens of focal length 30 cm is placed meet on a screen placed at a distance of 30 cm from
between a screen and a square plate of area 4 cm^. the concave lens. Calculate the focal length of the
The image of the plate formed on the screen is concave lens. (Ans. 15 cm)
16cm^. Calculate the distance between the plate 13. In the following ray-diagram are shown the posi
and the screen. (Ans. 135 cm)
tions of the object O, image 1, two lenses and a plane
6. A planoconvex lens is made of glass of refractive mirror. The focal length of one of the lenses is also
index 1.5. The radius of curvature of the convex
given. Calculate the focal length of the other lens.
surface is 25 cm.
(Ans. 20 cm)
(i) Calculate the focal length of the lens, /-20 cm
(jr) If an object is placed 50 cm in front of the lens, V7
find the nature and position of the image
formed. [CBSE D 221 l
O /'

w
[Ans. (/) / = +50 cm (a) u = oo] ><■

7. A source of light and a screen are placed 90 cm apart. LX


Where should a convex lens of 20 cm focal length be h 45 cm ■H—16 cm -4 35 cm H
placed in order to form a real image of the source on
the screen ? (Ans. 60 cm or 30 cm from the source)
8. An object is placed at a distance of 1.5 m from a
Flo Fig. 9.98

ee
14. In the following diagram, an object 'O is placed
screen and a convex lens is interposed between
15 cm in front of convex lens of focal length 20 cm
them. The magnification produced is 4. What is the

Fr
and the final image is formed at' /' at a distance of
focal length of the lens ? (Ans. 0.24 m)
80 cm from the second lens Find the focal length
9. A screen is placed at a distance of 100 cm from an of the lens ICBSE F 16]
object. The image of the object is formed on the
for
ur
(Ans. 40 cm)
screen by a convex lens for two different locations
of the lens separated by 20 cm. Calculate the focal
length of the lens used. [CBSE OD 16]
ks

(Ans. 24 cm)
Yo
oo

10. A screen is placed 80 cm from an object. The image I

of the object on the screen is formed by a convex


B

lens at two different locations, separated by 20 cm.


Calculate the focal length of the lens used.
re

[CBSE F 08 ; D 20] (Ans. 18.75 cm)


11. When a slide is placed 15 cm behind the lens in a
ou
ad

projector, an image is formed at a distance of 3 m in HINTS


front of the lens, (i) Show the image formation by
Y

a
ray diagram, (n) Find the focal length of the lens. f = 0.5 m.
(Hi) What will be the distance between the lens and
nd

the slide in order to get an image at a distance


Re

2, u = -12cm.
4 metre from the lens ? (iu) Determine the
Fi

magnification for the case {Hi). V -12


(ii) m = - = + 4.
-3
[Ans. (ii) + 14.3 cm (in) - 14.8 cm (iv) - 27] u

12. In the diagram shown below, rays are coming from 3. (i) For real image : m = - 3, / = + 15 cm, u = 7
infinity and after passing through both the lenses m =
/ or - 3 =
15
or u = - 20 cm.
Screen I /+» 15 + u

A (ii) For virtual image : m = + 3, f - + 15 cm, w = ?


15
+ 3 = or « = -10 cm.
15 + «

4. The minimum distance between an object and its


real image formed by a convex lens is 4/.
20 cm 10 cm -4 20 cm
.'. 4 / = 80 or / = 20 cm
i=l + i=_L_ J
30 cm 1
or y = - 60 cm.
Fig. 9.97 V
/ u 20 15 60
RAY OPTICS AND OPTICAL INSTRUMENTS 9.55

16cm^ L1 ^2
5. Areal magnification = = 4

Linear magnification = ± Vi = ± 2
4cm^
A A i 1
I I

For real image. O


T
I
1 1
V
m=- = -2or v = -2u I I

1
u

1 1 1 3 V V
●H20 cm4
/ V u -lu u 2u Fig. 9.100

3/ 3x30
or u = - = -45 ^ ^ ^ _L 1-2+3^ 5 _ 1
2 2
/ V u 60 "^40” 120 120 24
y = - 2u = + 90 cm

Distance between the plate and the screen or / = 24 cm.


= I «I + [ III = 45 + 90 = 135 cm. 10. Here D = 80 cm, fr = 20 cm
6. (0 For the planoconvex lens :
80^-20^ _ 100x60 cm =18.75 cm.
p=1.5, R^=co, R2=-25cm /= 4x80 4x80
4D
1
4=(1.5-1) - ~
50
/= + 50cm. 11. (/) The ray diagram is shown in Fig. 9.101.
/ 00 -25

(rV) Here, u = -50cm, / = +50cm


1-1 1-1. L=0 X7 = CO.

V / w 50 -50
Real, inverted and highly enlarged image is formed
at infinity on the other side of the lens.
7. Let u = - X, then v = 90-x, / = + 20 cm
uv
As /= 300 cm
u-v hl5 cm-4

-a: (90-a:) 90 a: +
20 =
-x-(90-x) -90
Fig. 9.101
or - 1800 = - 90 a: +

or a:^ - 90 a: + 1800 = 0 (ii) Here u = - 15 cm, i? = + 3 m = + 300 cm


or (a:-60)(x-30)=0 1 1 1 1 1 1 .
As - +

/ 300 -15 300


or a: = 30,60 / V u

or 1/ = - 30 cm. 60 cm. 300


V V
or /=+ 21
= + 14.3 cm.

8. Here = - = - 4 or w =-
u 4 300
(jjj) Here o = + 4 m = + 400 cm, / = 21
cm
Also «| + i u| = 1.5
V Distance between lens and slide, «= ?
or - + v = 1.5
4 Using thin lens formula, we have
or v= 1.2m
21 1 1 or 1 1 21 ^ - 81
1.2 300 400 u u 400 300 1200
and u - - = - 0.3 m
4
1200
- 0.3 X 1.2 or u- - -14.8 cm.
uv
= 0.24 m. 81
/ =
u-v -0.3-1.2
V 400
= -27.
(ii?) Magnification, m = -u
9. Let and be the two conjugate positions of the -1200/81
lens.
12. Here » = + 10 cm, t? = + 30 cm
Clearly, AT + 20+X = 100 cm or .x=40cm
1 1 1 1 1 2
When the lens is in position 1^, we have 30 10 30
or / = -15 cm.
u --X - -40cm, v= 20+ A' = 20+ 40 = 60cm / V u
9.56 PHYSICS-XII

13. For convex lens. « = - 45 cm, / = + 20 cm


*■
■ + 1 1 or V = + 36 cm
" V f H 20 - 45 36

For concave lens, w = + (36 - 16) = 20 cm, i? = + oo

V
1 1
or / =-20 cm.
/ ^ U 00 20 20

Fig. 9.103 Power of a lens.


14. For convex lens Lj : t<, = - 15 cm, /^ = + 20 cm,
h
● 2=j_ 1=1. 1
Clearly, tan 5 = —
V
i a'" 20 -15 /

w
1 1
For convex lens L2: /' is virtual object and / is the real If /i = 1, then tan 5 = — or P =—

image.
/ /
Thus f/te power of a lens may also be defined as the
1^2 =-(60+ 20) = 80 cm, t;2 = + 80 cm
80
Flo
reciprocal of its focal length.
SI unit of power. The SI unit of power is dioptre,

ee
h^l=|z^2l = 2/2 =i> /2=y = 40 cm. denoted by D. If / =1 m, then

Fr
1
P = = lm ^ = 1 dioptre (D)
9.28 POWER OF A LENS Im

46. What is meant by power of lens ? Give and define One dioptre is the power of a lens whose principal focal
for
ur
its SI unit. Which type of lens has a positive power and ^ metre,
which one negative ? Express power of a lens in terms of The focal length of a converging lens is positive and
its refractive index and radii of curvature. that of a diverging lens is negative. Thus, the power of a
ks

Power of a lens. The power of a lens is a measure of (Converging lejis is positive and that of a diverging lens is
Yo
oo

the degree of convergence or divergence of the light negative. We can measure the power of a lens directly
rays falling on it. As shown in Fig. 9.102, a convex lens by a device called dioptremeter. Thus, when an optician
B

of shorter focal length bends light rays towards the prescribes a corrective lens of power + 2.5 D, the requi
red lens is a convex lens of focal length, f = 1 / (+ 2.5 D)
re

principal axis through a larger angle, by focussing


= +0.40 m = + 40 cm. Similarly, a power of -4.0 D
them closer to the optical centre. Hence smaller the focal means a concave lens of focal length -25 cm.
length of a lens, more is ability to bend light rays and greater
ou
ad

is its power. By using lens maker's formula, the power of a lens


can be expressed in terms of its refractive index p and
Y

radii of curvature R., and R2 as follows :


1
nd

P = -=(p-l) J l_
Re

●»>
/
Fi

As the power of a lens is reciprocal of its focal


length, so it characterises the focal properties of the
lens, such as nature, size and position of image, etc.
Fig. 9.102 (a) Large /, small bending power, 9.29 COMBINATION OF THIN LENSES
(ft) Small/, large bending power.
47. Why do we use lens combinations in optical instru
ments ? Write an expression for the total magnification
The power of a lens is defined as the tangent of the produced by combination of lenses.
angle by which it converges or diverges a beam of
Lens combinations. In many optical instruments,
light falling at unit distance from the optical centre. two or more lenses are used either in contact or with a

In Fig. 9.103, a beam of light is incident at distance/i S^P between them. The purpose of using a lens
combination is
from the optical centre O of a convex lens of focal
length /. It converges the beam by angle & (/) To magnify an image.
RAY OPTICS AND OPTICAL INSTRUMENTS 9.51

(n) To increase the sharpness of the final image by For the combination of thin lenses in contact, if / is
minimising certain defects or aberrations in it. the equivalent focal length, then
(Hi) To erect the final image. ...(4)
(iv) To increase the field of view.
V u /

Different lens combinations are used in the


From equations (3) and (4), we find that
objectives of cameras, microscopes, telescopes and 1-X J_
other optical instruments. fi
Total magnification. When lenses are used in Equivalent power,
combination, each lens magnifies the image formed by
the preceding lens. Hence the total magnification m is P=P,4-P,
equal to the product of the magnifications m^, For n thin lenses in contact, we have
and produced by the individual lenses. 1 1
— = — + — + -—+
1 1
+—
1

m=m^.y mjX / /l fl /s In
48. What is an equivalent lens ? Obtain an expression .-.Equivalent power,
for the effective focal length of two thin lenses placed in + + P„
contact coaxially with each other.
49. Derive an expression for the focal length of the
Equivalent lens. A single lens which forms the image of combination of two thin lenses offocal lengths and f^,
an object at the same position as is formed by a combination when they are separated by distance d.
of lenses is called an equivalent lens.
Thin lenses separated by a small distance. As
Equivalent focal length and power of two thin shown in Fig. 9.105, consider two thin lenses and
lenses in contact. As shown in Fig. 9.104, let and
of focal lengths and /j, respectively. The two lenses
be two thin lenses of focal length f and /2 respec are placed coaxially, distance 'd' apart.
tively, placed coaxially in contact with one another. Let L t-2
O be a point object on the principal axis of the lens D
system. O

h1
h-
C F f
c. 9

V
d
r

Fig. 9.105 Two thin lenses separated by a small distance.


Suppose a ray OA traversing parallel to the
principal axis is incident on lens Lj. It is refracted along
AF, F being the second principal focus of Lj. The
Fig. 9.104 Two thin lenses in contact. deviation produced by is
Let OCj = M. In the absence of second lens L^, the 6j - tan Sj = —
first lens will form a real image /' of O at distance A
C /' = v'. Using thin lens formula, ^ ^ c_ .. j u j
1 ® The emergent ray is further refracted by second
I (1) lens £2 slong HF'. Since the incident ray OA is parallel
A v' u to the principal axis, F’ should be second principal
, ,. , , ^, , focus of the combination. The deviation produced by
The image /' acts as a virtual object (a = ti ) for he
second lens which finally forms its real image 1 at ^
K
distance v. Thus 62 - tan 62 =
fl
1 1_J_ ...(2)
fl V v' The final emergent ray BF', when produced back
wards, meets the incident ray at point D. Obviously, 6
Adding equations (1) and (2), we get is the final deviation produced. A single thin lens
1 . 1 1_1 placed at C will produce the same deviation as by
the combination of two lenses. Thus distance CF' is the
/i fl V u
9.58 PHYSICS-XII

second focal length of the combination. If / is the focal Example 70. A double convex lens of+5 Dis made ofglass
length of the combination, then of refractive index 1.5 with both faces of equal radii of
g_^ curvature. Find the value of curvature. \
[CBSE F 15]
/ Solution. Let Rj = + R and =-R. Then
It is obvious from Fig. 9.101, that 1 1
P = -
5= + 62 / Ri R2
Ih-lh + lh.
5 = (1.5-1)!^r'^ R
1 2
or = 0.5x —
/ fi A R

As AACj F ~ ABC2F, therefore, we have R =


0.5x2 1
m = — m = 20 cm.
5 5
AC^ BCj or
h
1 _ A
Example 71. Calculate the radius of curvature of an equi-

w
\ /, /.-rf
concave lens of reactive index 1.5, when it is kept in a medium
or
h,=^.h, of refractive index 1.4, to have a poioer of-5D ? [CBSE D 19]
Solution. Here Rj =-R, p2 = + R, P = -5D,
Hence
/ /i
I/i-^ .h
/1/2
1

Flo g,=1.4, P2 = 1-5

ee
/ = — = — m=-20cm
J d_ ■' P -5

Fr
or

f A A AA
i= ^-1 1_
In terms of powers of the lenses, / Pi Rj R2
for
ur
P = Pj + ?2 - fr. Pj. P2
-^=[1:5-1
-20 11.4
J
-R RJ~ 1.4
X 2 ■

R
Examples based on
ks

20
(i) Power of Lenses or R = —cm = 2.86 cm.
Yo

7
(ii) Combination of Lenses
oo

Formulae Used Example 72. A convex lens is made of glass of refractive


B

1 100 index 1.5. If the radius of curvature of the each of the two
1. Power of a lens, P = surfaces is 20 cm, find the ratio of the powers of the lens,
re

/(m) /(cm)
when placed in air to its power, when immersed in a liquid of
1 / -r^ 1 1
2. P=- = (g -1) — refractive index 1.25.
ou

/ R, R,
ad

Solution. Here R.j =20 cm =0.2 m,


3. For a combination of lenses, m=: x m^ x...
Y

R2=-20cm=-0.2m, g^=1.5, Pj=1.25


4. For two lenses in contact, equivalent focal length F For the lens placed in air :
is given by
nd

1
= (1.5-1) j i_
Re

1 1 1 Pi=iP -1) —- = 5D
— =—+ — or Power, P= S
A 0.2 - 0.2
P A A
Fi

1 For the lens placed in liquid :


For n lenses in contact, — = — +— + ...+ — 'y ■
/ A A fn 1 1_ 1.5
J ^
P,= -1 - 1 = 2D
Power P=PI + R + ... + P.
n R^ R2J U-25 0.2 - 0.2

5. The equivalent focal length f of tv^o lenses sepa


rated by distance d is given by A = 5^5:2.
P2 2
2_J_ J_ ^
f A A A A Example 73. (/) If f = + 0.5 m, what is the power of the
lens ? (ii) The radii of curvature of the faces of a double
or Power, P=^ + /^-d.f^.]’
convex lens are 10 cm and 15 an. Its focal length is 12 cm.
Units Used
What is the refractive index of glass ? (Hi) A convex lens has
Focal lengths / /, /2, etc. and radii of curvature 20 cm focal length in air. What is its focal length in water ?
Rj and Rj are in metre or cm and powers P, ^ {Refractive index of air-water =1.33, refractive index for
etc. are in dioptre (D). air-glass =1.5.) [NCERT]
RAY OPTICS AND OPTICAL INSTRUMENTS

Solution, (i) Here, / = + 0.5 m As i = J_ —


P = 1=-7_=+2D.
/ +0.5
- 50 50 25
(ii) Here, Rj = + 10 cm, R2 = -15 cm, / = + 12 cm
or - 25 cm = - 0.25 m
1 1
As
/
8 R The second lens is a diverging lens. Its power is
1
1

1 1
^.4fi -0.25 m
= -4D.

^ -I = (ti -l)x-
12 ^ 10 15
Example 76. Tzyo /e«ses of powers + 15 D and - 5 Dare
or p„-l = 0.5 in contact with each other forming a combination lens.
8
or = 1.5. (a) What is the focal length of this combination ?
8
(b) An object of size 3 cm is placed at 30 cm from this
(Hi) Here, = + 20 cm, p w
= 1.33, p^=1.5, 4 = ? combination of lenses. Calculate the position and size
of the image formed. ICBSE D 02]
For the lens placed in air :
1 1 1
Solution, (a) Power of combination,
4 = (Pc-l)
8 P=Pj + P2= + 15-5=10D
fa R
1
Focal length of combination,
1 1 1
or rr = a-5-l) . 1 1
20 R
1 ^2 /=^P 10
m =10 cm.

1 1
or J 1 (i?) Given =3 cm, u = -30 cm, / = 10 cm
Rj R2 20 X 0.5 10 1 1 1
Using thin lens formula, —
For tlte lens placed in water : V u /
1 1 1
1 J^8 -1
1 1 f 1.5
-llx — or

u / u 10 30 15
1.33 10
fw ^zv R
1
or u = +15 cm
10x1.33
or
u= 0.17
=+ 78.2 cm.
As m= —
h
2 _
V

/ij u
Example 74. Find the two possible positions of an object V 15
kept in front of a lens o/+5D D, so that the image formed in /I2 = -u X llj = -30 X 3 = -1.5 cm

both cases is four times magnified. [CBSE OD 22]

Solution. P =+ 5.0 D => /=+— m = + 20 cm Real inverted image of size 1.5 cm is formed at 15 cm
3 on the other side of the combination.

(i) For real hiiage : m = -A Example 77. A converging lens of refractive index 1.5 has
m =
/ => -4 =
20
=> w = - 25 cm. a
power of 10 D. When it is completely immersed in a liquid,
/+4 20+ u it behaves as a diverging lens offocal length 50 cm. Find the
refractive index of the liquid. (CBSE OD 20]
(ii) For virtual image : w = + 4
20 Solution. Here P = + 10D
+4 = => M = -15 cm.
20+ M 1
= — m =10 cm
P 10
Example 75. A converging lens of focal length 50 cm is
placed coaxially in contact with another lens of unknown 1 J ^
focal length. If the combination behaves like a diverging lens
But
fa
= (V,-i) R,~R^
offocal length 50 cm, find the power and nature of the second
1
lens. (CBSE OD 04C] or = (1.5-1)
Solution. Here
10 ' R, R2_
1 1 1 1
fi= + 50 cm (converging lens) or

/ = -50 cm (diverging combination) R2 10x0.5 5


.60 PHYSICS-Xil

When the lens is immersed in the liquid, Solution. (/) When both the lenses are convex :
f
1 1 1 /j = ^ = + 25 cm
-1
/,
a

^1 R1
Focal length / of the combination is given by

or
1 1.5
a
-1 X -
1 / ///2 25 ■"25 “25
-50 5
F/ or
/ = 12.5 cm
or
1.5
-1 = -
1 Now « = -15 cm, / = 12.5 cm
V/ 10
1=1 + 1= J__±=±
1.5
V f u 12.5 15 75
or = 1-0.1 =0.9
a or = + 75 cm

w
a 1.5 Thus the object is at 15 cm on one side of the
or
0.9
= 1.67.
combination and the image is at 75 on the other side of
the combination.
Example 78. A real image is formed by the lens at
distance of 20 cm from the lens. The image shifts towards the
combination by 10 cm ivhen a second lefts is brought in contact
Flo
a
Distance between the object and image
= 15 + 75 = 90 cm.

ee
with the first lens. Determine the power of the second lens. (//) When both the lenses are concave :
/i = /2=-25 cm.

Fr
Solution. As the image is real, so the lens is convex.
Let its focal length be fy It forms image on the other 1 = 1- + J_=_J__±-_1_
side, so u = + 20 cm.
/ /i /2 ” 25 25 “ ^
for
I l l
ur
Using lens formula, = _, we have or
/ = -12.5 cm
V u f
1 1 1
Now M = -15 cm, / = -12.5 cm
...(0 1 = 1 + 1= I L-_ll
ks

20 « /, " V f 12.5 15 75
Yo

M
oo

When the second lens is placed in contact with the 75


or v = - = - 6.8 cm
first one, image shifts 10 cm closer to the combination, 11
B

so the second lens is also convex. If f^ is the focal length Thus the image is at 6.8 cm from the combination
of second lens and/that of the combination, then
re

on the same side as the object.


1 = J- +J_
f /, f2 Distance between the object and the image
ou
ad

= 15 -6.8 = 8.2 cm.


For the combination, v =20 -10 =+ 10 cm, so from
Example 80. An object is situated at 20 cm on the left of a
Y

lens formula.
1 1 ]__J_ J_ convex lens of focal length 10 cm. Another convex lens of
10
...(ii) focal length 12.5 cm is placed at a distance of 30 cm on the
nd

u
Re

right of the first lens. Find the position and magnifica


Subtracting (i) from (ii), we get tion of the final image. State also the nature of the image.
Fi

Solution. As shown in Fig. 9.106, the lens L, forms


fj 10 20 20 image A' B of object AB. The image A' B lies within
focal length of lens and acts as an object for it. A” B
tr

^ =20 cm =0.2 m
Power of second lens, is the final image formed by l^.
For the lens : i<=-20cm, / = +10cm
P = -l — = 0.5 D.
From thin lens formula,
/2 0-2
A = 1 A= ^ ^ 1

Example 79. A small object is placed at a distance of 15 cm u u 10 20 ^20


from two coaxial thin lenses in contact. The focal length V = + 20 cm
of each lens is 25 cm. What will be the distance between the
object and its image when both the lenses are (i) convex, (ii)
i.e., l^A' = 20 cm
concave.
A' 1^=^ L^L^ - L^A' =30 -20 =10 cm
RAY OPTICS AND OPTICAL INSTRUMENTS 9.61

Though the image is real it serves as a virtual object for


cm

the second lens, which means that the rays appear to


come from it. For the second lens,

^2
1 1 _ 1 or = CO

^2 10“-10 2

The virtual image is formed at an infinite distance


20 cm *{^ 10 cm to the right of the second lens. This acts as an object for
the third lens.

^3 % /s
1 1 1
Fig. 9.106 or cm or
zjj = 30 cm
30
^3
00
For the lens :

M = -10 cm, / = +12.5 cm. The final image is formed 30 cm to the right of the
third lens.
1^J_ 1 1

u 12.5 10 50 Example S2. An equiconvex lens with radii of curvature of


magnitude R each, is put over a liquid layer poured on top
u = 50 cm
or
ofa plane mirror. A synall needle, zvith its tip on the principal
The final image A' B' is formed at 50 cm from I2 axis of the lens, is moved along the axis until its inverted real
its left. It lies at the position of object AB and is virtual image coincides zoith the needle itself The distance of the
and inverted with respect to the object. needle from the lens is measured to be a. On removing
A'B' V 20 the liquid layer and repeating the experiment the distance is
= -l
m^ =
AB u -20
found to be 'b'.
Given that two values of
A'B' distances measured represent
A'B u -10 the focal length values in the Liquid

two cases, obtain a formula i


Total magnification.
for the refractive index of the
m = m^x jn2=-lx 5=-5. liquid. [CBSE SP 081 Fig. 9.108
Example 81. Find the position of the image formed by the Solution. Clearly, equivalent focal length of
lens combination given in Fig. 9.107.
[NCERT ; CBSE D 19 ; JEE Main 21]
equiconvex lens and water lens, f = a
/=+10,-10, + 30 cm Focal length of equiconvex lens, f^=b
Focal length .f^ of water lens is given by
A AV7 A
Vrh
o
a b ab
30 cm ab
or
A"
v/A V b-a

The water lens formed between the plane mirror


and the equiconvex lens is a planoconcave lens.
Fig. 9.107
For this lens, Kj = - R and R2=^
Solution. For the image formed by the first lens,
1 1 1 Using lens maker's formula,

^1 u
1 /i — = (p -1) —- —
A R
1
1 1 _ 1 or =15 cm
-30 "10 b-a
or = (f-i) — —
ab -R 00
The image formed by the first lens serves as the
ia-b)R
object for the second. This is at a distance of
(15-5) cm =10 cm to the right of the second lens.
or ,-l = (^
ab
or p=l +
ab
s 9.62

roblems For Practice

1. A thin converging lens of focal length 12 cm is kept


PHYSICS-XII

Given that the two values of the distance measured


represent the focal length values in the two cases,
calculate the refractive index of the liquid.
in contact with a thin diverging lens of focal length [CBSE D 08C] (Ans. 1.33)
18 cm. Calculate the effective/equivalent focal HINTS
length of the combination. [ISCE 22]
1. l=-i J_-J_ 1 _ 1
(Ans. 36 cm) / ^ /2 ~12'^-18‘36
2. A convex lens of focal length 20 cm is placed / = 36 cm.
coaxially in contact with a concave lens of focal
length 25 cm. Determine the power of the 2. Power of convex lens.
20
combination. Will the system be converging or
diverging in nature ? [CBSE D 13] 100
Power of concave lens, i| = -25 ^-4D

w
(Ans. +1D, converging)
3. The radius of curvature of each surface of a convex
Power of the combination, P=/^ + i^=+lD
lens of refractive index 1.5 is 40 cm. Calculate its
As the power Pis positive, the combination will be
power.

4. Two lenses of powers 10 D and - 5 D are placed in


contact,
Flo
[Ans. 2.5 D]
convergingin nature.
3. Here p= 1.5, P, =+40 cm = 0.40 m,

ee
(i) Calculate the power of the new lens. - 40 cm - 0.40 m

Fr
(//) Where should an object be held from the
lens, so as to obtain a virtual image of J \_
magnification 2 ? [CBSE OD 08] / A
for
ur
[Ans. (0 5 D («) - 10 cm]
1
5. Find the focal length and power of a convex lens, = (1-5-1) = 2.5 D.
0.40 0.40
which when placed in contact with a concave
ks

lens of focal length 25 cm, forms a real image 5 times 4. (0 P= ^ + ?2 = 10-5 = 5D.
Yo

the size of the object placed 20 cm from the


oo

(ii) / = -^ = ^m =20cm
combination. (Ans. 10 cm, 10 D)
6. The power of a thin convex lens of glass is 5 D.
B

Wlien it is immersed in a liquid of refractive index As m- /


re

|i, it behaves like a divergent lens of focal length 1 m. f^u


Calculate jj of liquid, if p of glass = 3/2 (Ans. 5/3) 20
2 =
ou

It = -10 cm.
7. A compound lens is made of two lenses having powers or
ad

20+ u
+ 15.5 D and - 5.5 D. An object of 3 cm height is
Y

placed at a distance of 30 cm from this compound 5. Here m = ~5, U--20cm


lens. Find the size of the image. (Ans. - 1.5cm) As m =
/
8. Rays coming from an object situated at infinity, fall f+u
nd
Re

on a convex lens and an image is formed at a f 50


-5 =
^ 3
or cm
distance of 16 cm from the lens. When a concave
Fi

f~2Q
lens is kept in contact with the convex lens, the
1 1 1
image is formed at a distance of 20 cm from the lens Now
combination. Calculate the focal length of the fz f A
concave lens. (Ans. - 80 cm)
3 1 ^3+2_ 1
9. An equiconvex lens, witln radii of curvature of 50 -25 ~ 50 "lO
magnitude 20 cm eacli, is put over a liquid layer 100
poured on top of a plane mirror. A small needle, or
/2 = +10 cm and P = = 10 D.
10
with its tip on the principal axis of the lens, is
moved along the axis until its inverted real image 6. J 1_
coincides with the needle itself. The distance of the
R,"R2
needle, from the lens, is measured to be 30 cm.
3
On removing the liquid layer, and repeating the or + 5= -:1 J__ J_ ...(«)
experiment, the distance is measured to be 20 cm. u A
RAY OPTICS AND OPTICAL INSTRUMENTS 9.63

1 g
-1 I 1 Solution. Figure 9.109(a) shows a convex lens L
Ei =
fi Pi placed in contact with plane mirror M. P is the point
object, kept in front of this combination at a distance of
or
1 [j l' ...Hi)
20 cm, from it. As the image coincides with the object
-1 R
1 itself, the rays from the object, after refraction from the
lens, should fall nomnlly on the mirror M, so that they
Divide (i) by (i7) to get the value of p retrace their path. For this, the rays from P, after refrac
7. Equivalent power, P = + 15.5 - 5.5 = 10 D tion from the lens must form a parallel beam perpendicular
. . , , r 100 100 to M. For clarity, M has been shown at a small distance
= 10 cm
Equivalent focal length, ^ 10 from L, in Fig. 9.109(17). As the rays from P, form a
f 10 1 parallel beam after refraction, P must be at the focus of
Now in =
the lens. Hence focal length of the lens is 20 cm.
/+ 10-30 2

But m - —

Ih
K I , T
-^ = — or /i, = -1.5 cm.
3 2 ^
8. Focal length of convex lens, /| = + 16 cm
Combined focal length, / = + 20 cm

/ ./ /2
1
A= = J__ J-
■■ f, f f, 20 16^ 80

or fi-- 80 cm.
9. Proceed as in Example 82.

9.30 COMBINATION OF A LENS AND


A MIRROR

Image formed by a combination of a lens and a Example SA. A convex lens is placed over a plane mirror.
mirror. Consider an object placed in front of a lens pi,j jg positioned so that there is no parallax betiveen
coaxially arranged with a mirror. The rays from the Hs image formed by this lens-mirror
object first suffer refraction through the lens and then combination. How can this observation be used to ifnd the
get reflected by the mirror. We can study the image length of the convex lens ? Give appropriate reasons in
formation by such combinations through some support of your answer. [CBSE SP 13]
numerical examples given ahead. ^ Solution. The rays must
fall normally on the plane
Examples based on mirror so that the image of
Image Formation by a combination the pin coincides with itself.
of a lens and a mirror
Hence rays, like CA and
Concept Used DB, form a parallel beam
incident on the mirror.
We first find the position of the image formed by
the lens by using thin lens formula. Taking this .●. P is the position of the
focus of the lens.
image as real (or virtual) object for the mirror, we
use mirror formula to locate the position of the .●. Distance OP equals the
final image formed by the combination. focal length of the lens.
Example 85. The distance between a convex lens and a
Example 83. A convex lens is placed in contact with a plane mirror is 15 cm. The parallel rays incident on the
plane mirror. An axial point object, at a distance of 20 cmconvex lens, after reflection from the mirror, form image at
from this combination, has Us image coinciding with itself the optical centre of the lens. Draw the ray-diagram and find
What is the focal length of the convex lens? [CBSE D 14] out the focal length of the lens.
9.64 PHYSICS-XII

Solution. The convex lens converges the parallel The final virtual image is formed at a distance,
rays towards its focus T. Tlie plane mirror reflects and 60-40 =20 cm behind the mirror,
converges these rays at the optical centre O. So O is the
image of f formed by the plane mirror.
^3
A
\ ,
V
>
M F h20 cm-H^20 cm-*l<-20 cm-*l

Fig. 9.112 {b}


V
15 cm 15 cm Example 87. The image of an object, formed by a
combination of a convex lens (offocal length f) and a convex
mirror (of radius of curvature R), set up, as shown is

w
Fig. 9.111
observed to coincide with the object.
Focal length of convex lens
Redraw this diagram to mark on it the position of the
= OM + Mf = 15 + 15 = 30 cm.
centre of curvature of the mirror.
Example 86. ApointobjectisplacedatadistanceofSOcm
from a convex lens of focal length 30 cm. If a plane mirror Flo /

ee
were put perpendicular to the principal axis of the lens and Image
at a distance of 40 cm from it, what would be the location and

Fr
nature of the final image formed by this combination.
Object 6
INEET 21]
for
ur
A
a d

Fig. 9.113
ks

V Obtain the expression for R in terms of the distances,


Yo

marked as aand d,and thefocal length, f, of the convex lens.


oo

60 cm +■— 40 cm —H
[CBSED16C]
B

Fig. 9.112 Solution. The centre of the curvature C is shown in


the redrawn diagram [Fig. 9.113(fl)].
re

Solution. For first refraction from the lens :


u = -60 cm, / = +30 cm, v.^ =7
/
ou
ad

_ uf _ -60 X 30
I

V = 60 cm
Image
Y

^~ii+f “^0 + 30 Object


I
I

The lens forms image f at 20 cm behind the plane


I
/
\
nd
Re

mirror which acts as virtual object the mirror. The a d R


mirror forms real image at 20 cm in front of it.
Fi

Fig. 9.113 (a)

For convex lens : u = -a, v- R + d

f R+d (-a)
■20 cm-W
1 1
+ -

Fig. 9.112 (a) R +d a

For seco7id refraction from the lens : or


1 1 \_a-f
u = -20 cm R+d f a
«/
V =
uf _-20x30 = -60 cm
Hence, R = -a.
u+ f -20 + 30 a-f
RAY OPTICS AND OPTICAL INSTRUMENTS

Example 88. A point object is placed at a distance of 12 cm


on the principal axis of a convex lens offocal length 10 cm. A
convex mirror is placed coaxially on the other side of the lens
at a distance of 10 cm. If the final image coincides with the
object, sketch the ray diagram and find the focal length of the
convex mirror. [CBSE OD 20]

Solution. Here the image of the combination


coincides with the object itself. This implies that / is the
centre of curvature of the convex mirror.
f u 10 -12
Focal length of the mirror. y = 60 cm
Ml
fm 2 LI-LM=60-10 50 cm

50
= 25 cm.
For the convex lens : w=-12cm, /=+10cm 2

Example 89. A convex lens, offocal length 20 cm, is placed co-axially with a convex mirror of radius of curvature 20 cm. The
two are kept 15 cm apart from each other. A point object is placed 60 cm in front of the convex lens. Draw a ray diagram to show
the formation of the image by the combination. Determine the nature and position of the image formed. ICBSE OD 14]

Solution. The ray diagram, for the image formed by the combination, is shown below in Fig. 9.115.

L
M
/t,
I
I
/
I
P
I

60 cm 30 cm H
Fig. 9.115

For the convex lens ● u.^=- 60 cm and / = +20 cm For the convex mirror : »2 =+15 cm and R =+20cm.
Using thin lens formula, we get Using mirror formula, we get
. 1 1 2
1 1 _ 1 or
1
i.e., — + — = —
V
I
-60 20 20 60 ~ 30 ^2 "2 R v^ 15 20
or = 30 cm
or

In the absence of the mirror, the lens would have "^^^”10 15 ”30
formed the image of P at f, which acts as a virtual object V2 = + 30 cm
for the convex mirror.

.-. O’/j = distance of virtual object from convex Hence the final image / is a virtual image formed at
a distance of 30 cm behind the convex mirror.
mirror = 0/j -OCy =(30-15)cm = 15 cm.

Example 90 A convex lens, offocal length 20 cm and a con lens. This image, now acts as a real object for the
cave mirror, of focal length 10 cm, are placed co-axially 50 cm concave mirror.
apart from each other. A beam of light coming parallel to the For the concave mirror: » =-30 cm and /=-10cm
principal axis is incident on the convex lens. Find the position Using mirror formula, we get
of the final image formed by this combination. Draw the ray 1+ 1 ^ 1
diagram showing the formation of the image. — or
V -30 ” -10 i;"30 10 ” 15
[CBSE OD 14]
or V = -15 cm
Solution. The incident beam, on lens L, is parallel
to its principal axis. Hence the lens forms an image f at The lens-mirror combination, tlierefore, forms a
its focal point i.e., at a distance O/j(=20 cm) from the real image / at a distance of 15 cm from M.
PHY5ICS-XII

The ray diagram is as shown in Fig. 9.116. 4. Figure 9.118 shows a plane mirror M placed at a
distance of 10 cm from a concave lens L. A point

'A
object is placed at a dis L M
tance of 60 cm from the
lens. The image formed,
\T7
ro due to refraction by the
lens and reflection by
the mirror, is 30 cm
behind the mirror.
ZJA
h—20 cm What is the focal length 10 cm
of this lens ?
Fig. 9.116 Fig. 9.118
(Ans. - 30 cm)
Example 9^.A concave lens offocal length 10 cm is placed

w
on the axis of a concave mirror of 10 cm radius at a distance HINT
of 5 cm from the mirror. An object is placed so that the light
coming from it first passes through the lens, then gets 2. For the object at infinity, the convex lens forms
reflected from the mirror, again passes through the lens to image at its focus F i.e., at 25 cm from the lens. This
form an inverted image coincident luith the object itself
Determine the position of the object.
Flo image then acts as an object for the convex mirror.
/= 25 cm

ee
i? = 20cm

Solution. The rays starting from the object O after

Fr
refraction from the lens and reflection from the mirror
retrace their path and form inverted image at O itself.
Thus the rays fall normally on the concave mirror and I
for
should be its centre of curvature and is the virtual
ur
image of O.
For the concave lens :
40 cm h-20 cm-H Fig. 9.119

V
ks

For the convex mirror:


/ = -10cm, i;=:-5cm, w = ?
Yo

» =-(40-25) =-15 cm, R=+20cm


oo

1=1 ^ O
■—■*— J' P
u V
/ Using mirror formula, 14-1 2-1
B

u
f~ R V

1 2_=_±
1 = 1_1=1. L-J_ 2.-^
re

5 10 10 or

}●— 5 cm V R u 20 -15“ lo'*'l5”6


or w = -10 cm.
10 cm v = +6cm
ou
ad

Fig. 9.117 Hence, the combination forms a virtual image at a


roblems For Practice distance of 6 cm behind the convex lens.
Y

1. A point object is placed 60 cm in front of a convex 3. For the convex lensw = -15cm, / = 4-10 cm
lens of focal length 30 cm. A plane mirror is placed Using thin lens formula,
nd
Re

10 cm behind the convex lens. Where is the image 1_1 1 or 1 l = _l


formed by this system ?
Fi

V u
/ V -15 4*10
(Ans. At the optical centre of the convex lens) i = 2__2.-2_
or => 4- 30 cm
2. A convex lens, of focal length 25 cm, and a convex V 10 15 “ 30
mirror, of radius of curvature 20 cm, are placed V 4-30
m-- = -2
co-axially 40 cm apart from each other. An incident u -15
beam, parallel to the principal axis, is incident on
the convex lens. Find the position and nature of the A real, inverted and magnified image is formed on

the other side of the lens at a distance of 30


image formed by this combination. [CBSE OD 16C]
cm.

3. An object is placed 15 cm in front of a convex lens of The final image formed by the concave mirror will
focal length 10 cm. Find the nature and position of be at the position of the object itself only if the
the image formed. Where should a concave mirror image formed by the lens lies at the position of the
centre of curvature of the mirror.
of radius of curvature 20 cm be placed so that the
.'. Distance of the mirror from the lerrs
final image is formed at the position of the object
itself ? (CBSE OD 15] (Ans. 50 cm from the lens) = (304- R)ccn ={304-20) cm =50 cm.
RAY OPTICS AND OPTICAL INSTRUMENTS 9.67

9.31 REFRACTION THROUGH A PRISM the face AB, and f and i' be angles of incidence and
emergence at the face AC. Let A be the angle of the prism.
50. What is a prism ? What do you mean by refracting
faces, refracting edge, base, angle of prism and principal From the quadrilateral AQNR, A + Z QNR = 180°
section of a prism. From the triangle QNR,
r+Z + Z QNR =180° A = r+f
Prism. A prism is a wedge shaped portion of a
transparent refracting medium bounded by two plane faces Now, from the triangle MQR, the deviation produced
inclined to each other at a certain angle. by the prism is
The two plane faces {ABED and ACFD) inclined to each 8= Z MQR + Z MRQ = {i-r) + {i'-r')
other are called refracting faces of the prism. or 5= deviation at the first face
The line (AD) along which the tzoo refracting faces meet + deviation at the second face
is called the refracting edge of the prism. = {i+ i')-{r+ r')
D
Refracting
or 8=i+ i' - A or i + A +8
edge N
Refracting-^ Principal->
faces \ A i.e.. Angle of incidence + Angle of emergence
section \ a
= Angle of prism + Angle of deviation
Angle ^
of prism / £, F
So when a ray of light is refracted through a prism, the sum
of the angle of incidence and the angle of emergence is equal to
the sum of the angle of the prism and the angle of deviation.
B C B C
Factors on which the angle of deviation depends :
Fig. 9.120 (a) A glass prism, (b) Principal section of a prism. (i) The angle of incidence,
The third free (BCFE) of the prism opposite to the (ii) The material of the prism.
refracting edge is called the base of the prism. (Hi) The wavelength of light used
The angle A included between the two refracting faces is (lb) The angle of the prism.
called angle of the prism. 52. Discuss the variation of the angle of deviation
Any section of the prism cut by a plane perpendicular to xvith that of the angle of incidence for a ray of light
the refracting edge is called principal section of the prism, passing through a prism. Derive an expression for the
51. Discuss the phenomenon of refraction through a refractive index of the material of a prism in terms of the
prism. Derive an expression for the angle of deviation for angle of prism and the angle of minimum deviation.
a ray of light passing through an equilateral prism of Variation of angle of deviation with angle of
refracting angle A. incidence. Fig. 9.122(o) shows the path of a ray of light
Refraction through a prism : Deviation produced suffering refraction through a prism of refracting
by a prism. In Fig. 9.121, let ABC represent the angle ‘A. Fig. 9.122(&) shows
A
the variation of angle of
principal section of prism. A ray PQ is incident on face
AB. As it enters the denser medium (glass), it bends
towards the normal along path QR. The ray QR suffers Normal
Normal
" at R
another refraction at face AC ; bending away from
the normal, it emerges along RS. The angle of deviation
8 is the angle between the incident ray and the emergent ray.
Let i and r be the angles of incidence and refraction at
A (a)
Refracting edge—»
Angle of
■^prism
, >

Normal Refracting
Normal
atO
o o

(b)

i'

Fig. 9.122 (a) A ray of light passing through a prism. (6) Plot of varia
Fig. 9.121 Refraction through a prism. tion of angle of deviation S versus angle of incidence i for a prism.
9.68 PHYSICS-XII

deviation 6 with the angle of incidence i. For a given Hence deviation produced by the prism is
prism and for a given colour of light, the angle 6 5 = f + i' - A = p r + p r' - A
depends on i only. As i increases, the angle of 6 first
decreases and reaches a minimum value 6^ and then = p(r+r')-A=p A-A
increases. Clearly, any given value of 6 corresponds to [v r+r' = A]
two angles of incidence / and This fact is expected or
5=(p-l)A
from the symmetry of i and i' in the equation :
5 = / + f - A i.e., 5 remains the same as i and i' are Clearly, the deviation produced by a small angled
interchanged. Physically, it means that the path of the ray prism does not depend on the angle of incidence. It
in Fig. 9.122(a) can be traced back, resulting in the same depends on the angle of the prism and the refractive
angle of deviation. index of its material.

The minimum value of the angle of deviation suffered by For Your Knowledge
ray on passing through a prism is called the angle of

w
a

minimum deviation and is denoted by 6 m'


> A ray of light suffering refraction through a prism is
bent towards the base of the prism.
Relation between refractive index and angle of
minimum deviation. When a prism is in the position of > The deviation produced by a prism is maximum
minimum deviation, a ray of light passes symme
trically (parallel to the base) through the prism so that
Flo when the angle of incidence is 90°.
5 max = 90°+ i'-A

ee
/ = /', r = r', 8 = 6m > For a small angled prism, angle of deviation
6 = (p - 1) A But for a prism with larger refracting

Fr
As A + d= i + i'
angle, 8 = i + /' - A
. A+ 5m
A + 6
m
= 1 + 1 or 1 =
> There are two angles of incidence i and /' for which a
2
ray of light passing through a prism deviates through
for
ur
Also A=r+ r' = r+ r=lr the same angle 6.
A
> There is one and only one angle of incidence for
2
which the angle of deviation is minimum.
ks

From Snell's law, the refractive index of the


^ The deviation [6 = (p - 1) A] produced by a prism of
Yo

material of the prism will be


oo

small angle is independent of the angle of incidence.


. A+5 m Moreover, this expression indicates that thin sheets of
sm
B

sm i
M =- or ^ =
2 glass (A w 0°) cannot deviate light rays.
sm r . A
re

sm -
2 9.33 DISPERSION OF WHITE LIGHT
By measuring the values of A and. 8,„ , with the 54. What is dispersion of light ? Explain it with a ray
ou
ad

help of a spectrometer, the refractive index p of the diagram. Also explain the cause of dispersion oflhht
prism glass can be determined accurately. r^● t , -
Y

^ Dispersion of white light. When a narrow beam of


9.32 DEVIATION THROUGH A PRISM OF sunlight is incident on a glass prism, the emergent light
when made to fall on a screen shows several coloured
nd
Re

SAAALL ANGLE
bands. Broadly, the component colours are in the
53. Derive an expression for the angle of deviation of a sequence : violet, indigo, blue, green, yellow, orange
Fi

small prism in terms of the refractive index and the angle and red (given by the acronym VIBGYOR). The red
of the prism. colour bends the least and the violet colour bends the
Deviation produced by a prism of small angle, niost, as shown in Fig; 9.123.
Refer to Fig. 9.117(fl). Suppose the light is incident at a
small angle / on the prism, then angles r, r' and i' will
also be small. Screen

For refraction at face AB, we have Red


sm i I
Orange
i =pr a'-® Yellow
sm r r Glass
Green
prism
For refraction at face AC, we have Blue
Indigo
sin /' Violel
P = i' =p r'
sin r' r'
Fig. 9.123 Dispersion of white light by a glass prism.
RAY OPTICS AND OPTICAL INSTRUMENTS 9.69

The phenomenon of splitting of white light into its Thus the red colour is deviated the least and the
component colours on passing through a refracting violet is deviated the most. Other colours are deviated
medium is called dispersion of light. The pattern of the by angles between and So different colours
coloured bands obtained on the screen is called contained in white light emerge from the glass prism
spectrum. in different directions due to their different wave
lengths, which is called dispersion.
Newton's classic experiment on dispersion of white
light. It can be easily seen from experiments that a Table 9.2 Refractive indices for
prism only separates the colours already present in Different Wavelengths
white light but the prism itself does not create any
colour. Colour Wavelength A Crown glass Flint glass
Violet 3969 1.533 1.663

Blue 4861 1.523 1.639

Yellow 5893 1.517 1.627

Red 6563 1.515 1.622

55. What are dispersive and non-dispersive media ?


Give examples.
Dispersive media. In some lefracting media,
Fig. 9.124 Recombination of white light. different colours of light travel with different speeds.
The variation of refractive index with wavelength is

As shown in Fig. 9.124, take two prisms Py ?2 of the


highly pronounced for such media. These media which
bring about a good dispersion of white light are called
same glass material and of same refracting angle (A).
dispersive media. For example, glass, quartz, etc.
Place the second prism P2 upside down with respect to
the first prism Py Allow a narrow beam of white light Non-dispersive media. In vacuum, all colours of
to fall on the prism Pj and observe the emergent beam light travel with the same speed. So the refractive
from prism P2 on a screen. A patch of white light is index of vacuum is independent of wavelength. White
seen on the screen. Clearly, the first prism disperses light does not undergo dispersion in vacuum. Such a
the white light into its component colours, which are medium is called a non-dispersive medium,
then recombined by the inverted prism into white
9.34 ANGULAR DISPERSION AND
light. This proves that white light itself consists of
DISPERSIVE POWER
different colours which are just dispersed by the prism.
Cause of dispersion. Each colour of light is 56. Define angular dispersion and dispersive power.
associated with a definite wavelength. In the visible Wn'fe expression for these quantities in terms of
spectrum, red light is at the long wavelength end refractive index.
(~700nm) while the violet light is at the short Angular dispersion and dispersive power. When a
wavelength end (~400nm). Dispersion takes place beam of white light passes through a prism, it gets
because the refractive index of the refracting medium is dispersed into its constituent colours. Let dy, 5^ and 5
different for different wavelengths. The refractive index p ^^e angles of deviation for violet, red and yellow
of a material for wavelength X is given by the Cauchy's (^ean) colours respecHvely, as shown in Fig. 9.125.
relation :
b c

where a, b and c are constants, the values of which


depend on the nature of the material. Also, for a
small-angled prism, the angle of deviation is given by
6=A(p-l)
Now X red
> X
violet

^‘^ed violet'“Whence 6,^ < 5


violet Fig. 9.125 Angular dispersion.
9.10 PHYSICS-Xil

Then
h^={)^y-\)A !Examples based on ●V.

_{■ Refraction and Dispersion of Light


5 = (m-1)A through a Prism
where and ^ are the refractive indices of the Formulae Used
prism material for violet, red and yellow (mean)
1. For refraction through a prism,
colours, respectively.
A+ 8 = i + i' and r + r' = .4
The angular separation between the two extreme 2. In the condition of minimum deviation.
colours (violet and red) in die specPum is called the A+ 6
m
angular dispersion. sm -
2
/ = !', r = r', 5 = 5 ni ; p =
Angular dispersion .
Sin
A
2
= 5^, - 5^

w
= -1) A = (p^, -Pr) A 3. Deviation produced by a prism of small angle,
8 = (p-l)A
Clearly, the angular dispersion produced by a
prism depends upon (/) angle of the prism and 4. Angular dispersion = 5^,-6^ =(py,-p^) A
(n) nature of the material of the prism.
Dispersive power is the ability of the prism material
Flo
to
5. Dispersive power, o) = ——^
6 p - 1

ee
cause dispersion. It is defined as the ratio of the angular 8w + 6
V A
dispersion to the mean deviation. 6. Mean deviation, 6 =

Fr
2
Dispersive power.
Angular dispersion _ 5y, - 6^ 7. Mean refractive index, p = ^ ^ 2 ^ ^ .
co =
Mean deviation
for
5 Units Used
ur
(p^^ -1) A-(p^ -1) A Angles i, i', r, r'. A, 5,5^^ 5^, and 5^ are in degrees;
dispersive power co and refractive indices p y-, p ^
ks

and p have no units.


Yo

or Ci> =
p-1
oo

Example 92. Calculate the refractwe index of the material


of an equilateral prism for which the angle of minimum
^ For Your Knowledge
B

deviation is 60°. [CBSE SP 981


> The refractive index (p) of any material for yellow
re

light is equal to the mean of the refractiveindices for Solution. For an equilateral prism, A =60°.
the violet and red lights, i.e.. Also 8_
m
=60°
ou

Refractive index of the prism material is


ad

2
. A + 5m . 60® + 60®
sm sm
sin 60° ^/3 2 r~
Y

That is why yellow light is called mean light and its 2 2


F = = — X — = V3.
deviation is called mean deviation, which is given by .
sm
A .
sm —
60° sin 30° 2 1
+ 5
6 =
R 2 2
nd
Re

> Due to its small wavelength, violet light is harmful to Example 93. A ray of light passes through an equilateral
glass prism, such that the angle of incidence is equal to the
Fi

our eyes. So in experiments, angular dispersion and


dispersivepower of a material are measuredfor blue angle of emergence. If the angle of emergence is 3/4 times the
and red lights. Thus angle of the prism, calculate (i) the angle of deviation and
OJ =
F B ~ Fr (ii) the refractive index of the glass prism. [CBSE OD13CI
p-1

> The dispersive power depends on the nature of the


Solution, (i) Here A =60°, i = /' = ^ A = 45°, p = ?
As A+ 5=1 + 1''
material of the prism and not on its refracting angle,
A However, both angular dispersion and mean 60+ 5 = 45°+45° or 5 =90°-60° = 30°.
deviation also depend on the angle of the prism. . A + 5m . 60° + 30°
> Greater the dispersive power of a material, larger is the sm
2
sm
2
(«) F =
spread of a spectrum produced by the prism of the sm
A
sm
60°
material. 2 2

> Dispersive power of flint glass is more than that of sm45°_l/>^ = ^^ = 1.414.
crown glass. sin30° ~ iJT
RAY OPTICS AND OPTICAL INSTRUMENTS 9.71

Example 94. A ray of light passing through an equilateral Let i be the angle of incidence at face AB. Then,
sinf
triangular glass prism from air undergoes minimum deviation P =-
sin I
or V2 =
when angle of incidence is 3/4th of the angle of prism. Calailate smn
1
sinl5°

the speed of light in the prism. [CBSE D 08 ; OD 17]


i = sin ^(V2sin 15°).
Solution. Here i = — A = — x 60= 45°
4 4 Example 97. A ray PQ incident on the face ABofa prism
= 30° ABC, as shown in [Fig. 9.127(a)], emerges from the face AC
In the position of minimum derivation, ^ = y such that AQ = AR. Draw the ray diagram showing the
sin i sin 45°
= -=
1
X ?- = T2
- passage of the ray through the prism. If the angle of the prism
is 60° and refractive index of the material of the prism is -73,
P =.-
sin r sin 30° J2 1
3x10
8 determine the values of angle of incidence and angle of
Hence, v = — ms ^ [ V p=—] deviation. [CBSE OD 15]
p -Jl V

= 0.707 X 3 X 10®ms”^ = 2.12 x 10® ms"^


Example 95. A ray of light incident on a face of an equi
lateral prism shows minimum deviation of 30°. Calculate the
speed of light through the prism. [CBSE D17; SP 23] S

Solution. In the position of minimum deviation.


A
r = —=30°
2

I =
A+5...
m
60°+30°
= 45° Solution. The ray QR passes parallel to the base of the
2 2
prism. It is minimum deviation position of the prism.
sin/ sin45° _ 1 xi: = V2 A 60°
= 30°
P = -
smr sin30°”^''1 2 2

Speed of light in the prism, As P =.-


sini

c 3x10® ms"* smr


v = = 0.707 x3xl0®ms"* = 212x10® ms"*.
P V2 smj
sin / = 73 X —
1
V3 = or
sin30° 2 ” 2
Example 96. Calcidate the value of the angle of incidence
when a ray of light incident on one face of an equilateral glass Angle of incidence, i = 60°
prism produces the emergent ray, which just grazes along Also, / -I- /' = A + S or 60°+60° =60°+5
the adjacent face. Refractive index of the prism is -Jl. Angle of deviation, 5 = 60°.
[CBSE D 17, 22 ; OD 17C ; SP 23]
Example 98. A ray of light is incident on a prism at an
Solution. As shown in the Fig. 9.126, let ^ be the angle of 45° and passes symmetrically as shoivn in thefigure.
angle of incidence at face AC. For the grazing ray RC,
the angle of emergence, e =90°.

Calculate : (a) the angle of minimum deviation,


. 9.126 (b) the refractive index of the material of the prism, and
sin90°
P=.— = ^/2 (c) the angle of refraction at the point P. [CBSE OD 22]
smr2 Solution, (a) As the ray passes symmetrically
1 through the prism, so
or smr, =-7=
^ -J2
=> r2=45° i = e = 45°

n1 +45°=60° As A+ 5= /+ e
But
rj + rj = A or

.-. 60°+5... =45°+45° => 6... = 30°.


"i = 15° III in
PHYSiCS-XII

A = r+r = 60° => r=30°


Solution. Refer to Fig. 9.129(&). Applying Snell's
sin; sin45® 12/- law at face AB,
P =.-
sinr 1X sin60°= V3 sin r
A
1
(c) Angle of refraction at point P, r = — =30'’. inr = -s/3 X = ●. r=30°
y/3 2
Example 99. A ray of light PQ is incident at angle of => The transmitted ray is perpendicular to AD.
60® on the face AB of a prism of angle 30° as shown in t =30°
1 1
Fig. 9.128(a). The ray emerging out of the prism makes an
M =- = 2.
angle of 30° with the incident ray. Show that the emergent sm^ sin 30°

ray is perpendicular to the face BC through which if emerges.


Also calculate the refractive index of the prism material. Example 101. An equilateral glass prism has a refractive
index 1.6 in air. Calculate the angle of minimum deviation of
(CBSE D 02C]

w
the prism, when kept in a medium of refractive index 4V2 / 5.
[CBSE D19]

Solution. Here A = 60°,


5 ' P2=1.6

Flo ^2_
sin
. A+8 m
2

ee
. A
sin —

Fr
2
. 60°+5 m
sin —
1.6 2
Solution. Here i =60° A =30° 4^/2
for
ur
sin 30°

As the emergent ray makes an angle of 30° with the 5


incident ray, so . 60°+8 m —
1.6x5
sm = sin 45°
4^ "" 2 “ ^
ks

angle of deviation, 8=30° 2


Yo

Now I + i' = A + 8 60°+8 m


oo

.'. Angle of emergence. ^ =45° or =90°-60°=30°.


B

i' = A + 8 -1 =30° + 30° - 60° = 0° Example 102. One face of a prism of refracting angle30°
Thus the emergent is perpendicular to the face BC and refractive index 1.414 is silvered. At zvhat angle must a
re

through which it emerges, as shown in Fig. 9.122(&). ray of light fall on the unsilvered face so that after refraction
When r=:0°, F=o° into the prism and reflection at the silvered surface it retraces
ou

its path ?
ad

r= A-/ =30°-0°=30°
sin I _ sin 60° -Js/2 = V3. Solution. The ray will retrace its path at the
Y

p =- silvered face when it is incident normally on it.


sinr sin 30° 1/2
i'=0 and so /=0 A
nd

Example 100. A composite prism ABC is made up of two


Re

As r + r" = A or r + 0=30°
identical right-angled prisms ABD and ADC made up of r = 30° '30"
Fi

different materials of refractive indices ^/3 and p respectively. sin i


A ray of light is incident on face AB of this prism at 60° as As p =- 90"i
[R
shown in Fig. 9.129(a). It is observed that the final emergent sm r

ray grazes along face AC. Find the value o/p. [CBSE 20C] 1.414 =
sm i
= 2 sin /
I
Q
..jr

A A
sin 30°

. . 1.414 V2 1 ^
^4 or sm 1 =
2 2 ^/2
a c

\ Hence i = 45°. Fig. 9.130


60‘
Incident
-J3 ' JcY' A^mergern Example 103. Cross-section view of a prism is the equilateral
i£L
^ triangle ABC shown in thefigure 9.131. The minimum deviation
is observ&i using this prism when the angle of incidence is equal to
a
D

tl-ie prism angle. Find the time taken by light to travel from P
(6) (midpoint of BC) to A
RAY OPTICS AND OPTICAL INSTRUMENTS

(Given, speed of light in vacuum For the prism placed in water.


/a
( ft
=3xl0^m/sand cos30°=—) P
b^=fp^-l)A = IV
-1 A

IJEE Main Aug 21]


Solution.
(312 -1
(9
A = --1
.'i A=-A
„ 1 „
4/3 > 8 8
. A + 8m
= A ^ 5... - A
2
III
Hence S^^l/8_1
A+8 m 6, 1/2 4
sm -
sin A
P =
2
=2cos—=2cos30°=V3 Example 107. A prism with refracting angle of 60°, gives
.A ■ . A 2 angle of minimum deviation 53°, 51° and 51° for blue, red
sm sin
2 2 and yelloza light respectively. What is the dispersive power of
the material of the prism ? [ISCE 96)
c ^3x10'
Solution. Here 5g =53°, 5j^=51°, 5y=52°
V
pnsm
p V3
■J3 Sr -5 R -
53-51
= 5^/3 X10'^ m = 0.038.
PA = ABcos30°= 10 X
2
cm
Dispersive power, co = —^ y
52

Time taken by light travel distance PA Exa m pie 108. 77jc reactive indices offlint glass for blue and
5^/3xl0"^x^^ red colours are 1.664 and 1.644. Calculate its dispersive pozuer.
s=5xlO’^V
3x10^ Solution. Here pg =1.664, p^^ =1.644
Example 104. Find the angle of dispersion between red 1.664 + 1.644
= 1.654
and violet colours produced by a flint glass prism of ^ 2 2
refracting angle60°. Refractive indices of prism for red and
Dispersive power.
violet colours are 1.622 and 1.663, respectively.
Solution. Here co =
Pg-p^ 1.664-1.644 = 0.0305.
p-1 1.654-1
A =60°, Pg =1.622, p^,. =1.633
Angular dispersion between red and violet Example 109. The refractive indices for a material for red,
colours is violet and yellow lights are 1.52,1.62 and 1.59 respectively.
Calculate the dispersive power of the material. If the mean
Sp ~h = V -p^) = 60°(1.663-1.622) deviation is 40°, then what zvill be the angular dispersion
= 2.46°.
produced by a prism of this material ?
Example 105. A thin priszn of refracting anglel° deviates Solution. Dispersive power.
an incident ray through an angle ofV. Find the value of Py P/; _ 1-62 —1.52 = 0.169
refractive index of the material of the prism. co =
1.59-1
Py -1
Solution. Here A =2°, 6 = 1° p =1
5„
V
-6 R
Also
Deviation through a prism of small angle is given by co =

5y
6 = {p-l)A
Angular dispersion
6 1
+ 1 = - + 1 = 1.5.
P =
A 2
= 5^^ - = to6y =0.169 X 40° = 6.76°.
Example 110. In a certain spectrum produced by a glass
Example 106. Show that the angle of minimum deviation prism of dispersive power 0.031, it is found that the
produced by a thin prism is reduced to one-fourth (with refractive index for the red ray is 1.645 and that for the blue
respect to air) when it is immersed in water. Given ray is 1.665. What is the refractive index of yellow ray ?
a

p^=3/2«nd V^=4/3. Solution. Here o)=0.031, pj^= 1.645, pg =1.665


Solution. Deviation produced by thin prism,
As (0 =
8 = (p-l)A
Py -1
For the prism placed in air. 1.665-1.645
or
Py =1 + =1 +

5,={> -1).4=(|-1|/1 =
(0 0.031
^ = 1 + 0.645 = 1.645.
9.14 PHYSICS-Xli

Example 111. /4 deviation of 2° is produced m the yellow dp =0.5835{^^8,„)


ray when prisms of crown and flint glass are achromatically 1 71 12.76
combined. Take dispersive powers of crown and flint glass as = 0.5835 X — X
5 180 6300
0.02 and 0.03 respectively and refractive index for yellow
light for these glasses are 1.5 and 1.6 respectively. What will Hence % error =
dp xl00 =
12.76
X
100
be the refracting angle for crown glass prism in degrees ? 6300 1.6242
(Round off to the nearest integer) 12.76
= 0.12.
[JEE Main March 21] 100.8
Solution. Here 0)^ =0.02, (O2 =0.03,^^ =1.5,}i2=1.0
For the achromatic combination. 7^. roblems For Practice

e ttel = 0
=> 0j-e2=o 1. A ray of light is inclined to one face of a prism at an
angle of 50°. If the angle of prism be 60° and the ray
0^=02

w
be deviated through an angle of 42°, find the angle
C0j8j = 0)262 which the emergent ray makes with the second face
of the prism. (Ans. 28°)
S^=Si-52=2°
5. -1
o),8i = 2°
“2
^ 8, 1-^ =2°
I
0)

^ Flo 2. A ray of light strikes one face of the prism at an


angle of incidence 60° and angle of refraction is 30°.
If the angle of prism is 60°, find the angle of

ee
f 2 emergence. (Ans. 60°)
Si = 2° => 6,1 =2x3=6°

Fr
3J 3. The refracting angle of a glass prism is 60°. The
value of minimum deviation for light passing
But
5^=(Pj-l)Aj through the prism is 40°. Calculate the value of
for
6°=(1.5-1)A^ refractive index of the material of the prism. Given
ur
sin 50° = 0.766. (Ans. 1.532)
AI = 12°.
4. A ray of light incident at an angle of 48° is refracted
Example 112. A spectrometer measures angles correct
ks

to through a prism in its position of minimum


6' of att arc. If an experiment gives
Yo

deviation. The angle of prism is 60°. Calculate the


oo

A=60°0'; 6,„=48°36', refractive index of the material of the prism,


calculate the percentage accuracy of the value ofp. (sin 48° = 0.74, sin 30° = 0.50) (Ans. 1.48)
B

. A + 5m . r60°+48°36
I \
5. A glass prism has a refracting angle of 60°. The
re

sm sm
2
angle of minimum deviation is 40°. If the velocity of
2
Solution, p = . A . 60°
light in vacuum is 3xl0^ms“^ calculate the
velocity of light in glass. At what angle the ray
ou

sin sin —
ad

2 2
should be incident ? (Ans. 1.958 x 10® ms'^, 50°)
= 2 sin 54°18' = 2 x 0.8121 =1.6242
Y

6. A ray of light falls normally on the face of a glass


Differentiating p with respect to 5^, we get prism having refractive index of 1.5. Find tlie angle
of prism, if the ray just fails to emerge from the
nd
Re

dp 1
cos
second face. (Ans. 41.8°)
. A 2 2
7. As shown in Fig. 9.132, PQ
Fi

m sm —
2
is the ray incident on a
1 1 A+d tit prism ABC. Show the corres
X — cos
sin 30° 2 2 ponding refracted and ^
emergent rays. The critical
A + 8
= cos
m
= cos 54°18' =0.5835 angle for the material of the
2 prism is 45°. Also find
the refractive index of the
or dp - 0.5835 db tn
material of the prism.
Now the error in measuring 6 m
= ± 6' (Ans. V2)
8. A ray of light falls on a glass prism at an angle of
.●. Total range of error is
incidence of 30°. Find the direction of the emergent
fiX 1 71
ray if the angle of the prism is 60° and refractive
db m =12'= - — X rad
5J 5 180 index is 1.50. (Ans. /' = 77°7')
RA'^ OPTICS AND OPTICAL INSTRUMENTS 9J5

9. A thin prism of 6® angle gives a deviation of 3°. Mj and «2 depend on X, the wavelength of light,
What is the refractive index of the material of the according to the relation -14
-14
prism ? (Ans. 1.5) 10.8x10 1.8x10
«i = 1.2 +
X^
and ti2 = 1.45 + X^
10. One face of a prism with a refracting angle of 30° is
coated with silver. A ray incident an another face at Find the wavelength in nm for which rays incident
an angle of 45° is refracted and reflected from the silver at any angle on the interface BCpass through without
coated face and retraces its path. Find the refractive bending at that interface. [JEE Main July 21]
index of the material of the prism, [CBSE F 09] (Ans. 600 nm)
(Ans. 1.414) HINTS
11- Calculate the angle of minimum deviation for an 1. In Fig. 9.133, i = 90° - 50° = 40°, A = 60°, 6 = 42°
equilateral prism of refractive index ^f2. [CBSE D 22] As i + i'= A + 5
(Ans. 30°) i'= A + 6 - i = 60° + 42° - 40° = 62°
12. In the given figure, the face AC of the equilateral Angle made by emergent ray with second face
prism is immersed in a liquid of refractive index 'n = 90° - 62° = 28°.
A

Liquid
n

B C

For incident angle 60° at the side AC, the refracted Fig. 9.133
light beam just grazes along face AC. The refractive
2. Here 1 = 60°, r = 30°, A = 60°. Refer to Fig. 9.126.
index of the liquid ^ . Find the value of x.
As r+r' = A
r' = A - r = 60° - 30° = 30°
(Given refractive index of glass = 1.5)
[JEE Main July 22] i.e., r = r' and the ray passes symmetrically through the
(Ans. 27) prism
i' = i = 60°.
13. A thin prism ^ of angle 6° and refractive index for
yellow light py = 1.5 is combined with another . A+6 m . 60°+ 40°
sm sm -

prism ?2 of angle 5° and p y = 1.55. The combination 3. p =


2 2
. A . 60°
produces no dispersion. sm sin —-

The net average deviation 2 2

(5) produced by the sin 50° 0.766


= 1.531
sin 30° 0.5
combination is . Find
\xj 4. Given i = 48° A = 60°
the value of x.
In the position of minimum deviation,
[JEE Main July 22]
A = r+r =r+r or r =—
(Ans. 4) 2

14. A prism of refractive index n^ and another prism of =


sin i
-
sin i _ sin 48° _ 0.74 = 1.4a
refractive index are stuck together (as shown in sm r
gin A “sin 30°
the figure). 2
60° + 40°
sm ●
c C 2 sin 50° _ 0.7660
5. p =-
V sin 30° sin 30° 0.5
8
0.5 X 3 X 10
= 1.958x10® ms‘^
0.7660
. A+5 m 60° + 40°
_
= 50°.
2 2
PHYSICS-XII

6. Refer to Fig. 9.134. As the ray PQ is incident 11. Here A =60° m=V2, r = —= 30°
normally on face AB, so the refracted ray QR goes 2

straight and is incident on face ACat an angle equal sini


=> V2 =
sm?

to A. As the ray RS just fails to emerge from the face smr sin 30°
AC, so angle A equals the critical angle for the
prism material.
1
sin/ = V2x-^=sin45° =* i=45°
sm = —
=2/'-A =2x45°-60°=30°.
1 12. Using Snell's law for refraction at face AC,
sin A =
1.5 sin 60°= Jtsin90°

V3
1.5x— = ti~
2 4

3^/3 = V? => -JT? = Vx

w
.r = 27.

13.
5 = S, ~ 83

Flo = (5y-l)A,-(p;,~l)A2
= (1.5-l)x6-(1.55-l)x5

ee
= 3-2.75=0.25

1 _ 25 ^ 1

Fr
=> AT =4.
a; 100 4
7: Refer to Fig. 9.135. As the ray PQ is incident
normally on face AB, so the refracted ray QR goes 14. For no bending at the interface,
straight and is incident on face AC at 45°. Now the
for
-14
ur
-14
10.8x10 1.8x10
angle of incidence is equal to critical angle 1.2 + = 1,45 +

(i = 1^=45°), the emergent ray QR goes along


second face RC. 9xl0"*‘‘
= 0.25
ks

=>

1 1
Yo

P = = V2.
sin sin 45° -xlO"^ =6x10 -7 m
oo

=>
5

8. For refraction at first face,


B

= 600 nm.
sin i sin 30°
re

= |i or = 1.5
sin r sm r
9.35 PURE AND IMPURE SPECTRA*
r = sin 30° _ 0.5
ou

or sin = 0.3333 or r = 19°287. 57. Distinguish between monochrornatic light and


ad

1.5 "Is
polychromatic light.
Angle of incidence at second face,
Y

Monochromatic and polychromatic lights. A light of


r' = A - r = 60° - 19°28' = 40°32' single wavelength is called monochromatic light. The
nd

commonly used source of monochromatic light is


Re

For refraction at second face, — = —


sin /’ ij sodium lamp which emits yellow light of two wave
lengths 5890 A and 5896 A. As the two wavelengths
Fi

are
or sin r= )j xsin r'= l.Ssin 40°32’
ver)' close, so sodium lamp can be regarded as a source
= 1.5 x 0.6499 = 0.9748
of monochromatic light of mean wavelength 5893 A.
.'. Angle of emergence, Generally, the sources of light are polychromatic
i’ = sin"’ (0.9748)= 77°7'. which give light of several wavelengths. Optical filters can
be used to obtain light of particular wavelength from
9. Here A = 6°, 5 = 3°, them.

For a prism of small angle, 8 = (|.i - 1) A 58. What are pure and impure spectra ? Give the basic
principle for the production of a pure spectrum.
A 6
Pure and impure spectra. A spectrum in which the
or IX = 1+ 0.5 = 1.5. component coloiirs of the spectra of different rays overlap
each other and the various colours are not distinctly seen is
10. Proceed as in Example 102 on page 9.72. called impure spectrum. On the other hand, a spectrum in
RAY OPTICS AND OPTICAL INSTRUMENTS 9.77

which there is no overlapping of different colours and the focal plane of lens When a light source is kept in
different colours are distinctly seen is called the pure front of the slit, a parallel beam of light emerges from the
collimator.
spectrum.

White light
\

Fig. 9.136 (a) An impure spectrum.

Production of pure spectrum (Basic principle). As


shown in Fig. 9.136(b), light from a bright source illumi Fig. 9.137 Spectrometer.
nates a narrow slit S. The slit is adjusted at the focus of a
convex lens Lj. Parallel rays emerging from the lens fall 2. Prism table. It is a circular horizontal plate on
on a prism. Rays of different colours are refracted by ^^hich the prism is placed. It can be adjusted at a
different amounts but the rays of same colour remain desired height with the help of a clamping screw. It can
parallel to one another. The emergent rays are focussed rotated about a vertical axis. Its position can be
by convex lens on a screen placed at the focus of noted with the help of the verniers and V2 attached
to it and moving over a graduated circular scale
Slit
carried by the telescope.
3. Telescope. It is used for observing the spectrum.
It is an astronomical telescope having a convex lens
\ V at one end and Ramsden eyepiece at the other. It is
Violet mounted horizontally on a vertical arm attached to the
Visible main circular scale. It can be rotated about the same
spectrum vertical axis about which the prism table rotates. Its
Fig. 9.136 (b) Production of pure spectrum. position can be noted on the circular scale by the
vernier's and A cross-wire is fixed at the focus of
the eyepiece.
As rays of different colours suffer different
deviations, they cannot be focussed on the same Working, for getting a pure spectrum, the following
screen. To overcome this difficulty, the prism should adjustments are made in a spectrometer :
be placed in the minimum deviation position with respect 1. Focussing the eyepiece. The eyepiece of the
to some mean (yellow) colour. Then the rays of telescope is moved in and out so that the cross-wires
different colours will suffer almost the same deviation are clearly visible.
and can be focussed on the same screen. 2. Focussing the telescope for parallel rays. The
telescope is turned towards a distant object. The
9.36 SPECTROMETER* distance between the eyepiece and the object is so
59. What is a spectrometer ? Explain its construe- adjusted that object becomes clearly visible. This sets
tion. How can it be used to obtain a pure spectrum ? the telescope for receiving the parallel rays.
Spectrometer. It is an optical device used for pro 3. Focussing the collimator for parallel rays.
ducing and studying the spectra of different light sources. Illuminate the slit with a bright source and view it
through the telescope. Adjust the distance between the
Construction. A spectrometer has three main parts: slit and the collimator lens till a clear image of the slit is
1. Collimator. It produces a parallel beam of light. seen. This sets the collimator to provide a parallel
It consists of two co-axial metal tubes. The outer tube is
beam of light.
mounted horizontally and carries a convex lens Lj at its
4. Setting the prism. The prism is placed at the
free end. The inner tube has an adjustable vertical slit
at the free end and can be slided inside the outer tube centre of the prism table. The prism table is rotated so
by a rack and pinion arrangement. The slit is adjusted in
that light from the collimator falls on refracting face AB
PHYSICS-XII

Vernier

Collimator , L Telescope
^2

0 z R
QSlit % L
Eyepiece
Vernier
V^2
Fig. 9.138 Working of a spectrometer.

and after refraction emerges from the other face AC.


The prism causes dispersion. The rays of a given colour
emerge parallel to each other. They are received by the

w
telescope. All red rays are focussed at R, all violet rays
at V and rays of other colours in between and a
spectrum RV is formed in the focal plane of the objec
tive, as shown in Fig. 9.138. A magnified spectrum is
viewed through the eyepiece.
Flo
ee
5. To get ird of overlapping of colours, the prism is Minimum
set in the minimum deviation position for some mean deviation

Fr
(yellow) colour. This gives a pure spectrum. position/ ●jCl Direct
T, image
60. State some of the important uses of a spectrometer. T: position
Uses of a spectrometer :
for
ur
1. To measure the angle of the prism, Fig. 9.139 Setting the prism in minimum deviation position.
2. To determine the refractive index of the prism
ks

material. Measurement of A. Place the prism ABC on the


Yo

3. To determine the wavelength of light. prism table so that light falls directly on faces
oo

AB and AC of the prism, as shown in Fig 9.140. Look


4. To measure the dispersive power of a prism.
for the brightest image of the slit formed by reflection
B

9.37 MEASUREMENT OF REFRACTIVE of light from laces ABand AC. Set the telescope in
re

INDEX BY A SPECTROMETER*
position Tj so that cross-wire coincides with the image
of the slit from face AR Turn the telescope to the
61. How can a spectrometer be used to determine the position Tj so as to focus the image of the slit from face
ou
ad

refractive index of the material of a prism ? AC. Let 0 be the angle through which telescope turns.
Y

Measurement of refractive index (p). The refractive Then, A = -.


2
index p of the material of a prism is given by s
nd
Re

. A + 5m
Sin
2
Fi

. A Collimator
sm
2

To determine p, we need to measure : k >1

(i) angle of minimum deviation ( 6,^) and


(«) the refracting angle of the prim (A).
Measurement of 5^^. Set the prism in the minimum
deviation position, as shown in Fig. 9.139. Turn the
telescope so that its cross-wire coincides with mean

(yellow) colour of the spectrum. Let this position be T^.


Remove the prism. Turn the telescope to position Tj so
that direct image of slit is seen. The difference between
the two positions gives 6 w ■ Fig. 9.140 To measure angle of the prism.
/
/

RAY OPTICS AND OPTICAL INSTRUMENTS 9J9\

9.38 SCATTERING OF LIGHT molecules scatter light in all directions. But scattering
is preferential. According to Rayleigh s law of
62. What do you mean by scattering of light ? What scattering, the intensity of scattered light,
are elastic and inelastic scatterings ? 1

Scattering of light. This is the phenomenon in which


light is deflected from its path due its interaction with the wavelength (blue) end of
particles of the medium through which it passes. Basically, spectrum is scattered about ten times more
the scattering process involves the absorption of light wavelength (red) end. When
by the molecules followed by its re-radiation in gj^y^ scattered light enters our eyes
different directions.
and this light contains blue colour in a larger
Two types of scatterings : proportion. That is why the sky appears blue.
1. Elastic or Rayleigh scattering. When the size 'a If the earth had no atmosphere, there would be no
of the scattering particles is much smaller than the scattering of light, the sky would appear black and
wavelength 'X of incident light, there is no exchange of stars could be seen during day hours. This is what
energy between the incident light and the scattering astronauts actually observe at heights 20 km above the
particles. Consequently, there is no change in the earth where the atmosphere becomes quite thin or on
frequency or wavelength of the scattered light. This the moon which has no atmosphere,
type of scattering is called elastic or Rayleigh 2. Reddishness at sunset and sunrise. When the
scattering. It obeys Rayleigh's law of scattering. sun is near the horizon at sunset or sunrise, the light
2. Inelastic scattering. When the size of the scattering rays have to traverse a larger thickness of the
particles is much greater than the wavelength of atmosphere than when the sun is overhead at noon. In
incident light i.e., a»X, there is interchange of energy accordance with Rayleigh's scattering law, the lower
between incident light and the scattering particles, wavelengths in the blue region are almost completely
Consequently, the scattered light has a frequency or scattered away by the air molecules. The higher
wavelength different from that of incident light. This wavelengths in the red region are least scattered and
type of scattering is called inelastic scattering. For reach our eyes. Hence the sun appears almost reddish
example, the Raman effect and Compton effect. at sunset and sunrise.

Sun nearly
63. State Rayleigh's law of scattering. ■ overhead

Rayleigh's law of scattering. According to Rayleigh's


laiu of scattering, the intensity of light of wavelength X Sun appears
Blue scattered Less blue
scattered
present in the scattered light is inversely proportional to the reddish away

fourth power of X, provided the size of the scattering


particles are much smaller than X. Mathematically,
/ocA-4 [For fl < < ^] Sun near Observer
X horizon

Thus the scattered intensity is maximum for shorter


Fig. 9.141 Absorption of sunlight at sunset and sunrise.
wavelengths.
64. Explain different phenomena of daily life which 3. Clouds appear white. Large particles like
are based on scattering of light. raindrops, dust and ice particles do not scatter light in
Daily life phenomena based on scattering of light, accordance with Rayleigh's law, i.e., their scattering
Several beautiful phenomena in nature are based on power is not selective. They scatter light of all colours
scattering of light. Some of these include the blue almost equally. Hence the clouds which have droplets
colour of sky, white clouds, the red hues of sunrise and of water with a»X are generally white,
sunset, the rainbow, the brilliant colours of some 4. Danger signals are red. According to Rayleigh's
pearls, shells, and wings of birds. We describe some of law, the intensity of scattered light is inversely
these phenomena. proportional to the fourth power of wavelength. In the
1. Blue colour of the sky. The blue colour of the sky visible spectrum, red colour has the largest wave-
is due to the scattering of sunlight by the molecules of length, it is scattered the least. Even in foggy conditions,
the atmosphere. As sunlight passes through atmos such a signal covers large distances without any
phere, the nitrogen and oxygen molecules of air absorb appreciable loss of intensity due to scattering.
some amount of sunlight and re-emit it. The free gas Therefore, red coloured signals are preferred.
9.80 PHYSICS-XII

9.39 RAINBOW* red rays emerge from the water drops at angle of 51°
65. What is a rainbow ? Explain the formation of and the outer violet rays emerge at angle of 54°.
primary and secondary rainbow.
Rainbow. The rainbow is nature's most spectacular
N

display of the spectrum of light, produced by refraction, J=.


^ " KX ^ Secondary
dispersion and internal reflection of sunlight by spherical
rain drops. It is observed when the sun shines on ram c/5
Ik
drops, during or after a shower. An observer standing *■
R
with his back towards the sun observes it in the form of R 's Primary
concentric circular arcs (bows) of different colours in the
horizon. The inner brighter rainbow is called the V

prmiary rainbow and the outer fainter rainbow is called

w
the secondary rainbow. 54°
O',

*0 I

Observer

Flo Fig. 9.143 Formation of primary and secondary rainbow.

ee
9.40 OPTICAL INSTRUMENTS

Fr
66. What are optical instruments ? What are the two
essential features that must be possessed by optical
instruments for viewing objects distinctly ?
for
ur
Optical instruments. Optical instruments are the devices
which make use of mirrors, lenses and prisms and are primarily
Sunlight used to extend the range of vision of human eye. For example,
ks

microscopes are used for viewing tiny objects clearly


Yo

while telescopes are used to see distant objects clearly.


oo

Purposes of using optical instruments :


B

1. To form the images of close lying objects at the


least distance of distinct vision.
re

2. To increase the visual angle and hence to obtain


magnification.
Fig. 9.142 Dispersion of sunlight of a single raindrop
ou
ad

(a) primary rainbow and (b) secondary rainbow. 3. To improve the resolving power of the eye.
Y

The primary rainbow is formed by rays which Essentia! features of an optical instrument. The
undergo one internal reflection and two refractions design of an optical instrument must meet the
following two requirements :
nd

and finally emerge from the raindrops at minimum


Re

deviation. The red rays emerge from the water drops at 1. High magnification. Magnification is the ratio of
angle of 43° and the violet rays emerge at another the size of the final image to the size of the object. An
Fi

one

angle of 41°. The parallel beam of sunlight getting instrument with high magnification makes
dispersed at these angles produces a cone of rays at the n^ore clear and comfortable, by increasing the
observer's eye, as shown in Fig. 9.143. Tlius the image,
rainbow is seen as a colourful arc, with its inner edge 2. Adequate resolution. The resolution of an optical
violet and outer edge red in colour. instrument is its ability to resolve the images of two
The secondary rainbow is formed by the rays spaced objects so that they can be seen
which undergo two internal reflections and two separately. An optical instrument with high resolution
refractions before emerging from the water drops at reveals the finer details of the objects.
minimum deviation. Due to two internal reflections, 9.41 THE HUAAAN EYE*
the sequence of colour in secondary rainbow is
opposite to that in the primary rainbow. Here the inner 67. Describe the main parts of the human eye. Briefly
explain its working.
* Not included in the latest CBSE Syllabus.
RAY OPTICS AND OPTICAL INSTRUMENTS 9.81

Human Eye. It is the most valuable and sensitive 8. Aqueous humour and vitreous humour.
sense organ. It is a remarkable optical instrument. Aqueous humour is a salty fluid ()i =1.337) that fills the
Ciliary muscle space between the cornea and the eyelens. Vitreous
Sclerotic humour is a jelly like fluid (p =1.437) that fills the
Iris ^ Choroid space between the retina and the eyelens.
Retina
Action of the eye. The transparent structures like
Crystalline
/ lens 'Yellow
cornea, aqueous humour, eyelens and vitreous
spot humours together constitute a single converging lens.
Pupil
Comea- Blind spot
As the rays from an object enter the eye, they suffer
W, refractions on passing successively through these
Aqueous
Suspensory n Optic structures and get converged. A real and inverted
image is formed on the retina. The light sensitive cells
nerve
humour
ligaments

Vitreous of retina get activated and generate electrical signals


humour that are sent to the brain through the optic nerves. Our
brain translates the inverted image into an erect image.
Fig. 9.144 Structure of the human eye.
68. VWm/ do you mean by the term accommodation ?
Structure of the eye. As shown in Fig. 9.144, the Explain, how can the eye see objects at afr and near distances.
main parts of the human eye are as follows : Accommodation. Accommodation is the ability or
1. Sclerotic. It has a tough and opaque white covering, property of the eyelens due to lohich it can change its ciirva-
called sclerotic which protects and holds the eyeball. hire or focal length so that images of objects at various
2. Cornea. It is the transparent membrane on the
front portion of the eyeball through which light enters leng* « automatically changed with the
g^ help of ciliary muscles as follows :
3. Choroid. It is a black membrane below the Viewing far off objects.
sclerotic. It, absorbs
,
stray
I.- i
light and avoids
● iu
any blurring
u 11
mu^les are completely relaxed,
Its focal length is maximum (equal to distance between
of image due to multiple reflections m the eyeball. retina). The rays coming from the distant
4. Iris and pupil. Iris is an opaque circular diaphragm are parallel to each other and they are focussed
having a small central hole called the pupil. Under the ^he retina as shown in Fig. 9.145(a).
muscular action of the iris, the size of the pupil Eye
becomes smaller in bright light and larger in dim light.
5. Eyelens. It is a double convex lens situated R
A
behind the iris. It is composed of a fibrous, jelly like ●>

material. The lens is held in position by suspensory Parallel rays


V
ligaments and connected to the sclerotic by the ciliary From far point
muscles. By contracting or relaxing, the ciliary muscles at 11 =«

can change the shape or curvature of the eyelens and


hence change its focal length. This ability of the eyelens
to change its focal length is called accommodation. This
enables the eyelens to focus the images of objects at
different distances on the retina of the eye.
6. Retina. It is a delicate membrane of nerve fibres
on the imier side of the backwall of the eye. It contains
light sensitive cells called rods and cones. Rods are sensitive
to intensity of light while cones are sensitive to colours.
These cells change light energy into electrical signals Fig. 9.145 Accomodation of eyelens
which send message to the brain via the optic nerves. (fl) Focussing parallel rays from infinity i.e., far point
(b) Focussing rays from near point N.
7. Blind spot and yellow spot. In the region where
the optic nerve enters the eyeball, there are no rods and {b) Viezvwg nearby objects. When we look at a
cones. This region is totally insensitive to light and is nea
rby object, the ciliary muscles contract, the eyelens
called blind spot. Yellow spot has maximum concen- bulges out and becomes thick and its focal length is
tration of light sensitive cells. It is situated in the centre reduced. Tliis focusses the light from the nearby object
of the retina. on the retina, as shown in Fig. 9.145(fc).
■9.82 PHYSICS-XII

69. Define the following terms and give their values card-board, the two impressions merge and the bird
for a normal eye : {i) range of normal vision, (ii) least will appear to be inside the cage due to persistence of
distance of distinct vision, (Hi) near point of the eye, (iv) vision.
far point of the eye, and (v) power of accommodation.
(0 Range of normal vision. Due to accommodation 71. What are rods and cones ? How do they differ i
in

their functions ?
property of the lens, a normal eye can clearly see the
object situated any where between infinity and 25 cm
Rods. These are rod-shaped light sensitive cells of the
retina which are responsible for twilight (black-and-white)
from it. At distance less than 25 cm, the ciliary muscles vision. These cells are
cannot bulge the eyelens any more, the object cannot very sensitive to intensity of
be focussed on the retina and it appears blurred to the incident light, that is, the degree of brightness and
darkness. The rods cannot distinguish between
eye, as shown in Fig. 9.146. The distance between infinity various colours.
and 25 cm point is called the range of normal vision.
Cones. These are cone-shaped light sensitive cells of the

w
Eye retina which are responsible for colour vision. Different
cones respond selectively to different colours. Three
A types of cones viz. R-cones, G*cones and B-cones are
R
respectively sensitive to red, green and blue light.
O-

u < 25 cm
I

Flo When red light falls on the retina, it mainly activates


the R-cones than the other kinds of cones. However,

ee
cone becomes active only in bright light. That is why,

Fr
we cannot see colours in very dim light.
Fig. 9.146 Object O within 25 cm from the eye is
not focussed on retina and seen blurred,
For Your Knowledge
for
ur
(n) Least distance of distinct vision. The minimum > Cinematography. Cinematography works on the
distance from the eye, at which the eye can see the object principle of persistence of vision. If photographs of a
clearly and distinctly without any strain is called the least moving object are taken at intervals of about (l/24)th
ks

distance of distinct vision. It is denoted by the letter D. of a second and then projected on the screen at the
Yo

For a normal eye, its value is 25 cm. same rate, the discontinuous pictures merge or blend
oo

(in) Near point. The nearest point from the eye, at together to produce the impression of the moving
which an object can be seen clearly by the eye is called its object on the eyes.
B

near point. The near point of a normal eye is at a distance of > Colour blindness. A person who cannot distinguish
re

25 cm. between various colours but can see well otherwise, is said
to be colour blind. Colour blindness is due to the lack of
(iv) Far point. The farthest point from the eye, at
ou

which an object can be seen clearly by the eye is called either one type, two types or all the three types of
ad

cones in the retina of the eyes. This defect occurs by


the far point of the eye. Fora normal eye, the far point is at
Y

infinity. inheritance. That is colour blindness is a genetic


disorder which cannot be cured even today.
(t;) Power of accommodation. The power of '> Cataract. In old age, the crystalline lens of some
nd

accommodation of the eye is the maximum variation of


Re

people becomes hazy or even opaque due to the


its power for focussing on near and far (distant) development of membrane over it. This results in the
Fi

objects. For a normal eye, the power of accommodation is development of cataract, which causes a decrease or
about 4 dioptres.
loss of vision of the eye. The vision can be restored
70. What do you mean by persistence of vision ? Give after getting cataract surgery.
an example.
Persistence of vision. The impression or sensation 9.42 DEFECTS OF VISION AND THEIR
of an image on the retina remains (or persists) for CORRECTION’
about (l/16)th of a second even after the removal of the
object. The phenomenon of the continuation of the 72. Mention the common defects of vision of the
impression of an image on the retina for some time even after human eye.
the light from the object is cut off is called persistence of Defects of vision. A normal eye can see objects
vision. For example, if there be a picture of a bird on clearly at any distance between 25 cm and infinity from
one side of a piece of card-board and a picture of a cage Sometimes, a human eye gradually loses its
just on the opposite side, then on rapidly revolving the power of accommodation. Then we cannot see the
RAY OPTICS AND OPTICAL INSTRUMENTS 9.83

objects clearly. Our vision becomes defective. There are Correction of myopia. A myopic eye is corrected by
mainly four common defects of vision which can be using a concave lens of focal length equal to the distance of
corrected by the use of suitable eye glasses. These the far point F from the eye. This lens diverges the
defects are : parallel rays from distant object as if they are coming
1. Myopia or near-sightedness. from the far point F. Finally, the eyelens forms a clear
2. Hypermetropia or far-sightedness. image at the retina.
3. Presbyopia. 74. Hoiv can we determine the focal length and power
4. Astigmatism. of the concave lens required to correct a myopic eye ?
73. What is myopia or short-sightedness ? What is its Calculation of focal length and power of correc
cause ? How can it be remedied ? Explain by ray diagrams. ting lens in myopia. Let x be the distance of the actual
far point from the eye and hence from the concave lens
Myopia or short-sightedness. U is a vision defect in
which a person can see nearby objects clearly but cannot see placed close to the eye. The rays coming from infinity,
the distant objects clearly beyond a certain point. This after refraction through the concave lens, appear to
defect is common among children. come from the far point f.
Cause of myopia. This defect arises due to either of w = - CO, v = -x, f=7
the following two reasons : By lens formula.
(f) The eyeball gets elongated along its axis so that the 1=1 ^ ^ ^ 1
--+0=-
1

distance between the eyelens and the retina / ^ u -X -CO X X

becomes larger.
Required focal length, f = ~x
(ii) The focal length of the eyelens becomes too short due
to the excessive curvature of cornea. 1 1
Required power. F =-
As a result of the above causes, the parallel rays / X

coming from a distant object do not meet at the retina The negative sign shows that the correcting lens is a
but at a point in front of the retina, as shown in concave lens.
Fig. 9.147(a) and the distant object is not seen clearly.
75. What is hypermetropia or long-sightedness ?
The object has to be moved closer to the eye to a point f
What is its cause ? How can it be corrected ? Explain by
to focus it on the retina, as shown in Fig. 9.147(b). Thus,
ray diagrams.
the far point of a myopic eye is not at infinity but only a
few metres from the eye. Hypermetropia or long-sightedness. It is a vision
defect in which a person can see the distant objects clearly
Image formed but cannot see the nearby objects clearly.
in front of retina
Cause of hypermetropia. Tliis defect arises due to
R
either of the following two reasons :
Rays from a (i) The eyeball becomes too small along its axis so that
distant object / the distance between the eyelens and the retina
is reduced,

(ji) The focal length of the eyelens becomes too large


resulting in the low converging power of the
eyelens.
As a result of the above causes, the rays coming
F
from an object placed at 25 cm (normal near point)
Far point of from the eye meet at a point behind the retina, as
myopic eye
shown in Fig. 9.148(a). So the object is not seen clearly.
To focus the rays again on the retina, the object has
> to be moved away from the eyes to a distance greater
F- = than 25 cm, as shown in Fig. 9.148(1?)- Thus, the near
point of the eye is not at 25 cm but it has shifted to N' at a
Concave lens causes rays L distance greater than 25 cm from the eyes.
to diverge as if coming from F
Correction of hypermetropia. A hypermetropic eye is
Fig. 9.147 Myopia and its correction. corrected by using a convex lens of suitable focal length.
PHYSICS-XIl

77. What is presbyopia ? How does it differ from


hypermetropia ?
N'
Normal
'-’■■I Presbyopia. This defect is similar to hypermetropia
Image a person having this defect cannot see nearby
l.e.,
near point formed
behind
objects distinctly, but can see distant objects without
the retina any difficulty. This defect differs from hypermetropia
in the cause by which it is produced. It usually occurs in
elderly persons, Due to the stiffening of the ciliary
More than 25 muscles, the eyelens loses flexibility and hence the
/
accommodating power of the eyelens decreases. Like
N'
\
I hypermetropia, this defect can be corrected by using a
Near point of
convex lens of suitable focal length.

w
hypermetropic eye 78. What is astigmatism ? How is it caused ? How is
(b) it corrected ?

Astigmatism. U is a defect of vision in which a person


R

Flo
cannot simultaneously see both the horizontal and vertical
views of an object with the same clarity. This defect
occur alongwith myopia or hypermetropia.
can

ee
N' /
Cause of astigmatism. This defect occurs when the

Fr
cornea is not perfectly spherical in shape. It may have a
Convex lens converges the rays ■A' large curvature in the vertical plane than in the
as if they are coming from N' (c) horizontal plane or vice versa. If one looks at a wire
for
ur
mesh with such a defect in the eyelens, focussing in the
Fig. 9.148 Hypermetropia and its correction. vertical plane may not be as sharp as in the horizontal
plane or vice versa. Astigmatism results in lines in one
ks

This lens converges the rays such that the rays coming t^irection well focussed while those in perpendicular
Yo

from normal near point N appear to come after direction will be distorted or curved, as shown in
oo

refraction, from near point N‘ of the defected eye. That ^*8- 9.149(u).
is a virtual image of the object placed at N is formed at
B

N'. Then the eyelens forms a clear image at the retina,


re

as shown in Fig. 9.148(c).


76. How can we determine the focal length and power
ou
ad

of the convex lens required to correct a hypermetropic


eye ?
Y

Calculation of focal length and power of correcting Image as formed


lens in hypermetropia. Refer to Fig. 9.148(c). Let y = on the retina
nd

distance of the near point N' from the defective eye.


Re

Now the near point N of the normal eye is at distance


Fi

D=25cm. The object placed at N forms its virtual


image at N' due to the convex lens.
Cylindrical
.-. u=-D, v = -\j, /=? lens
By lens formula Image as formed
ib)
1 1 1 1 on the retina
1 _y~D
f V u
-y - D yD Fig. 9.149 (a) Astigmatism and (fc) its correction.
yo
Required focal length. /=
y-D Correction of astigmatism. Astigmatism can be
corrected by a lens whose one surface is cylindrical.
Required power. P-i-y-p Such a surface focusses rays in one plane but not in the
/ yD
perpendicular plane. By suitably choosing the radius
As y > D, so both / and D are positive. That is the of curvature and axis direction of the cylindrical
correcting lens must be a convex lens. surface, astigmatism can be corrected.
RAY OPTICS AND OPTICAL INSTRUMENTS

From thin lens formula,


Examples based on
1
Defects oF Vision 1=1+1=^ +
V
/ u -40 - 00

Formulae Used
1 1
-0 = -
1. Correction of myopia or short sightedness. A -40 40
concave lens of focal length/equal to the distance
X of the far point from the defective eye is used. or = - 40 cm

/ =- X and P= -— Thus the far point of the eye is at 40 cm from the


X
eye.
2. Correction of hypermetropia or long sighted
ness. A convex lens of focal length/is used, where Example 115. (a) The far point of a myopic person js80 cm
-D in front of the eye. What is the power of the lens required to
/=^ enable him to see very distant objects clearly ?
y-D
Here (b) In what way does the corrective lens help the person
above ? Does the lens magnify very distant objects ? Explain
y = distance of the near point from the defective [CBSE D 09C upto part (fc)]
eye and carefully.
D - the least distance of distinct vision. (c) The person above prefers to remove his spectacles
while reading a book. Explain why. [NCERT]
Units Used
Solution, (a) The remedial lens should make the
Distances ,/ x, y and D are in metre, and power P
objects at infinity appear at the far point.
in dioptre.
For objects at infinity, » = - co
Example 113. What focal length should the reading Far point distance of the defective eye, u = - 80 cm
spectacles have for a person for whom the least distance of By thin lens formula,
distinct vision is 50 cm ? [NCERT]
1 1 1
Solution. The reading matter placed at 25 cm from / u

the corrective lens must produce the virtual image at 1 1 1 1


50 cm. Tlierefore, __ + 0=-
-80 - GO 80 80
u = - 25 cm, i? = - 50 cm
or / = - 80 cm = - 0.80 m
By thin lens formula,
1 1
1 1 1 = -1.25 D.
Power, p = -

/ u -50 -25 / -0.80 m


_-1+2_ 1 (b) No, the concave lens does not magnify the very
” 50 distant objects. In fact, it reduces the size of the object
or / = + 50 cm (image distance is less than object distance), but the
angle subtended by the distant object at the eye is the
The positive sign shows that the corrective lens same as the angle subtended by the image (on the far
must be a convex lens of focal length 50 cm. point) at the eye. The eye is able to see distant objects
Example 4. A per son wears glasses of power-2.5 D.Is not because the corrective lens magnifies the object,
the person far-sighted or near-sighted ? What is the far point but because it brings the object {i.e., it produces virtual
of the person zvithout glasses ? image of the object) at the far point of the eye which
Solution. Here, P =-2.5 D then can be focused by the eye-lens on the retina,
(c) The myopic person may have a normal near
Negative power shows tliat the lens is concave, so point i.e., about 25 cm (or even less). In order to read a
the person is near-sighted. book with his spectacles (for distant vision), he must
1 2
m = — m=-40cm keep the book at a greater distance than 25 cm so that
-2.5 5
the image of the book by the concave lens is produced
not closer than 25 cm. The angular size of the book (or
The object placed at infinity from the corrective lens
its image) at the grater distance is evidently less than
must produce the virtual image at the far point.
Therefore,
the angular size when the book is placed at 25 cm and
no spectacles are used. Hence, the person prefers to
v = ?
H = -00, remove his spectacles while reading.
9.86 PHYSICS-Xll

Example 116. (a) The near


point of a hypermetropic Solution, (fl) Power of the distance-viewing part of
person is 75 on from the eye. What is the power of the lens the lens,
required to enable him to read clearly a book held at 25 cm
from the eye ? Pj =-5.5 D
Focal length of this part,
(b) In what way does the corrective lens help the person 1 1
above ? Does the lens tnagnify objects held near the eye ? m = -18.73 cm.
-5.5
(c) The person above prefers to remove his spectacles
while looking at the sky. Explain why. [NCERT] (b) As power of the near-vision part is measured
Solution, (fl) The book placed at 25 cm from the to the main part of the lens of power - 5.5 D, so
we use
corrective lens must form the virtual image at 75 cm.
Therefore, P,-fP2 = P
-5.5 + P2 = +1.5

w
w = -25cm, u = -75cm or

By thin lens formula, or


P2 = + 6.5 D
1 1 1 Focal length of near-vision part,
/
1
W

1 _ 2
Flo f2=~
Pj
1

+ 6.5
m = +15.4 cm.

ee
-75 -25 75 ~75 (
roblems For Practice

Fr
75 75
or cm = m
200
1. The far point of a myopic person is 150 cm in front
P = l=^ = +2.67D of the eye. Calculate the focal length and the power
for
ur
/ 75 of the lens required to enable him to see distant
Thus the corrective lens must be a convex lens of objects clearly. (Ans. - 150 cm, - 0.67 D)
power + 2.67 dioptres. 2. A person can see clearly up to 3 metre only. Pres
ks

cribe a lens for him so that he can see clearly up to


(b) The corrective lens produces a virtual image (at
Yo

12 metre. (Ans. Concave lens, / = - 4 m)


75 cm) of an object at 25 cm. The angular size of this
oo

image is the same as that of the object. In this sense 3. The near point of hypermetropic persion is 50 cm
from the eye. Wliat is the power of the lens required
B

the lens does not magnify the object but merely


brings the object to the near point of the eye, which to enable the person to read clearly a book held at
re

then gets focused by the eye lens on the retina. 25 cm from the eye ? [CBSE OD 09]

However, the angular size is greater than that of the (Ans. + 2 D)


ou
ad

same object at the near point (75 cm) viewed without 4. A short-sighted person can see most clearly at a
the spectacles, distance of 15 cm acquires spectacles enabling him
Y

(c) The person prefers to remove his spectacles to see clearly objects at a distance of 60 cm.
while looking at the sky, because a hypermetropic eye Calculate the focal length of tlie lens and power of
nd

the lens. (Ans. - 20 cm, - 5 D)


Re

may have normal far point i.e., it may have enough


converging power to focus parallel rays from infinity HINTS
Fi

on the retina of the shortened eyeball. Wearing 1. Here, u = -oo, v = - 150 cm


spectacles of converging lenses (used for near vision)
will amount to more converging power than needed 1 1 1

for parallel rays. The result will be that distant objects / V li

may get focused in-front of the retina (like a myopic 1 1


eye) and will appear blurred. -150 - X

Example M7. A 52-year old near-sighted person wears 1


eye-glass with a power of- 5.5 dioptres for distance viewing. 150 150
His doctor prescribes a correction 0/4-1.5 dioptres in the
near vision section of his bi-focals. This is measured relative / = -150 cm
to the main part of the lens, (a) What is the focal length of his P=
1 1

distance-viezving part of the lens ? (b) What is the focal /(in m) - 1.50m
length of the near-vision section of the lens ? = - 0.67 D.
RAY OPTICS AND OPTICAL INSTRUMENTS 9.81

2. Here, u = -12m, u = -3m, / = ? the image and the object at the eye, when both are at the least
1=1 ^ distance of distinct vision from the eye. Thus,
f~v ii
Magnifying power,
1 1 1
Angle subtended by the image at the least
3 12 4 distance of distinct vision
m-
or
/ = - 4 m. Angle subtended by the object at the least
distance of distinct vision
3. The book placed at 25 cm from the corrective lens
must form the virtual image at 50 cm. As the eye is held close to the lens, the angles
M = - 25 cm, u = - 50 cm
subtended at the lens may be taken to be the angles
i = 2_2 1. subtended at the eye. The image A! B' is formed at the
f V u ”50*^ 25 50 least distance of distinct vision 'D'. Let ZA’OB'=p.
or / = 50 cm = 0.50 m Imagine the object AB to be displaced to position A"B'
at distance D from the lens. Let ZA"OB' = (X. Then
1
Power P- — = 2D
magnifying power,
f 0.50 m
B tan S
Thus, the corrective lens must be a converging lens m = —= [v a, pare small angles]
a tan a
of power 2 D.
AB/OB AE/OB
[V A!'B = AB]
9.43 SIMPLE MICROSCOPE A" B'/OB' ABIOB'

79. What is a simple microscope 7 Give its working OB' -D


principle. Write expressions for its magnifying power OB ~x

when it forms final image at the least distance of distinct D


vision and at infinity. or m = —
x

Simple microscope. A simple microscope or a magnifying Let /be the focal length of the lens. As the image is
glass is just a convex lens of short focal length, held close to formed at the least distance of distinct vision from the
the eye. lens, so
Working principle: When the final image is formed v = -D
at the least distance of distinct vision. Wlien an object
Using thin lens formula,
AB is placed between the focus f and optical centre O 1 1 1
of a convex lens ; a virtual, erect and magnified image
A' B' is formed on the same side of the lens as the
v u /
1 1 1
object. Since a normal eye can see an object clearly at vve get,
the least distance of distinct vision D (=25 cm), the
-D /
position of the lens is so adjusted that the final image is i = J- 1
or
formed at the distance D from the lens, as shown in
X f
Fig. 9.150.
D D
or — =1 + —
A' X /
D
A m =l +—
f
Thus shorter the focal length of the convex lens, the
greater is its magnifying power.
Working principle: When the final image is formed
at infinity. When we see an image at the near point, it
causes some strain in the eye. Often the object is placed
Fig. 9.150 A simple microscope with the eye
focussed at the near point.
at the focus of the convex lens, so that parallel rays
enter the eye, as shown in Fig. 9.151(fl). The image is
Magnifying power. The magnifying power of a simple formed at infmity, which is more suitable and
microscope is defined as the ratio of the angles subtended by comfortable for viewing by the relaxed eye.
PHYSICS-XII

For Your Knowledge


> Least distance of distinct vision (D). The minimum
distance from the eye, at which the eye can see the
"-.A : objects clearly and distinctly without any strain is
called the least distance of distinct vision. For a
normal eye, its value is 25 cm.
B
> Near point. The nearest point from the eye, at which
an object can be clearly seen by the eye is called its
(a) near point. The near point of a normal eye is at a
distance of 25 cm.
Eye focussed
A at infinity > Far point. The farthest point from the eye, at which an
h
object can be seen dearly by the eye, is called the far
a
point of the eye. For a normal eye, the far point is at

w
B
D infinity.
ib)
> Accommodation. It is the ability of the eyelens due to
Fig. 9.151 (c) With object at f, image is at infinity, which it can change its focal length so that images of
(i) Object at the near point.

Magnifying power. It is defined as the ratio of the Flo objects at various distances can be formed on the
same retina.

> Power of accommodation. The power of accommo

ee
angle formed by the image (when situated at infinity) at the dation of the eye is the maximum variation of its power
for focussing on near and far objects. For a normal

Fr
eye to the angleformed by the object at the eye, when situated eye, the power of accommodation is about 4 dioptres.
at the least distance of distinct vision.
> The magnifying power is expressed with a unit X. So
B tan 6
if a magnifying glass produces an angular magni
for
ur
m = -i- = [a, p are small]
a tan a fication of 10, it is called a 10 X magnifier.
> A simple microscope has a limited maximum magni
From Fig. 9.151(a), fication of about 10, for realistic focal lengths. For
ks

much larger magnifications, we use two convex lenses,


Yo

tan p = — one enhancing (compounding) the effect of the other.


/
oo

This is known as the compound microscope.


From Fig. 9.151(b),
B

● ●

h Examples based on
re

tan a = —
D Simple Microscope
Formulae Used
h/f
ou
ad

m =
h/D
1. When the final image is formed at the least distance
of distinct vision, the magnifying power is
Y

D D
or m = — m=l + —
/ /
nd
Re

This magnification is one less than the magni 2. When the final image is formed at infinity, the
D
fication when the image is formed at the near point. magnifying power is m =
Fi

/
But viewing is more comfortable when the eye is Units Used
focussed at infinity.
Magnification m has no units.
Uses of simple microscopes :
D = 25 cm, for a normal eye.
1. Watch makers and jewellers use a magni-
fying glass for having a magnified view of Example 118. /I thin convex lens of focal length 5 cm is
the small parts of watches and the fine as a simple microscope by a person with normal near
jewellery work. point (25 cm). Find the magnifying power of the microscope
2. In magrufying the printed letters in a book, image is formed at (i) infinity, and (ii) the least
textures of fibres or threads of a cloth, distance of distinct vision. [NCERT; ISCE 22]
engravings, details of stamp, etc. Solution. Here / = 5 cm, D = 25 cm
3. Magnifying glass is used in science laboratories
for reading vernier scales, etc. (i)m = -5/ = ^5 = 5. (ii)m =-L + ~=l
/
+ —5 = 6.
RAY OPTICS AND OPTICAL INSTRUMENTS

Example 119. A simple microscope is a combination of (ii) Angular magnification,


25
two lenses, in contact, of powers +15 D and + 5 D. m = 1 + — =1 + =1+4=5
Calculate the magnifying poioer of the microscope, if the / 6.25

final image is formed at 25 cmfrom the eye. When the final image is formed at infinity, the
Solution. Power of combination,
angular magnification becomes
P=Pj + P2=15 + 5 = + 20D m = —
D 25
= 4.
Focal length of combination, f 6.25
/^ = !P = 20 m = 5 cm
1
Example 123. A man with normal near point {25 cm)
reads a book with small print using a magnifying glass : a
Magnifying power, thin convex lens of focal length 5 cm.
1 25 ^
m = l + — = 1 + — = 6. (/) What is the closest and the farthest distajtce at which
/ 5
he can read the book when viewing through the magnifying
Example 120. An object is to be seen through a simple ^
microscope of power 10 D. Where should the object be placed (ii) What is the maximum and the minimum angular
so as to produce maximum angular magnification ? The magnification (magnifying power) possible using the above
least distance for distinct vision is 25 cm simple microscope ? [NCERTl

Solution. Angular magnification is maximum Solution. (0 For the closest distance :


when the final image is formed at the near point. V = -25 cm, / = 5 cm, u =7

.●. 17 = -25 cm, /■ = —= —m=10cm 1 1 1


P 10 As
1 1 1 1 1 7
V u
/
Now
u V
/ 25 10 50 1 1 1 1 1 _-1-5 -6
u 17 /“-25 5 " 25 25
or « = -50/7 = -7.1 cm.
25
or u = cm = -4.2 cm
Example 121. A simple microscope is rated 5X for a 6
normal relaxed eye. What will be its magnifying power for a This is the closest distance at which the man can
relaxed farsighted eye whose near point is 40 cm ? read the book.
Solution. For normal eye \ D =25 cm, m = 5
D
For the farthest image :
As m=— .-. 5=^ or
/ = 5 cm v = oo, / = 5 cm , u =?
/ /
1 1 1 1
For relaxed farsighted eye: U = 40 cm, / = 5 cm u
/ 00 5 5 5
m-
u = -5 cm.
/ 5 or

This is the farthest distance at which the man can


Thus the magnifying power of the simple micro
read the book.
scope is 8 X in the second case.
Example ^22. A converging lens offocal length6.25 cmis
(ii) Maximum angular magnification
D 25
used as a magnifying glass. If the near point of the observer = 6
is 25 cm from the eye and the lens is held close to the eye, "min 25/6
calculate (i) the distance of the object from the lens and Minimum angular magnification
(ii) the angular magnification.
D
Find the angular magnification, when the final image is u 5
formed at infinity. [ISCE 931 max

Solution. Here /= 6.25 cm, i7=-D = -25cm Example 124. A figure divided into squares each of size
1 mn? is being vieived at a distance of 9 cm through a
(i) Using thin lens formula,
magnifying glass (a converging lens of focal length 10 cm)
i= = L held close to the eye.
u~ V f~-25 6.25 (i) What is the magnification (image size/object size)
= _X_± = _i_=_l or u = -5 cm.
produced by the lens ? How much is the area of each
25 25 ” 25 " 5 square in the virtual image ?
PHYSICS-XII

(zi) What is the angular magnification (magnifying power) So lens should be held 7.14 cm away from the
of the lens ? figure.
{Hi) Is the magnification in (i) equal to the magnifying (») Magnitude of magnification,
power in (ii) ? Explain. [NCERT ; CBSE D 05] V 25
= 3.5
Solution, (i) Here, area of each square (or object)
m =
Ml 50/7
= 1 mm^
D _ 25
M = -9cm, / = + 10cm (Hi) Magnifying power = = 3.5
urSO/7
1 1 1
As
Yes, the magnifying power is equal to the
V u /
magnitude of magnification because image is formed
1_ 1 ^1_ 1 1 _9-10_ 1 at the least distance of distinct vision.
V / h10 9~90~W Example 126. What should be the distance between the

w
or u = - 90 cm object in Example 125 and the magnifying glass if the
Magnitude of magnification. virtual image of each square in the figure is to have an area
V 90 6.25 mtt^ ? Would you be able to see the squares distinctly
m =
u 9
= 10

Area of each square in the virtual image Flo with your eyes very close to the magnifier ?
Solution. Here the magnification in area
[NCERT]

ee
6.25 mm^
= (10)^ X 1 = 100 mm^ = 1 cm^. 1 mm^
=6.25

Fr
(ii) Magnifying power,
D 25 Linear magnification, m = V6.25 = 2.5
m = — = 2.8 V
9 As m = — V = mu =2.5 M
for
u
ur
u

(Hi) No. Magnification of an image by a lens and VT


Now
11 1

angular magnification (or magnifying power) of an V u


/
ks

optical instrument are two separate things. The latter is 1 1 _ 1 1-2.5 _ 1


Yo

the ratio of the angular size of the object (which is or


2.5 m ”10
oo

2.5 M M 10
equal to the angular size of the image even if the image
is magnified) to the angular size of the object if placed or 2.5 m = -1.5x10
B

at the near point (25 cm). Thus magnification 1.5x10


re

or H = - = -6 cm
magnitude is —M and magnifying power is —. Only 2.5

Hence M = 2.5 M = 2.5 X ( - 6) = -15 cm


ou

when the image is located at the near point |u |= 25 cm,


ad

the two quantities are equal. As the virtual image is closer than the normal near
point (25 cm), it cannot be seen by the eye distinctly.
Y

Example 125. (i) At what distance should the lens be held


from the figure in Example 124 in order to view the squares roblems For Practice
nd

distinctly with the maximum possible magnifying power ?


Re

1.
What must be the focal length of a lens used as
(ii) What is the magnification (image size/object size) in simple microscope of magnifying power 26 ? The
Fi

this case ?
final image is formed at the distance of distinct
(Hi) Is the magnification equal to magnifying power in vision. (Ans. 1 cm)
this case ? Explain. [NCERT] 2.
A converging lens of power 100 dioptre is used as a
Solution, (i) Maximum magnifying power is obtained simple microscope. What is its magnifying power,
when the image is formed at the near point (25 cm). if the distance of distinct vision is 25 cm ?

.'. M = -25cm, / = + 10cm, M = ? (Ans. 26)


As 1 1 ^ J. 3.
A converging lens of focal length 6.25 cm is used as
V
w / a magnifying glass. If the near point of the observer
is 25 cm from the eye and the lens is held close to
i-i_1 1 1 _“2“5_-7
the eye, calculate (i) the distance of the object from
M v f~ 25 10 “ 50 “so the lens and (ii) the angular magnification. Find the
50
or M = - = -7,14 cm angular magnification when the final image is
7 formed at infinity. [Ans. (z) 5 cm {ii) 5, 4]
RAY OPTICS AND OPTICAL INSTRUMENTS 9.91

4. The magnifying glass L made of combination of g COMPOUND MICROSCOPE


lenses of power + 20 D and - 4 D. If the distance of
distinct vision is 25 cm, calculate the size of an 80. With the help of a ray diagram, explain the
object 2 cm high seen through the magnifying glass, construction and working of a compound microscope.
Write an expression for its magnifying power.
(Ans. 10 cm)
5. A magnifying glass is a combination of a convex Compound microscope. A compound microscope is an
lens of focal length 5 cm and a concave lens of optical device used to see magnified images of tiny
power -5D. If the distance of distinct vision is ot>jects. A good quality compound microscope can
20 cm, calculate the magnifying power of the produce magnification of the order of 1000.
magnifying glass. (Ans. 4) Construction. It consists of two convex lenses of

6. Magnifying power of a simple microscope A is 1.25 short focal length, arranged co*axially at the ends of
less than that of a simple microscope B. If the power sliding metal tubes,
of the lens used in B is 25 D, find the power of lens 1. Objective. It is a convex lens of very short focal
used in A. Given that distance of distinct vision is
length /q and small aperture. It is positioned near the
25 cm. (Ans. 20 D) object to be magnified.
7. A child has a near point at 10 cm. What is the 2. Eyepiece or ocular. It is a convex lens of compara-
maximum angular magnification the child can have tively larger focal length f and larger aperture than
with a convex lens of focal length 10 cm ? (Ans.2) the objective (/^ > ff}. It is positioned near the eye for
HINTS viewing the final image.
The distance between the two lenses can be varied
1. Here m = 26, D = 25cm, /=?
D 25 by using rack and pinion arrangement.
As m= l + 26=1+— or /=lcm.
f f Working.(fl) When the final image is formed at the
D least distance of distinct vision. The object AB to be
2. Use / = — and m= 1+
f viewed is placed at distance Uq, slightly larger than the
focal length /q of the objective O. The objective forms a
3. (/) Here/ = +6.25 cm, u = -'25cm real, inverted and magnified image A B\ of the object
1 1_J. 1 1 AB on the other side of the lens 0, as shown in
u I' / 25 6,25 Fig. 9.152. The separation betw'een the objective O and
1.25 1 the eyepiece E, is so adjusted that the image A' B' lies
or II = - 5 cm.
6.25 5
within the focal length of the eyepiece. The image
A' B' acts as an object for the eyepiece which essentially
(if) Angular magnification, hi = —II = —
5
= 5. acts like a simple microscope. The eyepiece E forms a
virtual and magnified final image A" B" of the object
The final image is formed at infinity when « = /. AB, Clearly, the final image A" B" is inverted with
D 25 respect to the object AB.
m - — = 4.
/ 6.25
4, Power of combination, P=I^ + I| = 20-4 = 16D A
«Q
A
Focal length of combination, B h B'

1 1 25 B ,F0 W E
f= — = — m = — cm
P 16 4 -'A'
Objective
D 25 k
m= 1+ — ~l + = 5 Eyepiece
/ 25/4

Size of image, /^=h7x)Ij=5x2 = 10 cm.


7. For maximum angular magnification, A" I D

- 10 cm A

1 _ 1 _ J_ _ _ J_ _ J_ _ _ 1 oT
u V f 10 10 5 B D

« = - 5 cm
or
Fig. 9.152 Compound Microscope, final image at D.
V
m = -
II -5
Magnifying power, jhg magnifying power of a compound
microscope is defined as the ratio of the angle subtended at
9S2 PHYSICS-Xll

the eye by the final virtual image to the angle subtended at Obviously, magnifying power of the compound
the eye by the object, when both are at the least distance of microscope is large when both and f are small.
distinct vision from the eye. ^
P _ tanp _h'/u^ h' D For Your Knowledge
a tan a h / D h‘ ^ In a compound microscope, the objective is a convex
Here h' _ Vo lens of short focal length and small aperture, while
u
0
the eyepiece is a convex lens of short focal length and
large aperture.
As the eyepiece acts as a simple microscope, so In actual practice, each of the objective and the
D D
= —=1 + — eyepiece consists of combination of lenses. To
eliminate chromatic aberration, an objective consists
D of two lenses in contact. To minimise chromatic and
m=^ 1+— spherical aberrations, an eyepiece consists of two

w
e y
lenses separated by a certain distance.
As the object AB is placed close to the focus Fq of ^ In a compound microscope, the objective and the eyepiece
the objective, therefore, Mq - - /g are placed afixed distance apart. For focussing on an object,
Also image A' B' is formed close to the eyelens whose
focal length is short, therefore Ug - L=the length of the
microscope tube or the distance between the two lenses Flo the distance of the objective from that object is changed
with the help of a rack and pinion arrangement.
^ For large magnifying power, both and f have to be

ee
V
o_ I small. Also, f is taken larger than /g so as to increase

Fr
u
0 -fo the field of view of the microscope.
The visibility and quality of the image can be
m = -
[for final image at D] improved by illuminating the object and by using oil
for
f0
ur
V ey immersion objective.
When the final image is formed at infinity. jTjg ^ When the final image is formed at the least distance Dof
magnifying power of a compound microscope is defined as distinct vision, the length of the compound microscope,
ks

the ratio of the angle subtended at the eye by the final image
Yo

formed at infinity to the angle subtended (at the unaided eye) ^ When the final image is formed at infinity, the length
oo

by the object. When the image A' B lies at the focus of of the compound microscope, L = Ug + f.
the eyepiece i.e., u^ = f^, the image A” B" is formed at
B

infinity, as shown in Fig. 9.153. Examples based on


re

Eyepiece Compound Microscope


Formulae Used
ou
ad

fo-
1. Magnifying power, m = x
Y

2. When the final image is formed at the least


B ,F O 'E distance of distinct vision.
nd

V. D L D
Re

Objective m=^ 1+ 1+ —
“o I ./ fo . fey
'I
/A' ey
Fi

3. When the final image is formed at infinity,


Vr.0 D L D
m- —

^ fe fo /e
Fig. 5 Compound microscope, final image at co. Units Used
h' L
Magnification due to objective, 10^= — = The distances u^ , , Vq , , D and L are all in
^ ~fo metre or cm and magnification m has no units.
D D = 25 cm, for a normal eye.
Angular magnification due to eyepiece, mg= —
^ 27. ^ compound microscope with an objecti:: ive

Total magnification when the final image is formed of 1.0 cmfocal length and an eyepiece of 2.0 cm focal length
at mfiruty, L D
^ length of 20 cm Calculate the magnifying power
m=m^xm^ = ~x~ of the microscope, if the final image is formed at the near
JO h point of the eye. ICBSE D 04]
RAY OPTICS AND OPTICAL INSTRUMENTS 9.93i

Solution. Here /q =1.0 cm, f^ =2.0 cm, L = 20 cm, Example 130. ii) Draw a labelled ray diagram of a
D=25 cm compound microscope, showing the formation of image at
the near point of the eye. (ii) A compound microscope uses an
When the final image is formed at the near point of
objective lens offocal length 4 an and eyelens offocal length
the eye, the magnifying power is
10 cm An object is placed at 6 cm from the objective lens,
m-
(a) Calculate magnifying power of the compound micro
f0 V fe J
scope, if the final image is formed at the near point.
(b) Calculate the length of the compound microscope
20 25^ also. [CBSE OD 06C, 19]
1 + — =20x13.5 =270.
1.0 2 .
Solution, (i) For ray diagram, see Fig. 9.145
Example 128. You are given two converging lenses of
(ii) Here /g = 4 cm, f^= 10 cm,
focal lengths 1.25 cm and 5 cm to design a compound
microscope. If it is desired to have a magnification of 30, find Mg = - 6 cm, = - D = -25 cm
out separation between the objective and the eyepiece. 1 1 1
[CBSE OD 15] (a) As
/o V
0
u
0
Solution. Here /g = 1.25 cm, /^ = 5 cm, D = 25 cm.
m = 30 1111 J_^J_
When the final image is formed at the near point of
V
0 /o «o ^ -6 12

the eye, the magnifying power of the compound or = +12 cm


microscope is
Magnifying power of the compound microscope is
I D
m =— 1+— V D 12 25 ^
0
/o m = 1 + — 1 + — =7.
u
0 fe j 6 lOJ

or 30 = 7
1.25 5J ib)
fe -25 10 50

^ 30x1.25 = 6.25 cm.


or
50
= -7.14 cm
6
7

Example 129.^„ optical instrument uses a lens of power Length of the compound microscope is
100 D for objective lens and 50 D for its eyepiece. When the
tube length is kept at 25 cm, the final image is formed at L = Ug +1 uj = 12 + 7.14 = 19.14 cm
infinity. Example 131. The total magnification produced by a
(fl) Identify the optical instrument, compound microscope is 20, while that produced by the
(b) Calculate the magnification produced by the ^^i^ce alone is 5. When the microscope is focussed on a
instrument. [CBSEE D 20] certain object, the distance between objective and eyepiece is

Solution. 14 cm Find the focal length of objective and eyepiece^


1 distance of distinct vision is 20 cm.
m = 1 cm ; Solution. Here
100
m = 20, = 5, D = 20 cm, = - 20 cm
. 1 1
/g=—=—m=2cm
50
m
^0
5
As both the objective and the eyepiece are convex
lenses of short focal lengths, so the given optical As the eyepiece acts as a simple microscope, so
instrument is a compound microscope. D
= 1 + —
(ii) L = 25 cm, D = 25 cm, =lcm, /g =2 cm
When the final image is formed at infinity, the 20
magnification produced is or 5=1 +

L D 25 25
m = — X — = — x — = 312.5.
/o 1 2 /e = 5 cm
. : -r PHYSICS-XII

Also, HINTS
m,=
D 25
-20
1. For simple microscope, m = l + —= 1+ — = 6
or 5 = / 5

For compound microscope, = 6, = 40


or = - 4 cm = 40 X 6 = 240,
\

Distance between the objective and the eyepiece 2. Use m = mQ‘xm^.=niQ 1 + — .


* 14 cm cj

or =14
3, Here/q= 4 cm, /g=6cm, D = 25cm
or
4 + Uo = 14 or Vr.0 = 10 cm
For objective lens, +— ^
10 V.
/o
Now, => -4 = 1- — 0

/o /o Now, WQ=-6cm, /Q=+4cm

w
/o=2cm. -J_ + A = J_
-6 V.
0
+ 4
roblems For Practice

A convex lens of focal length 5 cm is used as a


simple microscope. What is its magnifying power,
Flo or 1 _ 1
0
4
1-3-2^ 1
6" 12 “ 12

ee
or Vn0 - 12 cm
if final image is formed at the distance of distinct V. D 12 25
vision i.e. 25 cm ? If it is used as an eyepiece in a 0

Fr
m =
1+^ 1 + —
compound microscope with objective of magni L /j 6 6

fying power 40, what is the magnifying power of = 2x ^^31 -


— =10.33.
the compound microscope ? (Ans. 6, 240)
for
6 3
ur
' A compound microscope has a magnification of 30. 4. Here /q= 1.25 cm, /g=5cm, m = 30
The focal length of its eyepiece is 5 cm. Assuming In normal adjustment, magnification produced by
the final image to be formed at least distance of
ks

the eyepiece.
distinct vision (25 cm), calculate the magnification
Yo

D 25
produced by the objective. (Ans. 5) = — =+— = 5
oo

The focal lengths of the objective and eye-piece of a


Now, m = wIq xm^ 30 = /tIq x5
B

compound microscope are 4 cm and 6 cm


respectively. If any object is placed at a distance of
or
"b = 6
re

6 cm from the objective, what is the magnification As real image is formed by the objective,
produced by the microscope ? Distance of the or
ou

distinct vision = 25 cm. (Ans. 10.33)


ad

: The focal lengths of the objective and eyepiece of a


Using thin lens formula for the objective,
Y

microscope are 1.25 cm and 5 cm respectively. Find 1


the position of the object relative to the objective in
order to obtain an angular magnification of 30 in
V.0
/o
nd
Re

normal adjustment. [CBSE D 12 ; OD 20] 1 _ 1 7 _ 1


or or
(Ans. - 1.46 cm)
6uq ” 1.25
Fi

-6iio 1.25
: The focal lengths of the objective and the eyepiece 7x1.25
= -1.46 cm
of a microscope are 2 cm and 5 cm respectively and
or
6
the distance between them is 20 cm. Find the
distance of the object from the objective when the Thus the object should be held at 1.46 cm in front of
final image seen by the eye is 25 cm from the the objective lens.
eyepiece. Also find the magnifying power. 5. Proceed as in Exercise 9.11 on page 9.133.
(/\r.'. Uq =- 2.3cm, m = 41.^
Q45 TELESCOPE
' A >.oqr loun^ n c osio,>e is adc ●g . I t, .r.
focal length 10 mm as objective and another lens of 81. Whui is a telescope 7 What are the different types
focal length 15 mm as eyepiece. An object is held at of telescopes commonly used ?
1.1 cm from the objective and final image is formed Telescope. A telescope i? an optical device which enables
at infinity. Calculate distance between objective us to see distant objects clearly. Il provides an angular
and eyepiece. (Ans. 12.5 cm) magnification of the distant objects.
RAY OPTICS AND OPTICAL INSTRUMENTS
?
1

Different types of telescope. Broadly, the tele- object falls on the objective at some angle a. The
scopes can be divided into two categories : objective focusses the beam in its focal plane and forms
1. Refracting telescopes. These make use of lenses to ^ averted and diminished image AB'. Ms image
view distant objects. These are of two types : B' acts as an object for tire eyepiece. The distance of
the eyepiece is so adjusted that the image A B lies
{a) Astronomical telescope. It is used to^e hea-
venly objects like the sun, stars, planets, etc. The final ^
magnifies this
^ g„ ^jg^ified and
image formed is inverted one \vhich is immaterial in inverted with respect to the object. The final image is
the case of heavenly bodies because of their round shape. seen distinctly by the eye at the least distance of
{b) Terrestrial telescope. It is used to see distant distinct vision,
objects on the surface of the earth. The final image
formed is erect one. This is an essential condition of Magnifying power. TJte magnifying power of a
^ c n telescope is defined as the ratio of the angle subtended at the
viewmg the objects on earth's surface correctly. ,, r- , ■ ,■ j , fj-x rj-x- x
° ^ eye by the final mage formed at the least distance of distinct
2.Re(lecting telescopes, mse make use of converging subtended at the eye by the object at
mirrors to view the distant objects. For example, Newtoraan ^
and Cassegrain telescopes.
As the object is very far off, the angle subtended by
9.46 ASTRONOMICAL TELESCOPE it at the eye is practically equal to the angle a
82. What is an astronomical telescope ? Give its subtended by it at the objective. Thus
Z A'OB'= a
construction. With the help of ray diagrams, explain its
working when it forms final image at the least distance of Also, let ZA'£B"=p
distinct vision and at infinity. Deduce expression for .●. Magnifying power.
magnifying power in each case.
p _ tan p
Astronomical telescope. It is a refracting type telescope m = — [●: a, p are small]
a tan a
used to see heavenly bodies like stars, planets, satellites, etc.
Construction. It consists of two converging lenses AB'IB'E _OB'
moimted co-axially at the outer ends of two sliding tubes. A B'tOB' ~ B'E
1. Objective. It is a convex lens of large focal length According to the new Cartesian sign convention,
and a much larger aperture. It faces the distant object. OB' = + /o = focal length of the objective
In order to form bright image of the distant objects, the
aperture of the objective is taken large so that it can
B' E = - Wg = distance of A B' from the eyepiece,
acting as an object for it
gather sufficient light from the distant objects.
2. Eyepiece. It is a convex lens of small focal length m = - A
and small aperture. It faces the eye. The aperture of the “e
eyepiece is taken small so that whole light of the Again, for the eyepiece :
telescope may enter the eye for distinct vision. u = -u^ and v = ~ D
Working, (a) When the final image is formed at the
least distance of distinct vision. As shown in Fig. 9.154,
the parallel beam of light coming from the distant
Objective
A

At infinity

Fig. 9.154 Astronomical telescope focussed for least distance of distinct vision.
PHYSICS-XII

1 1 _ 1 Applying new Cartesian sign convention,


-D
fe
OB' = + ^ = Distance of A’ B' from the objective
or 1 = J- i+A along the incident light
»e D B'E = - f^= Distance of A' B' from the eyepiece
against the incident light
Hence m = - A 1+A
fe D m-

fe
Clearly for large magnifying power, /q » /^. The Clearly for large magnifying power, /q » /^. The
negative sign for the magnifying power indicates that
negative sign for ?n indicates that the image is real and
the final image formed is real and inverted, inverted.

(b) When the final image is formed at infinity :


For Your Knowledge

w
Normal adjustment. As shown in Fig. 9.155, when a
parallel beam of light is incident on the objective, it > In a telescope, the objective has large focal length and
large aperture while the eyepiece has small focal
forms a real, inverted and diminished image A' B' in its length and small aperture.
focal plane. The eyepiece is so adjusted that the image
A' B exactly lies at its focus. Therefore, the final image
is formed at infinity, and is highly magnified and
Flo > A telescope is focussed on the distant object by
varying distance between the objective and the
eye-piece with the help of rack and pinion

ee
inverted with respect to the object. arrangement.

Fr
Magnifying power in normal adjustment. It is > The objective of the telescope should have large
defined as the ratio of the angle subtended at the eye by the aperture because then a much wider beam of light is
incident on it and is converged into a small cone
final image as seen through the telescope to the angle which, on entering the eye, produces sufficient
for
ur
subtended at the eye by the object seen directly, when both illumination on the retina. So even two distant faint
the image and the object lie at injinity. stars which cannot be seen by naked eyes, become
visible through such a telescope.
As the object is very far off, the angle subtended by
ks

it at the eye is practically equal to the angle a > In a telescope, the image is not actually magnified. A
Yo

telescope simply increases the visual angle. The visual


subtended by it at the objective.
oo

angle p for the image is much larger than the visual


Thus
angle a for the object. Consequently, the angular
B

A A OB'=a magnification p / a is quite large.


> In normal adjustment, the distance between the
re

and let ^A'EB'=p objective and the eyepiece = /q + ff


When the final image is formed at the least distance of
ou

Magnifying power,
ad

distinct vision, the magnifying power of the telescope


B tan B is larger than that in the case of normal adjustment
Y

m = —=
a tcin a
-
[v a, pare small angles] because the factor 1+ >1
D

_ A'B7B'£_ OB'
nd

^ An astronomical telescope forms an inverted image.


Re

~ AB'jOB' ~~BE As the celestial objects are oval in shape, so it does not
Fi

matter whether the final image is inverted or erect.

Eyepiece

B
At infinity

Fig. 9.155 Astronomical telescope in normal adjustment.


RAY OPTICS AND OPTICAL INSTRUMENTS 9.97

9.47 terrestria =SCOPE* As the erecting lens does not cause any magni
fication, the angular magnification of the terrestrial
83. What is a terrestrial telescope ? With the help of a telescope is same as that of the astronomical telescope.
ray diagram, explain its working. Write expression for its When the image is formed at infinity,
magnifying power. State its main drawbacks.
fo
Terrestrial telescope. It is a refracting hjpe telescope
used to see erect images of distant earthly objects. It uses an
additional convex lens between objective and eyepiece When the image is formed at the least distance of
for obtaining an erect image. distinct vision.

As shown in Fig. 9.156, the objective forms a real, fo


m=^ fc
1 + ^
inverted and diminished image. A' B' of the distant /el D,
object in its focal plane. Now the erecting lens is held at Drawbacks :
twice its focal length from the focal plane of the
objective. This lens forms a real, inverted and equal 1. The length of the terrestrial telescope is much
size image A"B' of A' B’. This image is now erect with larger than the astronomical telescope. In normal
respect to the distant object. The eyepiece is so adjustment, the length of a terrestrial telescope
adjusted that the image A"& lies at its principal focus. = /q + 4/ + /^, where/is the focal length of the
Hence the final image is formed at infinity and is erecting lens.
2. Due to extra reflection at the surfaces of the erec
highly magnified and erect with respect to the distant
object. ting lens, the intensity of the final image decreases.
A .
Objective Eyepiece

Fig. 9.156 Terrestrial telescope.

9.48 REFLECTING TELESCOPES A beam of light from the distant star is incident on
84. With the help of a ray diagram, explain the cons the objective. Before the rays are focussed at F, a plane
mirror inclined at 45® intercepts them and turns them
truction and working ofa Newtonian reflecting telescope.
towards an eyepiece adjusted perpendicular to the axis
Newtonian reflecting telescope. The first reflecting instrument. The eyepiece forms a highly
telescope was set up by Newton in 1668. As shown in jfjed, virtual and erect image of the distant object.
Fig. 9.157, it consists of a large concave mirror of large
focal length as the objective, made of an alloy of copper 85. With the help of a labelled diagram, explain the
and tin. construction and ivorking of a Cassegrain reflecting
telescope.

Plane ●
Cassegrain reflecting telescope. Fig. 9.158 shows
Parallel rays mirror Cassegrainian type reflecting telescope. It cor\sists of a
from distant F- large concave paraboloidal (primary) mirror having a
object hole at its centre. There is a small convex (secondary)
mirror near the focus of the primary mirror. The
Objective eyepiece is placed on the axis of the telescope near the
(Concave mirror) hole of the primary mirror.
!l> Eyepiece The parallel rays from the distant object are
reflected by the large concave mirror. Before these rays
come to focus at F, they are reflected by the small
convex mirror and are converged to a point / just
Fig. 9.157 Nevrtonian reflecting telescope.
PHYSICS-Xli

Paraboloidal
objective For Your Knowledge
Secondary mirror
> The largest refracting telescope is at the Yerkes
mirror'
Parallel rays f <
Observatory in Wisconsin, USA. It uses an objective
from distant lens of diameter 102 cm.
object
Eyepiece > The largest reflecting telescopes in the world are the pair
of Keck telescopes in Hawaii, USA. They use reflec
ting mirrors of diameter 10 m each.
Fig. 9.158 Cassegrain reflecting telescope, > The largest telescope in India is in Kavalur, Tamil-
nadu. It is a Cassegrain reflecting telescope having
outside the hole. The final image formed at 1 is viewed objective of diameter 2.34 m. It was ground, polished,
through the eyepiece. As the first image at F is inverted set up and is being used by the Indian Institute of
with respect to the distant object and the second image Astrophysics, Bangalore.

w
/ is erect with the respect to the first image f, hence the > Prism binocxilar. It is a double telescope that uses two
final image is inverted with respect to the object. sets of totally reflecting prisms. This makes the final
Let /q be the focal length of the objective and fr that image erect which is very desirable for observations
of the eyepiece.
For the final image formed at the least distance of
distinct vision. Flo on earth. Binoailars are much more compact and
easier to use than a refracting telescope, and allow use
of both eyes.

ee
Fr
fe D Examples based on
For the final image formed at infinity. Telescopes
for
Formulae Used
ur
/, fe 1. Astronomical telescope, (i) In normal adjustment.
86. State some important advantages of a reflecting
ks

type telescope over a refracting type telescope.


Yo

Advantages of a reflecting type telescope. A


oo

reflecting type telescope has the following advantages Distance between objective and eyepiece = /q + 4
over a refracting type telescope : (ii) When the final image is formed at the least
B

1. A concave mirror of large aperture has high distance of distinct vision, m = ^ 1+-^
re

D
gathering power and absorbs very less amount e ^

of light than the lenses of large apertures. The Distance between objective and eyepiece
ou

final image formed in reflecting telescope is


= /o + JiR.
ad

very bright. So even very distant or faint stars = /o +


Y

can be easily viewed.


2. Due to large aperture of the mirror used, the 2. Terrestrial telescope, (i) In normal adjustment.
/o
reflecting telescopes have high resolving power.
nd

m ~
Re

3. As the objective is a mirror and not a lens, it is fe


free from chromatic aberration (formation of Distance between objective and eyepiece
Fi

coloured image of a white object). = /o /e '


4. The use of paraboloidal mirror reduces the where / = focal length of the erecting lens.
spherical aberration (formation of non-point, 3. Galileo's telescope. In normal adjustment, ni = —
blurred image of a point object).
5. A mirror requires grinding and polishing of one Distance between objective and eyepiece - fr- f^
surface only. So it costs much less to construct a
/q Fn
reflecting telescope than a refracting telescope 4. Reflecting telescope, m =
of equivalent optical quality. fe fe
6. A lens of large aperture tends to be very heavy where /q = focal length of concave mirror,
and, therefore, difficult to make and support by f, = focal length of eyepiece.
its edges. On the other hand, a mirror of Units Used
equivalent optical quality weighs less and can

be supported over its entire back surface. Lengths f^, fr, f and D are in all in cm or metre.
RAY OPTICS AND OPTICAL INSTRUMENTS 9.99

Example 132. The magnifying power of an astronomical Calculate the diameter of the moon taking its mean distance
telescope in the normal adjustment position is 100. The from the earth to 38x10 km If the telescope uses an
distance betiveen the objective and the eyepiece is 101 an. eyeyiece of 5 cm focal length, zohat zoould be the distance
Calculate the focal lengths of the objective and the eyepiece. between the two lenses for (i) the final image to be formed at
[CBSE D 04] infinity and (ii) the final image (virtual) at 25 cm from the
[ISCE 97]
/o eye.
Solution. Here m = -^= 100 or /q = 100 fg Solution.
Objective
/o+/. = 101 cm
A
But

or 100/, + /, = 101 D moon

fg = lcm and /q =100 cm. >


a
>
IF
Example 133. An amateur astronomer wishes to estimate d D:
image
Lunar distance
roughly the size of the sun using his crude telescope
consisting of an objective lens offocal length 200 cm and an V
eyepiece of focal length 10 cm. By adjusting the distance of Fig. 9.159
the eye-piece from the objective, he obtains an image of the
sun on a screen 40 cm behind the eyepiece. The diameter of Angle subtended by the image at the objective
the sun's image is measured to be 6.0 cm. What is the = Angle subtended by the moon on the objective
estimate of the sun's size, given that the average earth-sun D.
image _ D.moon
distance is 1.5 x 10^^ m [NCERT ; CBSE OD 19] tana =
d
fo
Solution. Here /g =200 cm, fg =10 cm, a
image
= + 40 cm (for real image) or D moon X d
fo
As — — = 1. 0.92xl0”^m
/. x38xl0^km = 3496 km.
Im

1 1 11 1 ^1-4-3 (i) When the image is formed at infinity, the


fg 40 10 40 40 distance between the two tenses is
40
L= /g + fg =100 cm + 5.0 cm =105 cm.
Ug- ^ cm
or

(ii) When the image is formed by the eyepiece at the


Magnification produced by eyepiece is least distance of distinct vision, we have

mg =
V, ^ 40 =3 Vg=- D = -25 cm, fg=+5cm
w,| 40/3 Using thin lens formula,
Diameter of the image formed by the objective is
-25 5 25
6
d = - cm =2 cm
- fc
3 25
or = -4.17 cm
If D is the diameter of the sun (in m), then the angle 6
subtended by it on the objective will be Therefore, the distance between the two lenses is
D
a =
1.5x10
11
rad L' = /g + I ] = 100 + 4.17 = 104.17 cm.
Example 135. A telescope has an objective of focal length
Amgle subtended by the image at the objective will be
50 cm and eyepiece offocal length 5 cm. The least distance of
a -
size of image _ 2 — rad distinct vision is 25 cm. The telescope is focussed for distinct
fo 200 100 vision on a scale 200 cm away from the object. Calculate (a)
the separation between the objective and eyepiece and (b) the
Angle subtended by the sun on the objective [IIT]
magnification produced.
= Amgle subtended by tlie image at the objective Solution. For the image formed by the objective,
11
D 1.5 X 10
— or D = = 1.5 X 10^ m. we have
11
100 100
1.5x10
»g=-200cm, /g=+50cm
1 1 1
Example 134. A telescope objective of focal length 1m As
forms a real image of the moon 0.92 cm in diameter. fo V
0
u
0
.^100 PHYSICS-XII

1^2. +J_ = J 1 _4-l_ 3 Example 137. A small telescope has an objective lens of
V.
0 /o u.
0 50 200 200 200
focal length 150 cm and eyepiece offocal length 5 cm. What
or V
200
cm is the magnifying power of the telescope for viewing distant
® 3 objects in normal adjiistmeiit ?
Magnification produced by the objective is If this telescope is used to view a 100 m tall tower 3 km away,
v.
0 _ 200 1 xvhat is the height of the image of the tower formed by the
' 3x(-200) 3 objective lens ? [CBSE OD 15]
Solution. Magnifying power of the telescope in
The image formed by objective acts as an object for normal
the eyepiece. So adjustment.

t’g = -25cm, /^ = + 5cm = 30

1 1 1 _ 1 1 -1-5 -6 fe 5

w
100 1
u.
e
V
e 4“ 25 5 ~ 25 Angular size of the tower(a) =
3x1000
rad = —rad
30
25
or = - — cm
6 Angular size of the image (p) = — x 30 rad = 1 rad
Magnification produced by the eyepiece,
-25
Flo 30

m = ~
P

ee
=6 a

H,, -25/6
Height of the image = lx — m =0.05 m.

Fr
100
(fl) The separation between the objective and eyepiece
, , 200 25 425 Example 138. A terrestrial telescope has an objective of
= 70.83 cm.
focal length 180 cm and an eyepiece offocal length 5.0 cm.
for
ur
{b) Magnification produced, The erecting lens has a focal length of3.5 cm. What is the sepa
ration between the objective and the eyepiece 7 What is the
m-m^x m^=-^x6~-2 magnifying power of the telescope ? Can we use the telescope
ks

The negative sign indicates that the final image is for viewing astronomical objects ? [NCERTl
Yo

inverted.
oo

Solution. In a terrestrial telescope, the inverted image


Example 136. A small telescope has an objective lens of formed by the objective is made erect by positioning it at
B

focal length 150 cm and an eyepiece of focal length 5 cm. If erecting lens of focal length /.
this telescope is used to view a 100 m high tower 3 km away,
re

In normal adjustment, the separation between the


find the height of the final image when it is formed 25 cm objective and the eyepiece is
away from the eyepiece. [CBSE D 12]
ou
ad

Solution. Angle subtended by the 100 m tall tower /o + 4/+ h


at 3 km away is
Y

= 180 + 4x3.5 + 5.0 = 199 cm


100 1
a - tana = , = — rad Magnifying power,
3x10^ 30
nd
Re

/o 180 = 36
Let h be the height of image of tower formed by the /. 5
Fi

objective. Then the angle subtended by this image will


also be a and is given by Yes, the telescope can be used to view astronomical
h h objects though there is no need to make the 'inverted'
a = —

fo 150 image of a star 'upright'. But the final image is less


h 1
bright than in an equivalent astronomical telescope
or h = 5 cm because of the extra loss of some light due to reflection
150 ~ 30
and absorption by the erecting lens.
Magnification produced by the eyepiece,
D 1 25 ^
Example 139. (a) A Galilean telescope obtains the final
m^ = l + — = 1+ — =6
5
image erect (like in a terrestrial telescope) without an
intermediate erecting lens. It does so by using a diverging
Height of final image lens for its eyepiece. Show that the angular magnification of
a Galilean telescope is given by the formula : nt = - f^j
= X = 5 X 6 = 30 cm.
(negative sign because is negative).
RAY OPTICS AND OPTICAL INSTRUMENTS 9.101

{b) For a Galilean telescope with /q =150 cm, Solution. Because the final image through is
=-7.5 an, what is the separation between the objective seen at infinity, therefore, virtual image of the object
and the eyepiece ? produced by Lj should lie at the focus of
(c) What is the main disadvantage of this type of If
telescope ? But distance between and ^=~r
Solution, {a) As shown in the ray diagram of 2/ /
Fig. 9.160, the image formed by the objective lies at a Image distance from L^ = f - =—
distance from the eyepiece. So the rays refracted by
the eyepiece are rendered parallel. The final image is /
or V = —, and for L , v =-
formed at infinity. 3 ‘ 3

1 1 1
Vs from As
V u f
^‘stsnt object Objective Image at Eyepiece
Infinity YT7 1

u V f f f f
E B'

6
a

or U = -
f
4
A'
L f
So Lj should be placed at a distance of ^
from the

A
h /o H focus of the objective.
Such an arrangement of two plano-convex lenses is
Fig. 9.160 Galileo's telescope (normal adjustment). preferred to a simple double<onvex lens for an eyepiece
because it reduces chromatic and spherical aberrations.
In normal adjustment, the angular magnification is
given by roblems For Practice
m= —
p _ tanp [●.● a, P are small angles]
a tan a
1. The sum of focal lengths of the two lenses of a
A B'/EB' _ OB' refracting telescope is 105 cm. The focal length of one
lens is 20 times that of the other. Determine die total
A' B'/OB' ~ EB'
magnification of the telescope when the final image is
m=- A formed at infinity. [CBSE OD 14C]
fe (Ans. 20)
Focal length of the objective 2. An astronomical telescope has an angular magni
Focal length of the eyepiece fication of magnitude 5 for distant objects. The
separation between the objective and an eye piece is
The negative sign has been taken because the focal 36 cm and the final image is formed at infinity.
length fg of the diverging lens used for the eyepiece is Calculate the focal length of the objective and the
focal length of the eye piece ? [CBSE SP18]
negative.
(&) In the normal adjustment, the separation (Ans. 30 cm, 6 cm)
between the objective and the eyepiece is 3. A telescope has an objective of focal length 200 cm
and eyepiece of focal length 5 cm. Calculate its
L = /o -1 /J = 150 -! - 7.5 I = 142.5 cm.
magnifying power when the final image is formed
(c) The main disadvantage of a Galilean telescope is {a) at infinity and (fc) at distance of distinct vision.
its limited field of view. This is because the eye cannot
(Ans. 40, 48)
be positioned on the location of the eyering between
the two lenses. 4. A telescope consists of an objective of focal length
150 cm and an eyepiece of focal length 6.0 cm. If the
Example 140. An eyepiece of a telescope consists of two final image is formed at infinity, then calculate :
plano-convex lenses L, and each offocal length f separated (i) the length of tlie tube in this adjustment, and
by a distance of 2 f / 3. Where should be placed relative to
(n) the magnification produced. [CBSE OD 22]
the focus of the objective lens of the telescope so that the final
image through is seen at infinity ? [NCERT] [Ans. (0 156 cm (i7) 25]
9.102 PHYSICS-XII

5. An astronomical telescope is designed to have a 6. For seeing with relaxed eye, final image should be
magnifying power of 50 in normal adjustment. If formed at infinity. This happens when the image
the length of the tube is 102 cm, find the powers of formed by the objective lies at the focus of the
the objective and the eyepiece. (Ans. 1 D, 50 D) eyepiece.
6. A telescope has an objective of focal length 30 cm
For the objective:
and an eyepiece of focal length 3.0 cm. It is focussed
2.0 m=-200cm, /g = + 30cm
on a scale distant 2.0 m. For seeing with relaxed eye,
calculate the separation between the objective and 1 _ 1 ^1 _ 1 1 ^ 17
the eyepiece. (Ans. 38.3 cm)
V.
0 /o Uq 30 200" 600
7. The focal length of the objective of an astronomical or Vn0 = 35.3 cm

telescope is 1.0 m. If the magnifying power of the Distance between the objective and the eyepiece
telescope is 20, find the focal length of the eyepiece = Wq + /g = 35.3 + 3.0 - 38.3 cm.
and the length of the telescope for the relaxed eye.

w
8. m- /o _ 4m P
(Ans. 5 cm, 1.05 m) = 40. Also, m = —
yj, 10 cm a
8. The diameter of the moon is 3.5 x 10^ km and its
distance from the earth is 3.8 x 10^ km. It is viewed Diameter of moon

by a telescope which consists of two lenses of focal


lengths 4 m and 10 cm. Find the angle subtended at
Flo .-. p = 40 a = 40 X

40 X 3.5 X 10^
Radius of lunar orbit

ee
eye by the final image. (Ans. 21.1°) = 36.84x10"^ rad =21.1°.
3.8 X 10^
9. On seeing with unaided eye, the visual angle of

Fr
moon at the eye is 0.06°. The focal lengths of the 9. w =
a
objective and the eyepiece of a telescope are

respectively 200 cm and 5 cm. What will the visual fo 200


for
ur
●●● P = . a = X 0.06° = 2.4°.
angle on seeing through the telescope ? (Ans. 2.4°) fe 5

10. A telescope objective has a focal length of 100 cm. 10. Here/q = 100cm, /q+[ |= 105cm
When the final image is formed at the least distance
ks

.●. I I = 5 cm
of distinct vision, the distance between the lenses is
Yo

For the eyepiece, = - 5 cm, = - D = - 25 cm


oo

105 cm. Calculate the focal length of tire eyepiece


and the magnifying power of the telescope. -1-jL_i. _ J_ 1__£
B

fe 25 5 “25
(Ans. 6.25 cm, 20)
re

11. A refracting telescope has an objective of focal or


/p = 6.25 cm
length 1 m and an eyepiece of focal length 20 cm. fo f
The final image of the sun 10 cm in diameter is m = \+dJL
ou

D,
ad

formed at a distance of 24 cm from the eyepiece. e ^

What angle does the sun subtend at the objective ? 100 6.25
Y

= 1+ = 20.
(Ans. 0.0455 rad) 6.25 I, 25 .
12. A reflecting type telescope has a concave reflector 11. Let and li^ be tire sizes of the images formed by
nd
Re

of radius of curvature 120 cm. Calculate focal the objective and the eyepiece respectively. Then
length of eyepiece to secure a magnification of 20.
Fi

tan a =
(Ans. 3 cm) fo 100
HINTS
D'
Also =
1. Given 4-f/^ = 105and/^ = 20/^ fe
204 + /^. = 105 => /^ = 5 cm 10 24 44
/q = 20x5 = 100 cm
or
20 20
Hence, m- = 20. 50
fe 5 or
^ = 11
cm

2. /« = A=5 ^ fo=^fe or a =
50
— = 0.0455 rad.
11x100 22

But /„+/,= 36 6^ = 36
/o_«o/2_ 120/2
Hence, /^ = 6 cm and = 30 cm, 12- fe = VI m 20
= 3 cm.

You might also like